You are on page 1of 130

Table of contents

Table of Contents
CHAPTER 01: COMPOUND INTEREST (WITHOUT FORMULA)............................ 1
Introduction: ....................................................................................................................................... 1
Solved Examples 1.1: .............................................................................................................. 1
Unsolved Exercise 1.1: ............................................................................................................ 2
Important Results: ........................................................................................................................ 3
Solved Examples 1.2: .............................................................................................................. 4
Unsolved Exercise 1.2: ............................................................................................................ 4
Previous Board Question: ........................................................................................................ 5
Miscellaneous Exercise: .......................................................................................................... 5
Answer to Unsolved Exercise: ................................................................................................. 6

CHAPTER 02: COMPOUND INTEREST (USING FORMULA) ................................. 7


Introduction: ....................................................................................................................................... 7
Solved Examples 2.1: .............................................................................................................. 7
Unsolved Exercise 2.1: ............................................................................................................ 8
Solved Examples 2.2: ............................................................................................................ 10
Unsolved Exercise 2.2: .......................................................................................................... 10
Previous Board Question: ...................................................................................................... 12
Miscellaneous Exercise: ........................................................................................................ 12
Answer to Unsolved Exercise: ............................................................................................... 13

CHAPTER 03: SALES TAX AND VAT ................................................................... 14


Introduction: ..................................................................................................................................... 14
Sales Tax (TradeTax): ............................................................................................................. 14
Some important terms: ............................................................................................................... 14
Computation of Sales Tax: ......................................................................................................... 14
Solved Examples 3.1: ............................................................................................................ 15
Unsolved Exercise 3.1: .......................................................................................................... 15
Solved Examples 3.2: ............................................................................................................ 16
UnSolved Exercise 3.2: ......................................................................................................... 17
VAT (Value Added Tax): ............................................................................................................ 18
Solved Examples 3.3: ............................................................................................................ 19
UnSolved Exercise 3.3: ......................................................................................................... 20
Miscellaneous Exercise: ........................................................................................................ 21
Previous Board Question: ...................................................................................................... 22
Answer to Unsolved Exercise: ............................................................................................... 23

CHAPTER 04: BANKING (SAVING AND RECURRING ACCOUNT) .................... 25


INTRODUCTION: ......................................................................................................................................... 25
Solved Examples 4.1: ............................................................................................................ 26
Unsolved Exercise 4.1: .......................................................................................................... 28
Recurring Deposit Account (R.D. Account):............................................................................... 31
Volume 1 of 2

Universal Tutorials X ICSE Maths

Computing maturity value of a Recurring Deposit Account: ...................................................... 31


Solved Examples 4.2: ............................................................................................................ 31
Unsolved Exercise 4.2: .......................................................................................................... 32
Previous Board Question: ...................................................................................................... 32
Miscellaneous Exercise: ........................................................................................................ 35
Answer to Unsolved Exercise: ............................................................................................... 36

CHAPTER 05: SHARES AND DIVIDEND ............................................................... 38


Shares: ....................................................................................................................................... 38
Solved Examples 5.1: ............................................................................................................ 38
Unsolved Exercise 5.1: .......................................................................................................... 39
Solved Examples 5.2: ............................................................................................................ 40
Unsolved Exercise 5.2: .......................................................................................................... 42
Previous Board Question: ...................................................................................................... 43
Miscellaneous Exercise: ........................................................................................................ 44
Answer to Unsolved Exercise: ............................................................................................... 46

CHAPTER 06: LINEAR INEQUATIONS(IN ONE VARIABLE) ............................... 47


Introduction: ..................................................................................................................................... 47
Replacement Set and Solution Set: ................................................................................................ 47
Graphical Representation:............................................................................................................... 48
Solved Examples 6.1: ............................................................................................................ 49
Unsolved Exercise 6.1: .......................................................................................................... 50
Previous Board Question: ...................................................................................................... 51
Miscellaneous Exercise: ........................................................................................................ 52
Answers to unsolved exercise: .............................................................................................. 53

Universal Tutorials X ICSE Maths

Volume 1 of 2

Table of contents

CHAPTER 07: QUADRATIC EQUATIONS ............................................................. 56


Introduction: ..................................................................................................................................... 56
Zero product Rule:........................................................................................................................... 56
Solved Examples 7.1: ............................................................................................................ 56
Unsolved Exercise 7.1: .......................................................................................................... 57
Solved Examples 7.2: ............................................................................................................ 58
Unsolved Exercise 7.2: .......................................................................................................... 60
Unsolved Exercise 7.3: .......................................................................................................... 62
Previous Board Question: ...................................................................................................... 63
Miscellaneous Exercise: ........................................................................................................ 64
Answer to the Unsolved Exercise: ......................................................................................... 64

CHAPTER 08: SOLVING PROBLEMS (BASED ON QUADRATIC EQUATIONS) 66


Introduction: ..................................................................................................................................... 66
Problems based on Numbers:.................................................................................................... 66
Solved Examples 8.1: ............................................................................................................ 66
Unsolved Exercise 8.1: .......................................................................................................... 67
Solved Examples 8.2: ............................................................................................................ 68
Unsolved Exercise 8.2: .......................................................................................................... 68
Solved Examples 8.3: ............................................................................................................ 69
Unsolved Exercise 8.3: .......................................................................................................... 70
Solved Examples 8.4: ............................................................................................................ 71
Unsolved Exercise 8.4: .......................................................................................................... 72
Previous Board Question: ...................................................................................................... 72
Miscellaneous Exercise: ........................................................................................................ 73
Answer to the Unsolved Exercise: ......................................................................................... 75

CHAPTER 9: CO-ORDINATE GEOMETRY: REFLECTION ................................... 77


Introduction: ..................................................................................................................................... 77
Solved Examples 9.1: ............................................................................................................ 79
Unsolved Exercise 9.1: .......................................................................................................... 81
Previous Board Question: ...................................................................................................... 83
Miscellaneous Exercise: ........................................................................................................ 84
Answer of Unsolved Exercise: ............................................................................................... 86

CHAPTER 10: RATIO AND PROPORTION............................................................ 88


Ratio: ............................................................................................................................................... 88
Increase (or decrease) in a ratio: ............................................................................................... 88
Commensurable and Incommensurable quantities:................................................................... 88
Comparison of Ratios: ................................................................................................................ 89
Composition of Ratios: ............................................................................................................... 89
Solved Examples 10.1: .......................................................................................................... 90
Unsolved Exercise 10.1: ........................................................................................................ 91
Proportion: ....................................................................................................................................... 93
Volume 1 of 2

Universal Tutorials X ICSE Maths

Solved Examples 10.2: .......................................................................................................... 93


Unsolved Exercise 10.2: ........................................................................................................ 95
Some important properties of proportion: .................................................................................. 96
Solved Examples 10.3: .......................................................................................................... 97
Unsolved Exercise 10.3: ........................................................................................................ 99
Previous Board Question: ...................................................................................................... 99
Miscellaneous Exercise: ...................................................................................................... 100
Answers to Unsolved Exercise: ........................................................................................... 101

CHAPTER 11: REMAINDER AND FACTOR THEOREMS ................................... 103


Introduction .................................................................................................................................... 103
Factor Theorem: ....................................................................................................................... 103
Solved Examples 11.1: ........................................................................................................ 104
Unsolved Exercise 11.1: ...................................................................................................... 105
Solved Examples 11.2: ........................................................................................................ 105
Unsolved Exercise 11.2: ...................................................................................................... 106
Previous Board Question: .................................................................................................... 107
Miscellaneous Exercise: ...................................................................................................... 107
Answers to Unsolved Exercise: ........................................................................................... 108

CHAPTER 12: MATRICES .................................................................................... 110


Introduction: ................................................................................................................................... 110
Matrix: ....................................................................................................................................... 110
Additive Identity: ....................................................................................................................... 112
Additive Inverse: ....................................................................................................................... 112
Solved Examples 12.1: ........................................................................................................ 112
Unsolved Exercise 12.1: ...................................................................................................... 113
Multiplication of a matrix by a scalar (real number) ................................................................. 114
Solved Examples 12.2: ........................................................................................................ 114
Unsolved Exercise 12.2: ...................................................................................................... 115
Multiplication of Matrices: ......................................................................................................... 116
Solved Examples 12.3: ........................................................................................................ 117
Unsolved Exercise 12.3: ...................................................................................................... 118
Previous Board Question: .................................................................................................... 120
Miscellaneous Exercise: ...................................................................................................... 121
Answers to Unsolved Exercise: ........................................................................................... 122

Universal Tutorials X ICSE Maths

Volume 1 of 2

Chapter 01: Compound Interest (Without Formula)

Chapter 01: Compound Interest (Without Formula)


Introduction:
Interest (Simple Interest):
z
z

Principal Rate Time


100
Amount = Principal + Interest

S.I. =

PR T
100
i.e. A = P + I

i.e. I =

Note: When we say, interest, it always means simple interest.

Compound Interest (C.I.):


z

z
z

Money is said to be lent at compound interest, when the interest, which has become due at
the end of a certain fixed period (one year, half year etc. as given) is not paid to the money
lender, but is added to the sum lent.
The amount thus obtained becomes the principal for the next period. This process is
repeated until the amount for the last period is found.
The difference between the final amount and the original principal is called the compound
interest.
i.e. Compound Interest = Final Amount Original Principal
CI = A P

Note:
1) In case of Simple interest, the principal remains constant throughout. But in case of compound
Interest, the principal keeps on changing every year (or any other fixed period)
2) When the interest is compounded yearly, the principal changes every year.
3) When the interest is compounded half yearly, the principal changes every six months.

SOLVED EXAMPLES 1.1:


1) Find the sum, invested at 8% compounded annually, on which the interest for third year
exceeds the interest of first year by `199.68.
Sol: Let the sum (principal) = `100
100 8 1
C.I. of 1st year = `
= `8
100
Amount of 1st year = `100 + 8 = `108 or Principal for 2nd year = `108
108 8 1
C.I. of 2nd year =
= `8.64 or Principal for 3rd year = 108 + 8.64 = `116.64
100
116.64 8 1
= `9.3312
C.I. of 3rd year = `
100
Difference of compound interest of 3rd and 1st year = `9.3312 8.00 = `1.3312
As interest of 3rd year exceeds interest of 1st year by `1.3312,
Principal = `100
100 199 .68
When difference is `199.68, Sum =
= `15, 000
1.3312
Volume 1 of 2

Universal Tutorials X ICSE Maths

2) Find the amount and the compound interest on `10,000 at 8 per cent per annum and in 1
year; interest being compounded half-yearly.
1
1
year: P = `10, 000; R = 8% and T =
year
Sol: For 1st
2
2
P R T
10,000 8 1
I=
I=`
= `400 `
100
100 2
And, A = P + I = `10, 000 + `400 = `10, 400
1
1
For 2nd
year: P = `10,400; R = 8% and T =
year.
2
2
10,400 8 1
I=`
= `416
100 2
And, A = P + I = `10,400 + 416 = `10,816
Required amount = `10,816
Compound interest = A P = `10,816 10,000 = 816

UNSOLVED EXERCISE 1.1:


CW Exercise:
1) Calculate the amount and the compound interest on:
i) `12,000 for 2 years at 5% per annum compounded annually.
ii) `8,000 for 1 years at 10% per annum compounded half-yearly.
2) Calculate the amount and the compound interest on: `7,500 is 2 years when the rates of interest
for successive years are 8% and 10% respectively.
3) A man lends `5,500 at the rate of 8% per annum. Find the amount if the interest is compounded
half-yearly and the duration is one year.
4) A man borrows `10,000 at 5% per annum compound interest. He repays `35% of the sum
borrowed at the end of the first year and 42% of the sum borrowed at the end of the second
year. How much must he pay at the end of the third year in order to clear the debt?
5) Rachana borrows `12,000 at 10 per cent per annum interest compounded halfyearly. She
repays `4,000 at the end of every six months. Calculate the third payment she has to make at
the end of 18 months in order to clear the entire loan.
6) A sum of `13,500 is invested at 16% Per annum compound interest for 5 years. Calculate:
i) the interest for the first year.
ii) the amount at the end of the first year.
iii) the interest for the second year, correct to the nearest rupee.
7) A sum of `5,000 is lent for 3 years at compound interest. If the rates of interest for the
successive years are 10%, 12% and 15% respectively, find the difference between the
compound interest for the first year and the compound interest for the third year.
8) On a certain sum of money, invested at the rate of 10 per cent per annum compounded
annually, the interest for the first year plus the interest for the third year is `2,652. Find the sum.
9) It is estimated that every year the value of a machine depreciates by 20% of its value at the
beginning of the year. Calculate:
i) the value of the machine at the end of three years, if its original value is `35,000
ii) the original value of the machine, if its value after two years is `10,240
10) Find the sum that will amount to `4,928 in 2 years at compound interest, if the rates for the
successive years are 10 per cent and 12 per cent respectively.
11) Without using formula, find the sum on which the difference between the simple interest and the
compound interest at the rate of 8% per annum compounded annually be `64 in 2 years.
2

Universal Tutorials X ICSE Maths

Volume 1 of 2

Chapter 01: Compound Interest (Without Formula)

HW Exercise:
1) Calculate the amount and the compound interest on `8,000 for 1 years at 10% per annum
compounded yearly.
2) Calculate the amount and the compound interest on `12,500 in 3 years when the rates of
interest for successive years are 8%, 10% and 10% respectively.
3) A man borrows `8,500 at 10% compound interest. If he repays `2,700 at the end of each year,
find the amount of the loan outstanding at the beginning of the third year.
4) Calculate the compound interest for the second year and the third year on `16,000 invested for 5
years at 10% per annum.
5) Saurabh invests `48,000 for 7 years at 10% per annum compound interest. Calculate:
i) the interest for the first year.
ii) the amount at the end of the second year.
iii) the interest for the third year.
6) Mr. Mehta invested `8,000 every year at the beginning of the year, at 10% per annum compound
interest. Calculate his total savings at the beginning of the third year.
7) Mrs. Suneeta saves `8,000 every year and invests it at the end of the year at 10% per annum
compound interest. Calculate her total amount of savings at the end of the third year.
8) During every financial year, the value of a machine depreciates by 12%.Find the original cost of
a machine which depreciates by `2,640 during the second financial year of its purchase.
9) Mohit borrowed `7,500 from Sohan at 8% per annum compound interest. After 2 years, Mohit
gave `6,248 and an old T.V. set to clear his account. Find the cost of the T.V. set.
10) Calculate the difference between the compound interest and the simple interest on `10,000 in 2
years at 5% per annum.
11) Govind borrows `18,000 at 10% simple interest. He immediately invests the money borrowed at
10% compound interest compounded half-yearly. How much money does Govind gain in one
year?

Important Results:
On the same sum and at the same rate of interest:
nd
st
z The C.I. of 2 year is always more than the C.I. of 1 year
rd
nd
z C.I. of 3 year is more than C.I. of 2 year
th
th
z C.I. of 8 year is more than C.I. of 7 year and so on
The difference between the compound interest for any two consecutive years is the interest of
one year on the C.I. of the preceding year.
Similarly: the difference between the amounts for any two consecutive years is also the interest
of one year on the amount of the preceding year.

Relation between Simple Interest (S.I.) and Compound Interest (C.I.)


z

S.I. on a certain sum and at a certain fixed rate percent is the same every year. Eg.
st
If S.I on a certain sum is `500 for the 1 year, then for every year the S.I on that sum will
be `500 only, provided the rate of interest is also the same every year.
If S.I on a certain sum is `600 in 3 years; the S.I. on that sum for one year will be
600
= `200 and so on.
3
S.I. and C.I. are the same for the first year on the same sum and at the same rate percent.

Volume 1 of 2

Universal Tutorials X ICSE Maths

Note: When the interest is calculated yearly.


If the C.I. of any particular year is `x; then the C.I. for the next year on the same sum and at the
same rate = `x + Interest for one year on `x.
In the same way, if the amount at C.I. in a particular year is `x then the amount for the next year.
On the same sum and at the same rate = `x + Interest on `x for one year.

SOLVED EXAMPLES 1.2:


1) A man invests `1,200 for two years at compound interest. After one year his money
amounts to `1,275. Find the interest for the second year correct to the nearest paise.
Sol: Let the sum of `1,200 invested for 2 years of compound interest x%.
Now, C.I. of 1st year = `

1,200 x 1
= `12x
100

Amount after one year = `(1,200 + 12x) = `1,275


12x = 75

or

x = 6.25%

Now, interest for 2nd year = `

1,275 6.25
= `79.69
100

UNSOLVED EXERCISE 1.2:


CW Exercise:
1) A sum is invested at compound interest compounded yearly. If the interests for two successive
years be `570 and `741; calculate the rate of interest.
2) Mohit invests `8,000 for 3 years at a certain rate of interest, compounded annually. At the end of
one year it amounts to `9, 440. Calculate
i) the rate of interest per annum
ii) the amount at the end of second year
iii) the interest accrued in third year.
3) A certain sum amounts to `5,292 in two years and `5,556.60 in three years, interest being
compounded annually. Find:
i) the rate of interest. ii) the original sum
4) The simple interest on a certain sum computes to `256 in 2 years, whereas, the compound
interest on the same sum at the same rate and for the same time computes to `276.48. Find the
rate percent and the sum.
5) The interest charged on a certain sum is `720 for one year and `1,497.60 for two years. Find,
whether the interest is simple or compound. Also calculate the rate percent and the sum.
6) A sum of money placed out at compound interest amounts to `20,160 in 3 years and `24,192 in
4 years. Calculate:
i) the rate of interest. ii) Amount in 2 years
iii) amount in 5 years.
7) A sum of `24,000 is lent for 2 years at compound interest, the rate of interest being 10% for the
first year and 12% for the second year. The borrower returns some money at the end of the first
year and on paying `12,768 at the end of the second year the total debt is cleared. Calculate the
amount of money returned at the end of the first year.
8) The cost of a machine depreciated by `4,752 during the second year and by `4,181.76 during
the third year. Calculate:
i) the rate of depreciation;
ii) the original cost;
iii) the cost at the end of the third year.
HW Exercise:
1) A certain sum money is put at compound interest compounded half-yearly. If the interests for two
successive half-years amount to `650 and `760.50; find the rate of interest.
4

Universal Tutorials X ICSE Maths

Volume 1 of 2

Chapter 01: Compound Interest (Without Formula)

2) Geeta borrowed `15,000 for 18 months at a certain rate of interest compounded semi-annually.
If at the end of six months it amounted to `15,600; calculate:
i) the rate of interest per annum
ii) the total amount of money that Geeta must pay at the end of 18 months in order to clear the
account.
3) The compound interest, calculated yearly, on a certain sum of money for the second year is
`1,089 and for the third year it is `1,197.90. Calculate rate of interest and the sum of money.
4) On a certain sum and at a certain rate percent the simple interest for first year is `270 and the
compound interest for the first two years is `580.50. Find the sum and the rate percent.
5) The compound interest, calculated yearly, on a certain sum of money for the second year is
`864 and for the third year is `933.12, calculate the rate of interest and the compound interest,
on the same sum and at the same rate, for the fourth year.
6) `8,000 is lent out at 7% compound interest for 2 years. At the end of the first year `3,560 are
returned. Calculate:
i) the interest paid for the second year.
ii) the total interest paid in two years
iii) the total amount of money paid in two years to clear the debt.
7) The population of a town increases 10% every 3 years. If the present population of the town is
72,600; calculate:
i) its population after 6 years;
ii) its population 6 years ago.
8) The compound interest, calculated yearly, on a certain sum of money for the second year is
`880 and for the third year is `968. Calculate the rate of interest and the sum of money.

PREVIOUS BOARD QUESTION:


1) Mr. Kumar borrowed `15,000 for two years. The rate of interest for the two successive years are
8% and 10% respectively. If he repays `6,200 at the end of the first year, find the outstanding
amount at the end of the second year.
[2011]
2) Nikita invests `6000 for two years at a certain rate of interest compounded annually. At the end
of first year it amounts to `6720. Calculate
[2010]
i) the rate of interest
ii) the amount at the end of the second year.
3) Mr. Dubey borrows `1,00,000 from State Bank of India at 11% per annum compound interest.
He repays `41,000 at the end of first year and `47,700 at the end of the second year. Find the
amount outstanding at the beginning of the third year.
[2009]
4) Ramesh invest `12800 for three years at the rate of 10% per annum compound interest. Find:
i) The sum due to Ramesh at the end of the first year.
ii) The interest he earns for the second year.
iii) The total amount due to him at the end of the third year.
[2007]
5) A person invests `10,000 for two years at a certain rate of interest compounded annually. At the
end of one year this sum amounts to `11,200. Calculate:
[2006]
i) the rate of interest per annum
ii) the amount at the end of the second year.

MISCELLANEOUS EXERCISE:
1) Calculate the amount and the compound interest on `3,500 at 10% per annum in 2 year.
2) Find the compound interest, to the nearest rupee, on `3,600 for 2 years and 4 months at 10%
p.a. compound yearly.
3) Mr. Gupta invests `3,20,000 for 2 years at 12.5% p.a. compounded annually. If the income tax at
the rate of 20% is deducted at the end of each year, on interest accrued. Find the amount he
received at the end of 2 years.

Volume 1 of 2

Universal Tutorials X ICSE Maths

4) Find the money, invested at 10% compounded annually, on which the sum of interest for the first
year and for the third year is `1,768.
5) A sum of money is invested at compound interest payable annually. The interest in first two
successive years is `1,350 and `1,440 respectively. Find:
i) the rate of interest
ii) the original sum
iii) the interest earned in the third year
6) A sum of money amounts to `46,305 in 1 year and to `48,620.25 in 1 years at compound
interest, compounded semi-annually. Find the sum and the rate of interest per annum.
7) The cost of a machine is `32,000. Its value depreciates at the rate of 5% every year. Find the
total depreciation in its value by the end of 2 years.
8) Find the sum, invested at 8% compounded annually, on which the interest for the third year
exceeds the interest of the first year by `199.68.
9) Find, without using formula, the difference between compound interest and simple interest on
`16,000 at 5% per annum and in 2 years.
10) The compound interest, compounded annually, on a certain sum is `9,680 in second year and is
`10,648 in third year. Calculate.
iii) the interest for 1st year.
i) the rate of interest
ii) the interest for 4th year
11) A sum of money lent at C.I compounded yearly, amounts to `9,680 in 2 years and `10,648 in 3
years. Calculate:
i) the rate of interest
ii) the amount in 4 years iii) the amount in one year.

ANSWER TO UNSOLVED EXERCISE:


CW Exercise 1.1:
1) i) 13,230 and `1,230
ii) 9,261 and `1261
2) `8,910 and `1,410
3) `5,948.80 4) `3,307.50 5) `5,281.50
6) i) ` 2,160 (ii) `15,660 (iii) `2,506
7) `424
8) `12,000
9) i) `17,920 (ii) `16,000 10) `4,000
11) `10,000
HW Exercise 1.1:
1) `9,240 and `1,240
2) `16,335, `3,835
3) `4,615
4) `1,760, `1,936.
5) i) `4,800 (ii) `58,080 (iii) `5,808
6) `26,480
7) `26,480
8) `25,000
9) `2,500
10) `25
11) `45
CW Exercise 1.2:
2) i) `18% (ii) `11,139.20 (iii) `2,005.06
1) 30%
3) i) 5% (ii) `4,800
4) 16% `800
5) Compound 8% `9,000 6) i) 20% (ii) 16,800 (iii) 29,030.40
7) `15,000
8) i)12% (ii) 45,000 (iii) 30,666.24
HW Exercise 1.2:
1) 34%
2) i) 8% (ii) 16,872.96 3)10%, `9,908
4) `1,800; 15%
5) 8%, `1,007.77
6) i) `350 (ii) `910 (iii) `8,910
7) i) 87,846 (ii)60,000
8) 10% and `8000.
Previous Year Board Question:
1) `11,000
2) i) r = 12% (ii) `7526.40
3) `30,000
4) i) 14080 ii) 1408 iii) 17036.80
5) i) 12% (ii) Rs12,544
Miscellaneous:
1) `4,235 and `735
2) `901.20
2
3) `3,87,200 4) `8,000 5) i) 6 % (ii) `20,250 (iii) `1,536
3
6) `42,000 and 10%
7) `3,120
8) `15,000 9) `40
10) i) 10% (ii) `8,800 (iii) 11,712.80
11) i) 10% (ii) `11,712.80 (iii) `8,800
6

Universal Tutorials X ICSE Maths

Volume 1 of 2

Chapter 02: Compound Interest (Using Formula)

Chapter 02: Compound Interest (Using Formula)


Introduction:
When the interest is compounded yearly, the formula for finding the amount is:
n

A = P 1 +

100
Where A = Amount; P = Principal; r = rate of interest compounded yearly and n = number of years
Since, Compound interest = Amount Principal
n

C.I. = A P = P 1 +
p
100

r
C.I. = P 1 +
1
100

When the interest is not compounded yearly, the formula for finding the amount is:
mn

A = P 1 +

m 100

Where A = Amount, P = Principal, r = rate of interest, n = number of years, m = compounding/


year (Where m = 2 for yearly compounding; m = 4 for quarterly compounding; m = 12 for
monthly compounding and m = 365 for daily compounding)
When the rates for successive years are different then:

r
r
r

A = P 1 + 1 1 + 2 1 + 3 .....
100
100
100

Where r1%, r2%, r3% are the rates for successive years.
No. of years
Compounded yearly
Compounded halfyearly
1 2

i) n = 1 year

ii) n= 1

1
years
2

A = P 1 +

2 100

A = P 1 +

100

1 2

r
r
2

A = P 1 +

1 +
100 2 100
2

A = P 1 +

2 100

iii) n = 2 years

A = P 1 +

100

1
iv)n = 2
years
2

r
r
2

A = P 1 +

1 +

2
100
100

3 2

2
A = P 1 +

2 100

1 2

2 2

5 2

2
A = P 1 +

2
100

SOLVED EXAMPLES 2.1:


1) Calculate the amount when a sum of `4,800 is invested at 8% per annum for 4 years, the
C.I. being compounded half-yearly. Do not use mathematical tables, use the necessary
information from the following:
(1.08)4 = 1.3605;
(1.04)8 = 1.3686;
(1.08)8 = 1.8509;
(1.04)4 = 1.1699
Volume 1 of 2

Universal Tutorials X ICSE Maths

Sol: Given P = `4,800, r = 8% compounded half-yearly and n = 4 years


n 2

4 2

r
8

= 4,800 1 +
= 4,800 (1.04)8
A = P 1 +

2 100
2 100

= 4,800(1.3686) = `6,569.28
2) Calculate the amount and the compound interest on `12, 000 in 3 years when the rates of
interest for successive years are 8%, 10% and 15% respectively.

r
r
r

Sol: A = P 1 + 1 1 + 2 1 + 3
100 100 100
8
10
15

A = `12,000 1 +
1 +
1 +
= `16,394.40
100
100
100

Required amount = `16,394.40


C.I. = `16,394.40 `12,000 = `4,394.40

UNSOLVED EXERCISE 2.1:


CW Exercise:
1) Find the amount and the compound interest on `12,000 in 3 years at 5% compounded annually.
2) Calculate the compound interest accrued on `6,000 in 3 years, compounded yearly, if the rates
for the successive years are 5%, 8% and 10% respectively.
3) What sum of money will amount to `5,445 in 2 years at 10% per annum compound interest?
4) What principal will amount to `9,856 in two years, if the rates of interest for successive years are
10% and 12% respectively?
5) At what rate per cent per annum compound interest will `6,000 amount to `6,945.75 in 3 years?
6) A person invests `5,000 for three years at a certain rate of interest compounded annually. At the
end of two years this sum amounts to `6,272. Calculate:
i) the rate of interest per annum.
ii) the amount at the end of the third year.
7) In how many years will `7,000 amount to `9,317 at 10 per annum compound interest?
8) Divide `28,730 between A and B so that when their shares are lent out at 10 per cent compound
interest compounded per year, the amount that A receives in 3 years is the same as what B
receives in 5 years.
9) If the interest is compounded half-yearly, calculate the amount when principal is `7,400; the rate
of interest is 5% per annum and the duration is one year.
10) Find the difference between the compound interest compounded yearly and half-yearly on
`10,000 for 18 months at 10% per annum.
11) Ashok invests a certain sum of money at 20% per annum, interest compounded yearly. Geeta
invests an equal amount of money at the same rate of interest per annum compounded halfyearly. If Geeta gets `33 more than Ashok in 18 months, calculate the money invested by each.
12) Calculate the C.I. on `3,500 at 6% per annum for 3 years, the interest being compounded halfyearly. Do not use mathematical tables. Use the necessary information from the following:
(1.06)3 = 1.191016; (1.03)3 = 1.092727; (1.06)6 = 1.418519; (1.03)6 = 1.194052
13) On what sum of money will the difference between compound interest compounded annually
and compound interest compounded half-yearly be `63 in 18 months and at 10% per annum.
HW Exercise:
1) Calculate the amount of `15,000 in 2 years, compounded annually, if the rates for the
successive years are 8% and 10% respectively.
2) On what sum of money will the compound interest for 2 years at 5 per cent per annum amount to
`768.75?
8

Universal Tutorials X ICSE Maths

Volume 1 of 2

Chapter 02: Compound Interest (Using Formula)

3) At what rate per cent compound interest, does a sum of money become 1.44 times of itself in 2
years?
4) Find the time, in years, in which `4,000 will produce `630.50 as compound interest at 5 per cent
compounded annually.
5) A sum of `34,522 is divided between Rohit and Rajesh, 18 years and 21 years old respectively
in such a way that if their shares be invested at 5% per annum compound interest, both will
receive equal money at the age of 30 years. Find the shares of each out of `34,522.
6) What sum of money will amount to `7,123.20 in one and a half years at 12% per annum
compounded yearly?
7) In what time will `1,500 yield `496.50 as compound interest at 20% per year compounded semiannually?
8) Find the difference between compound interest and simple interest on `12,000 and in 1 years
at 10% compounded yearly.

Other applications of the Formula:


Growth:
z

The word growth can be used several ways, such as:


The growth of industries in the country
The rapid growth of plants (or, inflation), etc.
When the growth of industries (or, production in any particular industry) is taken into
consideration
n

The formula A = P 1 +
can be used as:
100
r

Production after n year = Initial (original) production 1 +

100

Where rate of growth in production is r%

In the similar manner, the formula A = P 1 +

100
Can be used for the growth of plants, growth of inflation, etc

Depreciation:
z

Let the cost of a machine depreciates by r% every year, then its value after n years can be
obtained by the formula:
n

Value after n years = Present value 1


.
100

Also, the present value of the machine = Its value n years ago 1
.
100

Population Problems:
z

When the population of a town, city, village etc. increases at a certain rate per year, then
n

the formula, A = P 1 +
can be used as:
100

Volume 1 of 2

Universal Tutorials X ICSE Maths

10

Population after n years = Present population 1 +

100

Present population = Population n years ago 1 +

100

i.e. for the population at two different times, the former population will come (in the
formula) at the place of principal (P) and the later population will come at the place of
amount (A).

SOLVED EXAMPLES 2.2:


1) The difference between the compound interest for a year payable half-yearly and the
simple interest on a certain sum of money lent out at 10% for a year is `15. Find the sum
of money lent out.
Sol: Let the principle = P, r = 10%, n = 1
P r n
P 10 1
P
=
=
= 0.1P
Simple interest basis =
100
100
10
r

On C.I. A = P 1 +

100

mn

10

= P 1 +

2
100

21

= P(1.05)2 = 1.1025P

C.I. = A P = 1.1025P P = 0.1025P


Difference of interest = 0.1025P 0.1P
15 = 0.0025P
P = 6,000
Sum lent out was `6,000/
2) The population of a town is 24000. It is increasing at the rate of 5% every year. What will
be the population of this town after 3 years? Also find the increase in population in 3
years.
n

R
5

Sol: For growth, V = V0 1 +


= 24000 1 +

100
100
3

21
= 24000 1 +
= 24000

20

20

21 21 21

= 27,783
20 20 20
Population of the town after 3 years = 27,783
And increase in population = 27783 24000 = 3783

= 24000

UNSOLVED EXERCISE 2.2:


CW Exercise:
1) The cost of a machine is supposed to depreciate each year at 12% of its value at the beginning
of the year. If the machine is valued at `44,000 at the beginning of 1995, find its value.
i) at the end of 1996
ii) at the beginning of 1994
2) The value of a machine depreciated for two years at the rate of 10% per year and then in the
third year it increased by 15%. Find the original value (initial cost of the machine), if its value at
the end of 3 years is `41,917.50.
3) According to census taken towards the end of the year 1994, the population of a rural town was
found that to be 32,000. The census authority also found that the population of this particular
town had a growth of 5% per annum. In how many years after 1994 did the population of this
town reach 37,044?
10

Universal Tutorials X ICSE Maths

Volume 1 of 2

Chapter 02: Compound Interest (Using Formula)

11

4) The height of a plant is 80 cm and it is expected to grow at the rate of 20% every month. Find its
height after 3 months.
5) The population of a town increases by 5% every year. What was the population in 1982, if in
1985 it was 1,85,220?
6) A sum of money lent out at C.I. at a certain rate per annum becomes three times of itself in 10
years. Find in how many years will the money become twentyseven times of itself at the same
rate of interest p.a.
7) Amol bought a plot of land for `70,000 and a car for `32,000 on the same day. The value of the
plot appreciates uniformly at the rate of 10% every year while the value of the car depreciates by
12.5% for the first year and 10% for the second year. If Amol sells the plot of land as well as the
car after 2 years, what will be the profit or loss on the whole?
729
of itself in one year,
8) Determine the rate of interest per annum for a sum that becomes
625
compounded half-yearly.
9) Anuj and Yash each lent the same sum of money for 2 years at 8% simple interest and
compound interest respectively. Yash received `64 more than Anuj. Find the money lent by each
and interest received.
HW Exercise:
1) The value of a machine is estimated to be `27,000 at the end of 1994 and `21,870 at the
beginning of 1997, supposing it depreciates of 1997. Supposing it depreciates at a constant rate
per year of its value at the beginning of the year, calculate:
i) the rate of depreciation.
ii) the value of the machine at the end of 1997 and at the beginning of 1994.
2) The population of a town decreased by 12% during 1998 and then increased by 8% during 1999.
Find the population of the town, at the beginning of 1998, if at the end of 1999 its population was
2,85,120.
3) The number of radiations given out by a certain sample of radio-active substance is halved in 3
years. Find in how many years will the number of radiations emitted by the same sample of
radio-active substance be one-eight at the same rate.
4) Find the effective rate per cent per annum equivalent to a nominal rate of 10% per annum
interest payable half-yearly?
1
5) A property decreases, in value, every year at the rate of 6
per cent at the beginning of that
4
year. If its value at the end of 2 years is `2,25,000, what was its worth at the beginning of these
2 years?
6) The value of a machine, purchased two years ago, depreciates at the annual rate of 10%. If its
present value is `97,200, find:
i) its value after 3 years.
ii) its value when it was purchased.
7) Rohit and Rajan are two brothers with ages 18 years and 16 years respectively. In what ratio
must they invest money at 5% compounded yearly so that each may get same sum at the age of
24 years?
8) Calculate the sum of money on which the compound interest (payable annually) for 2 years be
four times the simple interest on `4,715 for 5 years, both at the rate of 5 per cent per annum.
9) A sum of money was invested for 4 years, interest being compounded annually. The rates for
successive years were 10%, 12%, 15% and 18% years respectively. If the compound interest for
the third year amounted to `5,544, find the sum invested.

Volume 1 of 2

Universal Tutorials X ICSE Maths

11

12

PREVIOUS BOARD QUESTION:


1) On what sum of money will the difference between the compound interest and simple interest for
2 years be equal to `25 if the rate of interest charged for both is 5% p.a.?
[2012]
2) In what period of time will `12,000 yield `3,972 as compound interest at 10% per annum, if
compounded on an yearly basis?
[2011]
3) Rohit borrows `86,000 from Arun for two years at 5% per annum simple interest. He immediately
lends out this money to Akshay at 5% compound interest compounded annually for the same
period. Calculate Rohits profit in the transaction at the end of two years.
[2010]
4) What sum of money will amount to `9,261 in 3 years at 5% per annum compound interest?
[2009]
5) The simple interest on a sum of money for 2 years at 4% per annum is `340. Find (i) the sum of
money and (ii) the compound interest on this sum for one year payable half yearly at the same
rate.
[2008]
6) A person invests `10,000 for two years at a certain rate of interest compounded annually. At the
end of one year this sum amounts to `12,000. Calculate:
[2006]
i) The rate of interest per annum
ii) The amount at the end of the second year.
7) If the interest is compounded half yearly, calculate the amount when the Principal is `7,400, the
rate of interest is 5% per annum and the duration is one year.
[2005]
8) The compound interest on a certain sum of money at 5% per annum for two years is `246.
Calculate the simple interest on the same sum for three years at 6% per annum.
[2004]

MISCELLANEOUS EXERCISE:
1) Find the sum on which the compound interest for 3 years t 10% per annum amounts to `1,665.
2) On a certain sum, the compound interest in 3 years amounts to `4,453.20. If the rates of interest
for successive years are 5%, 8% and 10% respectively, find the sum.
3) At what rate per cent will `18,000 yield `5,958 as compound interest in 3 years?
4) A sum of `44,200 is divided between John and Smith, 12 years and 14 years old respectively, in
such a way that if their portions be invested at 10 percent per annum compound interest, they
will receive equal amounts on reaching 16 years of age.
i) What is the share of each out of `44,200?
ii) What will each receive, when 16 years old?
5) A man borrowed `16,000 for 3 years under the following terms:
16% simple interest for the first 2 years.
16% C.I. for the remaining one year on the amount due after 2 years, the interest being
compounded semi-annually. Find the total amount to be paid at the end of the three years.
6) On what sum of money will the C.I. for two and a half years at 10 per cent amount to `676.25?
7) Find the difference between compound interest and simple interest on `12,000 and in 1 years
at 10% compounded half-yearly.
8) On what sum of money will the difference between compound interest and simple interest be
`840 in 2 years at 20% per annum compounded annually?
9) A sum of money, invested at compound interest, amounts to `16,500 in 1 year and to `19,965 in
3 years. Find the rate per cent and the original sum of money.
10) The difference between C.I. and S.I on `7,500 for two years is `12 at the same rate of interest
per annum. Find the rate of interest.
11) The difference between compound interest for a year payable half-yearly and simple interest on
a certain sum of money lent out at 10% for a year is `15. Find the sum of money lent out
12) A man wishes to accumulate `50,440 capital at the end of 3 years from now. If he can invest his
savings at 5 per cent per annum compound interest, what equal sum must be put aside each
year, beginning at the end of the first year, to obtain the required amount.
12

Universal Tutorials X ICSE Maths

Volume 1 of 2

Chapter 02: Compound Interest (Using Formula)

13

13) Simple interest on a sum of money for 2 years at 4% is `450. Find compound interest on the
same sum and at the same rate for 1 year, if the interest is reckoned half-yearly.
14) Find the compound interest to the nearest rupee on `10,800 for 2 years at 10% per annum.
15) A man borrowed a sum of money and agrees to pay it off by paying `9,450 at the end of the first
year and `13,230 at the end of the second year. If the rate of compound interest is 5% per
annum, find the sum borrowed.
16) A sum of money is invested at 10% per annum compounded halfyearly. If the difference of
amounts at the end of 6 months and 12 months is `189, find:
i) the sum of money invested.
ii) the amount at the end of 1 years.
17) A sum of `5,000 invested at 8% p.a. compounded semi annually, amounts to `5,624.32.
Calculate the time period of the investment.

ANSWER TO UNSOLVED EXERCISE:


CW Exercise 2:1
1) `13,891.50 and `1,891.50
2) `1,484.40
3) `4,500
4) `8,000
5) 5%
6) i) 12% (ii) 7,024.64 7) 3 years
8) A = `15,730 and B = `13,000
9) `7,774.63
10) `26.25
11) `3,000
12) `679.18
13) `24,000
HW Exercise 2:1
1) `17,820
2) `7,500
3) 20%
4) 3 years
5) Rohit = `16,000 and Rajesh = `18,522
6) `6,000
7) 1 years
8) `60
CW Exercise 2:2
1) i) `34,073.60 (ii) `50,000
2) `45,000
3) 3 years
4) 138.24 cm
5) 1,60,000
6) 30 years
7) Profit = `7,900
8) 16%
9) `10,000 Anuj = `16,00 and Yash = `1,664
HW Exercise 2:2
1) i) 10% (ii) `19,683 : `30,000
2) 3,00,000
3) 9 years
5) `2,56,000
6) i) `70,858.80 (ii) `1,20,000
4) 10.25%
7) 441:400
8) `46,000
9) `30,000
Previous Year Board Question:
1) P = 10000
2) n = 3
3) Rohits Profit = `215
4) `8000
5) Sum 4250, CI 171.706) i) 20% ii) 14400
7) 7774.63
8) 432
Miscellaneous:
1) `5,000
2) `18,000
3) 10%
4) i) John = `20,000; smith = `24,200 ii) `29,282 5) `24,634.37
6) `2,500
7) `60
8) `10,000
9) 10% and `15,000
11) `6,000
12) `16,000
13) `227.25
10) 4%
14) `2,921
15) `21,000
16) `3,600; `4,167.45
17) 1 years

Volume 1 of 2

Universal Tutorials X ICSE Maths

13

14

Chapter 03: Sales Tax and VAT


Introduction:
Sales Tax (TradeTax):
It is the tax levied by a state government on the sale or purchase of items within the state.
In order to increase revenue, different state governments levy SalesTax on the sale of goods
within their respective states. For the interstate movement of goods, the Union government
also levies SalesTax, known as Central Sales Tax (C.S.T.).
The rate of sales tax depend upon the nature of goods purchased and are different for different
goods (items). Even different states have different rates of salestax on the same items
(goods).
Some items of necessity and/or of daily use for common persons are completely exempted
from salestax.

Some important terms:


Cost Price (C.P.):
z

It is the price at which a trader buys his goods. The cost price is also termed as basic price.

Selling Price (S.P.):


z

It is the price at which a trader sells his goods (without including any tax).

Profit or Loss:
z

If S.P. of an article is more than its C.P., Profit = S.P. C.P.


Profit
100%
C.P.
If C.P. of an article is more than its S.P., Loss = C.P. S.P.

And, Profit % =
z

And, Loss % =

Loss
100%
C.P.

List Price:
z
z
z

The Price marked on an article is called its marked price (M.P) or list price.
List Price is also known as, printed price or quoted price.
In fact, after buying the goods a trader (shopkeeper) marks them at a certain percent above
the cost price.

Sale Price:
z
z

The price at which an article is offered to the customer is called its sale price.
Sale Price = List Price Discount

Computation of Sales Tax:


The rates of Sales Tax depend upon the nature of goods purchased and are different for
different goods (items). Different states have different rates of Sales Tax even on the same
14

Universal Tutorials X ICSE Maths

Volume 1 of 2

Chapter 03: Sales Tax and VAT

15

items (goods). Some items of necessity and/or of daily use for common persons are
completely/ partially exempted from Sales Tax.
The sales tax is always calculated on the sale price of the article.
Calculation of sales tax involves the use of the concept of percentage.
Rate of sales tax sale price
100
Sales tax
100%
Rate of sales tax =
Sale price

Sales tax =

The amount of money paid by a customer for an article =


The sale price of the article + sales tax on it

SOLVED EXAMPLES 3.1:


1) Abhilasha buys following things from a departmental store :
Cosmetics worth `345; Medicines worth `228; Stationary worth `170 and Bakery products
worth `93. Sales tax is charged at the rate of 10% on cosmetics. 7% on medicines, x% on
stationery and the bakery products are exempted from sales tax. She paid ` 894.96 for the
total bill. Find the value of x.
Sol: Total amount paid by Abhilasha = ` 894.96
(345 + 10% of 345) + (228 + 7% of 228) + (170 + x% of 170) + 93 = 894.96
10
7
x

170 + 93 = 894.96
345 + 228 +
228 + 170+
345 +
100
100
100

345 + 34.50 + 228 + 15.96 + 170 + 1.7x + 93 = 894.96


886.46 + 1.7x = 894.96
1.7x = 8.5 x = 5%
2) Mr. Vikas purchased an article for `702 including Sales Tax. If the rate of Sales Tax is 8%,
find the sale price of the article.
Sol: Let the sale price of the article be `x
100
x = `702
= `650
x + 8% of x = `702
108
Sale price of the article = `650
3) The price of an article inclusive of Sales Tax of 12% is `2,016. Find its marked price. If the
Sales Tax is reduced to 7%, how much less does the customer pay for the article?
Sol: Let marked price be ` x x + 12% of x = 2016
112 x
Or, directly, 112% of x = 2016
= 2016 and x = 1800
100
Marked price of the article = `1,800
Since, new Sales Tax = 7%
107
`1, 800 = `1, 926
Now, the customer will pay = `1, 800 + 7% of `1, 800 =
100
Customer will pay for the article = `(2,016 1,926) less = `90 less

UNSOLVED EXERCISE 3.1:


CW Exercise:
1) Ramesh paid `345.60 as sales-tax on a purchase of `3,840. Find the rate of sales tax.
2) Sarita purchases biscuits costing `158 on which the rate of Sales Tax is 6%. She also purchases
some cosmetic goods costing `354 on which the rate of Sales Tax is 9%. Find the total amount
to be paid by Sarita.
Volume 1 of 2

Universal Tutorials X ICSE Maths

15

16

3) Peter goes to a departmental store and purchases the following articles:


i) bakery products costing `160 (sales-tax charged is 5%)
ii) cosmetics costing `547 (sales-tax charged is 12%)
iii) clothes costing `2,350 (sales-tax = 8%)
Calculate the total amount to be paid by Peter to the store.
4) Hamid purchased some articles costing `5,460. The shopkeeper charged Sales Tax at 8%. As
Hamid wanted to take the articles purchased outside the State, the shopkeeper charged 3% as
Central Sales Tax on the actual price of the articles. Find the total amount Hamid had to pay for
the articles.
5) Tanya buys a leather coat costing `1,650. The rate of sales-tax being 10%. She asks the
shopkeeper to reduce the price of the coat to such an extent that she does not have to pay
anything more than 1650 including sales tax. Calculate the reduction needed in the cost price of
the coat.
6) The price of a T.V. set inclusive of sales-tax of 9% is `13,407. Find its marked price. If the salestax is increased to 13%, how much more than the customer pay for the T.V.?
7) The price of an article is `8,250 which includes Sales Tax at 10%. Find how much more or less
does a customer pay for the article, if the Sales Tax on the article:
i) increases to 15%
ii) decreases to 6%
iii) increases by 2%
iv) decreases by 3%
HW Exercise:
1) Rajat purchases a wrist-watch costing `540. the rate of Sales Tax is 8%. Find the total amount
paid by Rajat for the watch.
2) Manoj purchases a bicycle for `1,337.50 including sales tax. If the rate of sales tax is 7%, what
is the sale price of the bicycle?
3) Mr.Lal buys a transistor, costing `625, for `650 including sales tax. Calculate the rate of salestax charged.
4) A colour T.V. is marked for sale at `17,600 which includes Sales Tax at 10%. Calculate the
Sales Tax in rupees.
5) The price of a washing machine, inclusive of Sales Tax, is `13,530/-. If the Sales Tax is 10%,
find its basic (cost) price.
6) A bicycle is available for `1664 including Sales Tax. If the list price of the bicycle is `1,600, find:
i) the rate of Sales Tax.
ii) the price, a customer will pay for the bicycle if the Sales Tax is increased by 6%.

Discount:
z

z
z

In order to saleout his old stock or for some other reason/ reasons, the shopkeeper gives
certain percentage of the list price as discount. This discount is always calculated on list
price/ marked price.
When no discount is given, marked Price of the article becomes the sale price and sales tax
is calculated on it.
When discount is given first calculate the sale price as under
Sale Price = Marked Price Discount,
then calculate the sales tax on the sale price so obtained

SOLVED EXAMPLES 3.2:


1) A trader from Meerut buys an article for `3,600 (inclusive of all taxes) from Kanpur. He
spends `1,200 on traveling, transportation of the article etc. If he desires a profit of 15
percent, how much will a customer pay for the article? The rate of Sales Tax paid by the
customer is 8%.
Sol: For the trader
Price paid for the article = `3,600
Overheads = `1,200
16

Universal Tutorials X ICSE Maths

Volume 1 of 2

Chapter 03: Sales Tax and VAT

17

Cost price of the article = `3,600 + 1,200 = 4,800


Profit desired = 15%
115
100 + 15
`4,800 = `5,520
Sale price =
% of `4,800 =
100
100

Money paid by the customer


= Sale-price of the article + Sales Tax on it
8
= `5,520 + 8% of `5,520 = `5,520 +
`5,520 = `5,961.60
100
2) The catalogue price of a computer set is `45,000. The shopkeeper gives a discount of 7%
on the listed price. He gives a further off-season discount of 4% on the balance. However,
Sales Tax at 8% is charged on the remaining amount Find:
i) the amount of Sales Tax a customer has to pay,
ii) the final price he has to pay for the computer set.
Sol: Since, the list price = `45,000
Discount = 7% of `45,000 = `3,150
S.P = List price Discount = `45,000 `3,150 = 41,850
Off-season discount = 4% of `41,850 = `1,674
Net S.P. = `41,850 `1,674 = 40,176
i) The amount of Sales Tax a customer has to pay = 8% of `40,146
= `3,214.08
ii) The final price, the customer has to pay for the computer
= Net S.P. + Sales Tax = `40,176 + `3,214.08 = `43,390.08

UNSOLVED EXERCISE 3.2:


CW Exercise:
1) A trader buys an unfinished article for `1,800 and spends `600 on its finishing, packing,
transportation, etc. he marks the article at such a price will give him 20% profit. How much will a
customer pay for his article including 12% sales tax?
2) A shopkeeper buys an article for `800 and spends `100 on its transportation, etc. He marks the
article at a certain price and then sells it for `1,287 including 10% sales tax. Find this profit as
percent.
3) The catalogue price of a colour T.V. is `24,000. The shopkeeper gives a discount of 8% on the
list price. He gives a further off-season discount of 5% on the balance. But Sales Tax at 10% is
charged on the remaining amount find:
a) The sales Tax a customer has to pay
b) The final price he has to pay for the T.V.
4) A shopkeeper gives three successive discounts of 20%, 10% and 10% on his goods. If the list
price of an article in his shop is `1,600 and the sales-tax on it is 13% of its sale-price, find how
much a customer will have to pay for it.
5) A shopkeeper marks his goods 40% above the cost price and then allows two successive
discounts of 10% each. Find how much a customer will pay for an article which costs the
shopkeeper `700 and a Sales Tax of 8% is levied on the sale price of the article. (Give your
answer correct to the nearest rupee).
6) A toy is purchased for `591.36 which includes 12% rebate on the printed price and 12% Sales
Tax on the sale price of the toy. Find the printed price of the toy.
7) A trader buys an article for `1,700 at a discount of 15% on its printed price. He raises the printed
price of the article by 20% and then sells it for `2,688 including sales tax on the new marked
price. Find:
i) the rate of sales tax ii) the traders profit as per cent.
Volume 1 of 2

Universal Tutorials X ICSE Maths

17

18

H.W. Exercise:
1) A shopkeeper announces a discount of 15% on his goods. If the marked price of an article, in his
shop is `1,850; how much a customer has to pay for it, if the rate of sales-tax is 9%?
2) The list price of an article is `2,400 and is available at a discount of 15%. If the shopkeeper
further allows a discount of 5% on cash purchase; find how much a customer has to pay
including 10% Sales Tax on the sale price.
3) The catalogue price of an article is `20,000. The dealer allows two successive discounts 15%
and 10%. He further allows an off-season discount of 10% on the balance. But Sales Tax at the
rate of 10% is charged on the remaining amount. Find:
i) the Sales Tax amount a customer has to pay.
ii) the final total price that customer has to pay for the article.
4) A shopkeeper buys an article at a rebate of 20% on its marked price and then spends `300 on
its transportation, etc. if he sells the article for `4,160 (including sales tax at the rate of 4% of the
marked price), find the shopkeepers profit as per cent.
5) A shopkeeper buys an article for `2,400 from a wholesaler at 20% rebate on its list price. He
marks up the list price of the article bought by 10% and then sells it for `3,498 including sales tax
on the marked-up price Find:
i) the rate of sales tax
ii) the shopkeepers profit as per cent.

VAT (Value Added Tax):


VAT: (Value Added Tax) is a new method of realizing tax on the sale/purchase of goods. In the
earlier form of Sales Tax, the tax used to be realized at single point only. The manufacturer or
wholesaler or retailer was liable to pay Sales Tax to the government. But in the VAT system, the
tax is realized by the government at every point right from the manufacturer to the retailer.
z Unlike sales tax, VAT is also collected by the state governments.
z It is not in addition to the existing Sales Tax, but is the replacement of Sales Tax. Presently,
a majority of state governments have accepted the VAT system.
z It is a tax on the value added at each transfer of goods, from the original manufacturer to
the ultimate customer.
z VAT is calculated on the sale value by applying the rate of tax applicable.
z VAT is the difference of tax recovered on the sale value and paid on the purchase
value deposited with the government.
z Example:
Assuming that the rate of tax is 10% and a trader purchase an article for `800
The tax he pays = 10% of `800 = `80
Now, if he sells the same article for `1,150
The tax he recovers (gets) = 10% of `1,150 = `115
VAT = Tax recovered on the sale Tax he paid on the purchase
= `115 80 = `35

Sale Price:
z

The price on which the central tax is calculated is known as sale price. Sometimes sale
price is the difference between list price and discount.
Sale price = List price Discount.

18

Sales tax = Sale price Rate of sales tax

Universal Tutorials X ICSE Maths

Volume 1 of 2

Chapter 03: Sales Tax and VAT

19

Selling price:
z

The price at which the article is actually sold to the customer after adding taxes into sale
price is known as selling price.
When discount is given, Selling price = Sale price + CST
When discount is not given, Selling price = List price + CST

SOLVED EXAMPLES 3.3:


1) The difference of tax recovered on the sale value and paid on the purchase value is
deposited with the government as VAT. To make the concept more clear, please see the
following table carefully (assuming the rate of tax = 10%)
Sol:
Cost of material = `600; Tax paid = 10% of `600 = `60
S.P. of prepared goods = `1,000,
tax charged = 10% of `1,000 = `100
For
manufacturer
VAT = `100 60 = `40
Purchase
Selling
Tax
Tax paid
VAT
price
Price
charged
For 1st trader
`1,000
`100
`1,200
`120
Rs(120 100)=`20
nd
For 2 trader
`1,200
`120
`1,450
`145
Rs(145 120)=`25
For0 retailer
`1,450
`145
`1,680
`168
Rs(168 145)=`23
Important: According to the old procedure, the tax is charged by the retailer only and tax charged
by the retailer in above case = 10% of `1,680 = `168.
Using this new VAT system, the sum of the taxes charged and paid to the government
= `(60 + 40 + 20 + 25 + 23) = `168
Total tax collected by the Govt. = `168
[Which is just equal to the total existing Sales Tax]
Thus, total VAT charged and paid at different stages is the tax on the price paid by the customer,
but it is collected and paid in stages.
2) A manufacturer sells a TV set to a wholesaler for `20,000. The wholesaler sells it to a
retailer and earns `2,000. The retailer further sells it to a customer at a profit of `1,500. If
the VAT is 8% find.
i) the amount of tax (under VAT) received by the State Government on the sale of the TV
set.
ii) the amount that the customer paid for the TV set.
Sol: i) Tax collected by manufacturer = 8% of `20,000
8
20,000 = `1,600
= Rs
100
Amount of VAT to be paid by the wholesaler = Tax on the value added by the wholesaler
= 8% of `2,000
8
2,000 = Rs160
=`
100
VAT to be paid by retailer = Tax on the value added by the retailer
8
1,500 = `120
= 8% of `1,500 = `
100
VAT received by the State Government
= `1,600 + `160 + `120 = `1,880
Volume 1 of 2

Universal Tutorials X ICSE Maths

19

20

ii) The value of the TV set (excluding VAT) paid by the customer
= The price charged by manufacturer + Profit of wholesaler + profit of retailer
= `20,000 + `2,000 + `1,500 = `23,500
8
23,500 = `1,880
Tax paid by the customer = 8% of `23,500 = Rs
100
Amount paid by the customer (including VAT)
For the TV set = `23,500 + `1,880 = `25,380

UNSOLVED EXERCISE 3.3:


CW Exercise:
1) A shopkeeper purchases an article for `6,200 and sells it to a customer for `8,500. If the VAT
rate is 8%; find the VAT paid by the shopkeeper.
2) A manufacturer buys raw material for `60,000 and pays 4% tax. He sells the ready stock for
`92,000 and charges 12.5% tax. Find the VAT paid by the manufacturer.
3) The cost of an article is `6,000 to a distributor. He sells it to a trader for `7,500 and the trader
sells it to a customer for `8,000. If the VAT rate is 12.5%; find the VAT paid by the:
i) distributor
ii) trader.
4) The printed price of an article is `2,500. A wholesaler sells it to a retailer at 20% discount and
charges sales-tax at the rate of 10%.Now the retailer, in turn sells the article to a customer and
charges the sales-tax at the same rate Find:
i) the amount that retailer pays to the wholesaler.
ii) the VAT paid by the retailer.
5) A retailer buys an article for `800 and pays the sales-tax at the rate of 8%. The retailer sells the
same article for `1,000 and charges sales-tax at the same rate Find:
i) the price paid by a customer to buy this article.
ii) the amount of VAT paid by the retailer.
6) A shopkeeper buys 15 identical articles for `840 and pays sales-tax at the rate of 8%. He sells 6
of these articles at `65 each and charges sales-tax at the same rate. Calculate the VAT paid by
the shopkeeper against the sale of these six articles.
7) A manufacturer marks an article at `4,800. He sells this article to a wholesaler at a discount of
25% on the marked price and the wholesaler sells it to a retailer at a discount of 15% on its
marked price. If the retailer sells the article without any discount and at each stage the sales-tax
is 8%, calculate the amount of VAT paid by:
i) the wholesaler
ii) the retailer
8) A shopkeeper buys an article at a discount of 40% and pays sales-tax at the rate of 12%. The
shopkeeper, in turn, sells the article to a customer at 10% discount and charges sales-tax at the
same rate. If the printed price of the article is `3,000; find:
i) the price paid by the shopkeeper
ii) the price paid by the customer
iii) the VAT (Value Added Tax) paid by the shopkeeper.
9) A shopkeeper sells an article at its list price (`3,000) and charges sales-tax at the rate of 12%. If
the VAT paid by the shopkeeper is `72, at what price did the shopkeeper buy the article inclusive
of sales-tax?
HW Exercise:
1) A purchases an article for `3,600 and sells it to B for `4,800. B, in turn, sells the article to C for
`5,500. If the VAT rate is 10%, find the VAT levied on A and B.
2) Rohit has a furniture shop in Delhi. He buys a dining table for `12,000 and sell it to a customer
for `15,000. Find the VAT paid by Rohit, if the VAT rate is 10%.
20

Universal Tutorials X ICSE Maths

Volume 1 of 2

Chapter 03: Sales Tax and VAT

21

3) The marked price of an article is `900 and the rate of sales-tax on it is 6%. If on selling the article
at its marked price, a retailer has to pay VAT = `4.80; find the money paid by him (including
sales tax) for purchasing this article.
4) A shopkeeper buys an article at a discount of 30% and pays sales-tax at the rate of 8%. The
shopkeeper, in turn, sells the article to a customer at the printed price and charges sales tax at
the same rate. If the printed price of the article is `2,500; find:
i) the price paid by the shopkeeper.
ii) the price paid by the customer.
iii) the VAT (Value Added Tax) paid of the shopkeeper.
5) A manufacturer marks an article for `6,000. He sells it to a wholesaler at 25% discount. The
wholesaler sells this article to a retailer at 15% discount on the marked price of the article. If
retailer sells the article to a customer without any discount and the rate of sales-tax is 8% at
each stage; find the amount of VAT paid by the
i) wholesaler
ii) retailer

MISCELLANEOUS EXERCISE:
1) Anisha purchased cosmetics for `172.50 including sales-tax. If the rate of sales tax is 15%, find
the list price of the cosmetics.
2) Lata goes to a shop to buy a leather coat, costing `654. The rate of sales-tax is 9%. She asks
the shopkeeper to reduce the price of the coat to such an extent that she has to pay `654,
inclusive of sales-tax. Find the reduction in the price of the coat.
3) The price of an article is `1,400 including a sales tax of 12%. Find its marked price.
i) increased to 15%?
ii) decreased to 10%? iii) increased by 7%? iv) decreased by 4%?
4) A bicycle is available for `1,664 including sales-tax. If the list price of the bicycle is `1,600, find:
i) the rate of sales tax.
ii) the price, a customer will pay for the bicycle if the sales-tax is increased by 6%.
5) An article is available for `1,430 inclusive of sales-tax at the rate of 10%. Find its list price. What
will be its new selling price if the rate of sales-tax changes to 12%?
6) Find the basic (cost) price on an article which is sold for 2,034 including 13% sales-tax.
7) The marked price of a taxable article was `4,000, which is sold to the first dealer at the same
rate. He sold to the second dealer for `4,240. The second dealer sold it to a customer for `4,320.
Find MVAT at every stage of trading at the rate 4%.
8) A mobile set is sold to wholesale dealer for `3,000. He sold the same set to a subdealer for
`3,200 and a customer purchased it from the second dealer for `3300.Find MVAT at every
stage of trading at the rate 12.5%.
9) A motor cycle production company sells one piece of motor cycle to the first sales dealer at the
cost of `30,800. He sells it to the subdealer for `36,000. Find the VAT to be paid at every stage
of trading at the rate 4%.
10) A company sells a car to Mahendrasingh for `1,800,000. Mahendrasingh sells the same car to
Hakimbhai for `2,20,000 and Hakimbhai sells it to Miss Mary for `2,50,000. Find MVAT to be
paid at every stage of trading at the rate 12.5%.
11) Hirachand has purchased goods worth `2350, and he has to pay `50 as other charges. He sells
the same product to Laxmibai for `2600.She then sells the purchased product to Manganlal for
`2800. Find the VAT at the rate 4% Laxmibai has to pay.
12) Neha bought a purse for `828 including 15% Sales Tax and a face-powder for `77 including
10% Sales Tax. Find the total amount of Sales Tax charged on both the items.
13) A shopkeeper buys an article for `7,500 and increases its price. He sells this article for `9,156
including 9% Sales Tax on the increased price. Calculate by how much per cent does the
shopkeeper increase the price of the article.

Volume 1 of 2

Universal Tutorials X ICSE Maths

21

22

14) An imported car marked at `6,43,800 is available at a discount of 8%. If Sales Tax is charged at
10%, find the amount required to buy the car.
15) Johny bought a pant for `672, including 12% Sales Tax and a shirt for `440, including 10%
Sales Tax. Find the total basic price of the pant and the shirt together.
16) Madan purchases a compact computer system for `47,700 which includes 10% rebate on the
marked price and then 6% Sales Tax on the remaining price. Find the marked price of the
computer correct to two decimal places.
17) A shopkeeper buys an article at 70% of its printed price. He spends `40 on transportation of the
article. After charging Sales Tax at the rate of 7% on the printed price, he sells the article for
`6,848. Find his profit as percent.
18) Mrs. Sheela bought a washing machine marked at `9,375. The rate of Sales Tax is 6%. She
asks the shopkeeper to reduce the price of the washing machine to such an extent that she has
to pay `9,275 inclusive of Sales Tax. Find the reduction needed in the price of the washing
machine.
19) An article is available for `13,789.44 which includes two successive discounts of 20% and 5%
respectively on the list price and then 8% Sales Tax on the remaining price. Find the list price of
the article.
20) The catalogue price of a colour T.V. is `18,000. The shopkeeper gives a discount of 8% on the
marked price. He gives a further off-season discount of 5% on the balance. But Sales Tax at
10% is charged on the remaining amount Find:
i) the Sales Tax amount a customer has to pay.
ii) the final price he has to pay for the colour T.V.
21) A retailer sells an article for `5,350 including 7% Sales Tax on the listed price. If he had bought it
at a discount and has made a profit of 25% on the whole, find the rate of discount he gets.
22) A wholesaler sells an article for `2,700 at a discount of 10% on the printed price to a retailer. The
retailer, in turn, raises the printed price of the article by 15% and sells it for `3,657 which
includes Sales Tax on the new marked price Find :
i) the rate of Sales Tax.
ii) the profit, as per cent, made by the retailer.
23) An article is marked at `500. the wholesale sells it to a retailer at 20% discount and charges
sales-tax on the remaining price at 12.5%. The retailer, in turn, sells the article to a customer at
its marked price and charges sales-tax at the same rate. Calculate:
i) the price paid by the customer
ii) the amount of VAT paid by the retailer.
24) The printed price of an article is `9,600. A shopkeeper buys it at a discount of 20% and sells it at
the printed price. Find the amount of VAT paid by the shopkeeper, if the rate of Sales Tax is
10%. Also, find the profit made by the shopkeeper, if he spends `120 on the transportation of the
article.

PREVIOUS BOARD QUESTION:


1) The printed price of an article is `60,000. The wholesaler allows a discount of 20% to the
shopkeeper. The shopkeeper sells the article to the customer at the printed price. Sales tax
(under VAT) is charged at the rate of 6% at every stage. Find:
[2012]
i) the cost to the shopkeeper inclusive of tax.
ii) VAT paid by the shopkeeper to the Government.
iii) the cost to the customer inclusive of tax.
2) A manufacture sells a washing machine to a wholesaler for `15,000. The wholesaler sells it to a
trader at a profit of `1,200 and the trader in turn sells it to a consumer at a profit of `1,800. If the
rate of VAT is 8% find:
[2011]
i) The amount of VAT received by the State Government on the sale of this machine from the
manufacturer and the wholesaler.
ii) The amount that the consumer pays for the machine.
22

Universal Tutorials X ICSE Maths

Volume 1 of 2

Chapter 03: Sales Tax and VAT

23

3) A manufacture marks an article for `5000. He sells it to a wholesaler at a discount of 25% on the
marked price and he wholesaler sells it to a retailer at a discount of 15% on the marked price.
The retailer sells it to a consumer at the marked price and at each stage the VAT is 8%.
Calculate the amount of VAT received by the Government from:
[2010]
i) the wholesaler
ii) the retailer
4) Tarun bought an article for `8000 and spent `1000 for transportation. He marked the article at
`11,700 and sold it to a customer. If the customer had to pay 10% sales tax, find.
[2010]
i) The customers price
ii) Taruns profit percent.
5) A shopkeeper bought a TV at a discount of 30% of the listed price of `24,000. The shopkeeper
offers a discount of 10% of the listed price to his customer. If the VAT (Value Added Tax) is 10%.
Find: i) the amount paid by the customer, ii) the VAT to be paid by the shopkeeper.
[2009]
6) A shopkeeper buys a camera at a discount of 20% from the wholesaler, the printed price of the
camera being `1600 and the rate of sales tax is 6%. The shopkeeper sells it to the buyer at the
printed price and charges tax at the same rate. Find:
i) The price at which the camera can be bought.
ii) The VAT (Value Added Tax) paid by the shopkeeper.
[2008]
7) Dinesh bought an article for `374, which included a discount of 15% on the marked price and a
sales tax of 10% on the reduced price. Find the marked price of the article.
[2007]
8) Kiran purchases an article for `5,400 which includes 10% rebate on the marked price and 20%
sales tax on the remaining price. Find the marked price of the article.
[2006]
9) The catalogue price of a computer set is `45,000. The shopkeeper gives a discount of 7% on the
listed price. He gives a further offseason discount of 4% on the balance. However, sales tax at
8% is charged on the remaining amount. Find:
i) The amount of sales tax a customer has to pay.
ii) The final price he has to pay for the computer set.
[2005]
10) Ms.Chawla goes to a shop to buy a leather coat which costs `735. The rate of sales tax is 5%.
She tells the shopkeeper to reduce the price of the coat, such that she has to pay only `735
inclusive of sales tax. Find the reduction needed in the price of the coat.
[2004]

ANSWER TO UNSOLVED EXERCISE:


CW Exercise 3.1:
1) 9%
2) `553.34
3) `3318.64
5) `150
6) 12,300 `492
7) i) `375 more (ii) `300 less (iii) ` 150 more (iv) ` 225 less

4) 6060.60

HW Exercise 3.1:
1) ` 583.20
5) `12,300

2) `1250
6) i) 4% (ii) 1760

3) 4%

CW Exercise 3.2:
1) `3,225.60
4) `1,171.58

2) 30%
5) `857

3) a) `2097.60 (b) `23073.60


6) `600
7) i) 12% (ii) 41 3/17%

HW Exercise 3.2:
1) `1,714.03
4) 14 2/7%

2) `2,131.80
5) 6% and 37.5%

3) i) `1377 (ii) `15,147

CW Exercise 3.3:
1) `184

2) `9100

3) i)187.50(ii)`62.50

Volume 1 of 2

Universal Tutorials X ICSE Maths

4) `1600

4) i) `2200 (ii) `50


23

24

5) i) `1080 (ii) `16


6) `4.32
8) i) `2016 (ii) `3024 (iii) `108

7) i) `38.40 (ii) `57.60


9) `2688

HW Exercise 3.3:
2) `300
1) A = `120; B = `70
4) i) `1,890 (ii) `2,700 (iii) `60

3) `869.20
5) i) `48 (ii) `72

Previous Board Question:


1) i) `50,880 (ii) `720 (iii) `63,600
4) i) `12,870 (ii) 30%
7) `400
8) ` 5000

2) i) `96 (ii) `19,440


3) i) `300 (ii) `40
5) `23,760; `480
6) i) 1696 ii) 19.2
9) i) 3214.08 ii) 43390.08 10) 35

Miscellaneous:
1) `150
2) `54
3) `1,250 (i) `37.50 more (ii) `25 less (iii) `87.50 more (iv) `50 less
4) i) 4% (ii) `1,760
5) `1,300; `1,456
6) `1,800
7) 160, 9.60, 3.2
8) 3.75, 25, 12.5
9) 1232, 128, 80
10) `22,500, `5,000, `3,750
11) `8
12) `115
13) 12%
14) `6,51,525.60
15) `1000
16) `50,000
17) 41.59%
18) `625
19) `16,800
20) i) `1573.20 (ii) `17305.20
21) 20%
22) i) 6% (ii) 27.78%
23) i) `562.50 (ii) `12.50
24) `192, `1800

24

Universal Tutorials X ICSE Maths

Volume 1 of 2

Chapter 04: Banking (Saving and Recurring Account)

25

Chapter 04: Banking (Saving and Recurring Account)


Introduction:
The business of receiving, safeguarding and lending of money is called banking. In general,
people who have some spare money do not keep it with them to avoid the risk of losing it by theft
etc. They deposit this spare money in a bank. In the bank, the money is safe as well as it fetches
interest on it.
On the other hand, some people need money to start a business or to expand their business. So
they borrow money from the bank at a nominal interest on the money borrowed from the bank.
Thus, a bank is an institution which carries on the business of taking deposits and lending money.
The rate of interest charged by the bank from its borrowers is usually higher than what it pays to
depositors.
In addition to moneytaking and moneylending, the banks also perform various other functions
and almost every individual and every section of society deals with banks in one way or the other.

The main functions of a bank are:


z
z
z
z

Receiving money from the depositors and payments for public utility services such as
telephone bills, electricity bills, water bills and house tax etc.
Lending and transferring money from one place to another.
Renting safe deposit lockers for the safe custody of valuables.
Helping travellers and tourists by providing travellers cheque and foreign currency etc.

Different Types of Bank Accounts:


z

Mainly there are four types of accounts


i) Savings Bank Account: It is the most popular savings schemes offered by the banks.
Any person can open an account even with a small amount of one thousand rupees
only. Whenever required account holder can deposit or withdraw money from his
account by cash or by a cheque. A passbook is issued to every depositor by the bank in
which datewise entries regarding the deposits, withdrawals and the interest earned are
recorded by the bank.
ii) Current Account: This account is usually maintained by businessmen, companies,
government organizations, etc who have to do a number of money transactions daily.
iii) Term or Fixed deposits: In this scheme, money is deposited for a fixed period. In such
deposits, money can be withdrawn after the expiry of the fixed period which is specified
at the time of opening the account.
iv) Recurring deposits: In this account, a depositor chooses a specified amount and
deposits that every month for a fixed period. This period may vary from six month to 10
years. The depositor is paid a lumpsum payment after the expiry of fixed period.
Out of the various types of accounts (deposit schemes) provided by the banks, we shall
confine our discussion to the following two popular and most commonly used bank
accounts, namely:
Savings Bank Accounts.
Recurring Deposit Accounts.

In Case of Saving Bank Account:

Volume 1 of 2

Universal Tutorials X ICSE Maths

25

26

Note:

i) The interest for a particular month is computed on the minimum balance between the 10th day
and the last day of the month in a multiple of `10.
ii) Add the balance of all the months and get the value of P.
iii) If account is opened after 10th of a month then no interest on any sum is given for that month, i.e.
minimum balance for that month, which qualifies for interest = 0
iv) If account is closed on any date of the month then also no interest on any sum is given for that
month, i.e. minimum balance for that month, which qualifies for interest = 0
For Savings Bank Account, Interest =

PRT
100

SOLVED EXAMPLES 4.1:


1) Mr.Gulati has a Saving Bank Account with the Central Bank of India. A page from his
passbook in a particular year is given below:
Date
Particulars
Withdrawals (`)
Deposits (`)
Balance (`)
Jan. 1
By Balance B/F
2,000.00
Jan. 7
By Cash
4,000.00
6,000.00
Jan. 21 To Cheque
2,000.00
4,000.00
Feb. 27 By Cheque
7,500.00
11,500.00
March 13 To Cheque
8,500.00
3,000.00
March 24 By Cash
4,000.00
7,000.00
March 31 By Interest
May 24 By Cheque
2,500.00
Sept. 8
To Cash
1,500.00
Sept. 20 To Cash
500.00
Sept. 31 By Interest
Oct. 20 By Cash
3,500.00
Oct. 31 Total Balance
The bank calculates interest at 6% p.a. and compounds it at the end of March and
September every year. Complete the passbook showing his total balance upto 31st
October of the same year.
Sol: Minimum amount to credit of Mr.Gulati between 10th Jan. to 31st Jan. = `4,000
Principal for January = `4,000
Principal for March = `3,000
Similarly, Principal for February = `4,000
Total principal for 1 month from January to March = `11,000
1
, R = 6%
Now, P1 = `11,000, T =
12
P R T
11,000 6 1
=
= `55
Interest x = 1
100
100 12
Principal for April = `7,000 + Int. `55 = 7,055
Principal for May= `7,055
Principal for June = `10,055
Principal for August = `10,055
Principal for July= `10,055
Principal for September = `8,055
Total principal for 1 month from April to Sept. = `52,330
26

Universal Tutorials X ICSE Maths

Volume 1 of 2

Chapter 04: Banking (Saving and Recurring Account)

27

1
, R = 6%
12
P R T
52,330 6 1
Interest = 2
=
= `261.65
100
100 12
Principal for October = `7,837.25
As passbook is filled up to March 24 showing balance `7,000.
Further we can update the passbook as under:
Date
Particulars
Withdrawals (`)
Deposits (`)
Balance (`)
March 31
Interest
75.00
7,075.00
May 24
By Cheque
2,500.00
9,575.00
Sept. 8
To cash
1,500.00
8,075.00
Sept. 20
To cash
500.00
7,575.00
Sept. 31
By interest
261.65
7,816.65
Oct. 20
By Cash
3,500.00
11,316.65
2) Mr. Jagdish has a Savings Bank Account in Canara Bank. His passbook has following
entries:
Date
Particulars
Withdrawals
Deposits
Balance
(` P)
(` P)
(` P)
Jan. 2
By Balance

2,300.00
Jan.9
By cash

600.00
2,900.00
Feb.7
To cheque
300.00

2,600.00
Feb.18
By cheque

800.00
3,400.00
Mar.3
To cheque
500.00

2,900.00
Mar.21
By cash

800.00
3,700.00
June 9
By cash

300.00
4,000.00
July 4
To cheque
300.00

3,700.00
Aug.11
By cash

500.00
4,200.00
Sep.8
By cash

400.00
4,600.00
Nov.16
To cheque
800.00

3,800.00
Dec. 5
By cash

500.00
4,300.00
Dec.23
To cheque
200.00

4,100.00
The rate of interest is 6% per annum and the interest is paid once a year at the end of
December. Calculate the interest earned by Mr. Jagdish in the given year.
Sol: Min. balance for the month of Jan = `2,900
Min. balance for the month of Feb = `2,600
Min. balance for the month of Apr = `3,700
Min. balance for the month of Mar = `2,900
Min. balance for the month of May = `3,700
Min. balance for the month of June = `4,000
Min. balance for the month of July = `3,700
Min. balance for the month of Aug = `3,700
Min. balance for the month of Oct = `4,600
Min. balance for the month of Sep = `4,600
Min. balance for the month of Nov = `3,800
Min. balance for the month of Dec = `4,100
Total = ` 44,300
PRT
Interest for the period Jan. to Dec =
100 12
44300 6 1
I=
= `221.50
100 12

Now, P2 = `52,330, T =

Volume 1 of 2

Universal Tutorials X ICSE Maths

27

28

UNSOLVED EXERCISE 4.1:


CW Exercise
1) a) Manu opened a Savings Bank Account in the State Bank of India on 3rd Sept. 96 with `2,700.
He withdrew `1,200 from the bank on 8th Sept. 96 and deposited `500 on 17th Sept. 96. If he
did not make any further deposits or withdrawals during this month; find the amount on which
he would receive interest for Sept. 96.
b) Geeta opened a Savings Bank Account in a Bank on 7th Nov. 95 and deposited `750. She
withdrew `200 on 30th Nov. 95. If no other withdrawal or deposit was made by her during this
month. Find the amount on which she would receive interest for the month of Nov. 95.
2) John had a Savings Bank Account in a bank. In the months of April, 97 and May, 97 he had the
following entries in this pass-book.
Date

Particulars

Withdrawals (`)

Deposit (`)

Balance (`)

April 1 By Balance

4,600.00

April 7 By Cash

1,200.00

5,800.00

April 24 To Cheque

800.00

5,000.00

May 16 By Cheque

2,000.00

7,000.00

May 29 To Cash
1,500.00

5,500.00
Find the amounts, on which John will get interest for the months of April, 97 and May, 1997.
3) A man holds a Savings Bank Account in the Union Bank of India. A page of his pass-book show
the following entries:
Date

Particulars

Withdrawals (`)

Deposits (`)

Balance (`)

May 1

Balance B/F

3,200.00

May 6

By Cheque

500.00

3,700.00

May 17 By Cheque

800.00

4,500.00

June 4 By Cheque

1,300.00

5,800.00

June 9 To Cheque
1,650.00

4,150.00
Calculate the rate of interest per annum, if the total interest earned for the month of May and
June is `39.25
4) Mr.Sen has a Saving Bank Account with a Post Office.
i) Calculate the interest earned by Mr.Sen during the year 1993 at 6.5% per annum payable in
December, if the entries during the year, in his passbook are as given below:
Date

Particulars

Withdrawals (`)

Deposits (`)

2.1.93

By Cash

250.00

9.1.93

By Cheque

825.00

13.3.93

To Cash

325.00

24.7.93

By Cash

1,237.00

6.10.93

To Cheque

250.00

22.12.93 By Cheque

958.00
ii) Also calculate the interest, when the principal for every month is taken as the nearest multiple
of `10.
5) Mrs.Swami had a Saving Bank Account with the State Bank in India from 13th Feb., 89 to 31st
July, 89. The following table shows the entries in her passbook for the above said period.
Calculate the interest earned by Mrs.Swami on her S.B. Account upto 31st July, 89 at the rate of
5% per annum.
28

Universal Tutorials X ICSE Maths

Volume 1 of 2

Chapter 04: Banking (Saving and Recurring Account)

29

Date
Particulars Withdrawals (`)
Deposits (`)
Balance (`)
Feb. 13 By Cash

500.00
500.00
March 3 By Cheque

735.00
1,235.00
March 14 By Cheque

1,040.00
2,275.00
May 10 To Cheque
240.00

2,035.00
May 22 To Cash
430.00

1,605.00
June 19 By Cash

780.00
2,385.00
July 26
To Cash
980.00

1,405.00
6) A page from the passbook of a Saving Book Account in a particular year is given below:
Date
Particulars Withdrawals (`)
Deposits (`)
Balance (`)
Jan. 3
By Cash

5,000.00
5,000.00
Feb. 13 To Self
1,000.00

March 24 By Cheque

2,000.00

March 31 By Interest

May 20 By Cash

800.00

July 7
To Cheque
1,400.00

July 18
By Cash

1,600.00

Sept. 15 To Cheque
3,200.00

Sept. 26 By Cheque

2,350.00

If the interest is calculate at 6% p.a. and is compounded at the end of March and September
every year, find the interest earned upto 31st March and then after completing all the entries, find
the amount that the account holder would have received had he closed the account on 20th Oct.
the same year.
7) Mrs. N. Batra has a Savings Bank Account with the Punjab National Bank. She had the following
transactions (from 1st January, 2007 to 31st December, 2008) with the bank:
ii) 13.03.2007; deposited `6,482/
i) 01.01.2007; B/F `8,764/
iv) 09.08.2007; withdrew `1,333/
iii) 22.06.2007; withdrew `4,369/
v) 24.11.2007; deposited Rs2,158/
Calculate the interest accrued up to 31st December 2007; if the rate of interest is 5%
compounded yearly and the principal for every month is taken as the nearest multiple of `10
HW Exercise:
1) A man opened a Savings Bank Account with a bank on 22nd Feb. 1998 and deposited `300. He
further deposited `1,500 on 5th March 1998 and withdrew `500 on 12th April 1998. Assuming that
he neither deposited nor withdrew any money upto the last day of May 1998; write the amounts
on which he would receive interest for:
a) February 1998
b) March 1998
c) April 1998
d) May 1998
2) A page from the pass-book of Asha is given below:
Date Particulars Withdrawals (`)
Deposits (`)
Balance (`)
Jan. 5 Balance B/F

3,750.00
March 7 To Cheque
1,200.00

April 2 By Cheque

2,300.00

April 10 By Cheque

820.00

Oct. 6 To Cheque
950.00

Dec. 8 By Cash

1,700.00

Complete the pass-book and then calculate the interest for the period January to December
5% per annum.
Volume 1 of 2

Universal Tutorials X ICSE Maths

29

30

3) Mr.Nayyar has a Saving Bank Account with Punjab National Bank. During the year 1995, his
passbook shows the following entries:
Date
Particulars Withdrawals (`)
Deposits (`)
Balance (`)
Jan. 1
By Balance
5,400.00
Jan. 8
By Cheque
2,500.00
7,900.00
March 3 By Cash
800.00
8,700.00
March 16 To Cheque
3,200.00
5,500.00
May 13 By Cash
2,400.00
7,900.00
May 29 To Cash
1,700.00
6,200.00
Sept. 18 To Cheque
900.00
5,300.00
Oct. 7
By Cash
750.00
6,050.00
Dec. 23 To Cash
2,000.00
4,050.00
Dec. 28 By Cheque
1,950.00
6,000.00
Calculate the interest earned by Mr. Nayyar for the year 1995 at 5% per annum payable in
December.
4) A Saving Bank Account was opened by Mrs.Roy on 3.7.92 and closed on 31.01.93. The entries
in the passbook of Mrs.Roy were as given below:
Date Particulars Withdrawals (`)
Deposits (`)
Balance (`)
3.7.92
By Cash

690.00

15.7.92 By Cash

1,153.00

27.8.92 By Cheque

2,468.00

30.8.92 To Cash
946.00

12.9.92 To Cheque
1,000.00

20.11.92 By Cheque

3,000.00

26.11.92 By Cash

750.00

28.11.92 To Cash
1,570.00

i) By finding the balance on different dates of entries calculate the interest earned by Mrs.Roy,
for the period she had this Saving Bank Account, at the rate of 6% p.a.
ii) Also calculate the interest, when the principal for every month is taken as the nearest multiple
of `10.
5) A page from the passbook of Mr.Rohit is given below. Find the interest for the period January to
December 1998 at 4.5% per annum.
Date
Particulars Withdrawals (`)
Deposits (`)
Balance (`)
Jan. 1
By Balance
7,500.00
March 7 By Cheque
1,875.00
March 10 By Cash
625.00
July 17
To Self
3,250.00
Oct. 5
By Cheque
2,160.00
Dec. 19 To Cheque
1,340.00
6) The entries in a Savings Bank passbook are as given below:
Date
Particulars
Withdrawals (`)
Deposits (`)
Balance (`)
01.01.03
B/F
14,000.00
01.02.03
By Cash
11,500.00
25,500.00
12.02.03
To Cheque
5,00.00
20,500.00
05.04.03
By Cash
3,750.00
24,250.00
15.04.03
To Cheque
4,250.00
20,00.00
09.05.03
By Cash
1,500
21,500.00
04.06.03
By Cash
1,500
23,000.00
Calculate the interest for six months (January to June) at 4% per annum on the minimum
balance on or after the tenth day of each month.
30

Universal Tutorials X ICSE Maths

Volume 1 of 2

Chapter 04: Banking (Saving and Recurring Account)

31

Recurring Deposit Account (R.D. Account):


As suggested by the name of this account, under this scheme, a depositor chooses a specified
amount (usually in multiples of `5) and deposits that amount every month for a fixed period,
chosen by the account holder at the time of opening this account.
This period may vary from 3 months to 10 years. At the expiry of this period (called, the
maturity period) the depositor is paid a lumpsum amount (called, the maturity value).
The maturity value of a R.D. Account includes the amount deposited by the account holder
together with interest compounded quarterly at a fixed rate. This rate is fixed by the Reserve
Bank of India and is revised from time to time.

Computing maturity value of a Recurring Deposit Account:


A sum of `500 deposited in a bank for 8 months will earn the same interest as `4,000 (`500
8) deposited in the bank for 1 month if the rate of interest is the same.
Similarly, `70 deposited in a bank for 25 months will earn the same interest as `70 25 =
`1,750 deposited in the bank for one month with the same rate of interest. And so on.
1 + 2 + 3 + 4 + + n =

n(n + 1)
.
2

Direct Method:
Let P = Installment per month, n = no. of months for which R.D. is opened, R = rate of
1
years, then the interest (I) can be obtained by using the
interest per annum and T =
12
n(n + 1) RT
formula: I = P

2
100
For Example:
1
P = `300, n = 24, R = 10% and T =
years
12
n(n + 1) RT
24(24 + 1)
10 1
I = `300

= 750
I=P

2
100 12
2
100
The amount that Rajesh will get at the time of maturity = `(300 24) + 750 = `7,950
z

SOLVED EXAMPLES 4.2:


1) Mr.Gulati has a recurring deposit account of `300 per month. If the rate of interest is 12%
and the maturity value of this account is `8,100; find the time (in years) of this recurring
deposit account.
Sol: As sum deposited per month = `300 and number of months = n (say)
n(n + 1)
= `150(n2 + n)
The equivalent principal for 1 month = `300
2
1 .5 2
n(n + 1)
150(n 2 + n ) R 1
=
(n + n) 12 =
3
100 12
12
2
3n(n + 1)
n
8,100 =
(600 + 3n + 3)
Maturity value = `300 n +
2
2
16,200 = 603n + 3n2 or n2 + 201n 5,400 = 0
n2 + 225n 24n 5,400 = 0
n(n + 225) 24(n + 225) = 0
(n 24) (n + 225) = 0
n = 24 or 225 (not possible)
n = 24 months
or 2 years

Interest earned = `

Volume 1 of 2

Universal Tutorials X ICSE Maths

31

32

UNSOLVED EXERCISE 4.2:


CW Exercise:
(Solve each of the following questions without using tables)
1) Find the maturity value of a recurring deposit account when a man deposits every month:
i) `100 for 9 months at 8% per year
ii) `225 for 1 years at 9% per annum.
iii) `540 for 3 years at 12% p.a.
2) Mrs. Mathew opened a recurring deposit account in a certain bank and deposited `640 per
month for 4 years. Find the maturity value of this account, if the bank pays interest at the rate
of 12% per year.
3) A man has a recurring deposit account in a bank for 3 years. If the rate of interest is 12% per
annum and the man gets `2,892 on maturity, find the value of monthly instalments.
4) Amit deposited `150/- per month in a bank for 8 months under the Recurring Deposits Scheme.
What will be the maturity value of his deposits, if the rate of interest is 8% per annum and
interest is calculated at the end of every month?
5) A recurring deposit account of `1,200 per month has a maturity value of `12,440. If the rate of
interest is 8% and the interest is calculated at the end of every month; find the time (in months)
of this recurring deposit account.
HW Exercise:
(Solve each of the following questions without using tables)
1) Manish opens a recurring deposit account with the Bank of Rajasthan and deposits `600 per
month for 20 months. Calculate the maturity value of this account, if the bank pays interest at the
rate of 10% per annum.
2) Ashish deposits a certain sum of money every month in a recurring deposit account for a period
of 12 months. If the bank pays interest at the rate of 11% p.a. and Ashish gets `12,715 as the
maturity value of this account, what sum of money did he pay every month.
3) Puneet has a recurring deposit account in the Bank of Baroda and deposits `140 per month for 4
years. If he gets `8,092 on maturity, find the rate of interest given by the bank.
4) Mrs.Geeta deposited `350/- per month in a bank for 1 year 3 months under the Recurring
Deposit Scheme. If the maturity value of her deposits is `5,565; find the rate of interest per
annum.

PREVIOUS BOARD QUESTION:


1) Kiran deposited `200 per month for 36 months in a banks recurring deposit account. If the bank
pays interest at the rate of 11% per annum, find the amount she gets on maturity.
[2012]
2) A page from the Savings Bank Account of Mr.Prateek is given below:
Deposits
Balance
Withdrawals
Date
Particulars
(in Rs)
(in `)
(in `)
Jan. 1st 2006
B/F

1,270
th
Jan. 7 2006
By Cheque

2,310
3,580
Mar. 9th 2006
To Self
2,000

1,580
Mar. 26th 2006
By Cash

6,200
7,780
June. 10th 2006
To Cheque
4,500

3,280
July. 15th 2006
By Clearing

2,630
5,910
Oct. 18th 2006
To Cheque
530

5,380
Oct. 27th 2006
To Self
2,690

2,690
Nov. 3rd 2006
By Cash

1,500
4,190
Dec. 6th 2006
To Cheque
950

3,240
Dec. 23rd 2006
By Transfer

2,920
6,160
If he received `198 as interest on 1st January, 2007, find the rate of interest paid by the bank.
[2012]
32

Universal Tutorials X ICSE Maths

Volume 1 of 2

Chapter 04: Banking (Saving and Recurring Account)

33
st

3) Mr.Choudhury opened a Savings Bank Account at State Bank of India on 1 April, 2007. The
entries of one year as shown in his pass book are given below:
Withdrawals
Deposits
Balance
Date
Particulars
(in `)
(in `)
(in `)
1st April, 2007
By Cash

8550.00
8550.00
12th April, 2007
To Self
1200.00

7350.00
24th April, 2007
By Cash

4550.00
11900.00
8th July, 2007
By Cheque

1500.00
13400.00
10th Sept., 2007
By Cheque

3500.00
16900.00
17th Sept., 2007
To Cheque
2500.00

14400.00
11th Oct., 2007
By Cash

800.00
15200.00
th
6 Jan., 2008
To Self
2000.00

13200.00
9th Mar., 2008
By Cheque

950.00
14150.00
If the bank pays interest at the rate of 5% per annum, find the interest paid on 1st April, 2008.
Give your answer correct to the nearest rupee.
[2011]
4) Ahmed has a recurring deposit account in a bank. He deposits `2,500 per month for 2 years. If
he gets `66,250 at the time of maturity, find
i) The interest paid by the bank
ii) the rate of interest.
[2011]
5) Mr. Gupta opened a recurring deposit account in a bank. He deposited `2500 per month for two
years. At the time of maturity he got `67,500. Find
i) the total interest earned by Mr. Gupta
ii) the rate of interest per annum.
[2010]
th
6) Mrs. Kapoor opened a Savings Bank Account in State Bank of India on 9 January 2008. Her
pass book entries for the year 2008 are given below:
[2010]
Withdrawals
Deposits
Balance
Date
Particulars
(in `)
(in `)
(in `)
Jan 9, 2008
By Cash

10,000
10,000
Feb 12, 2008 By Cash

15,500
25,500
April 6, 2008
To Cheque
3500

22,000
April 30, 2008 To Self
2000

20,000
July 16, 2008 By Cheque

6500
26,500
Aug 4, 2008
To Self
5500

21,000
Aug 20, 2008 To Cheque
1200

19,800
Dec 12, 2008 By Cash

1700
21,500
Mrs. Kapoor closes the account on 31st December, 2008. If the bank pays interest at 4% per
annum, find the interest Mrs. Kapoor receives on closing the account. Give your answer correct
to the nearest rupee.
7) Mr. Mishra has a Savings Bank Account in Allahabad Bank. His pass book entries are as
follows:
Date
Particulars
Withdrawals (`)
Deposits (`)
Balance (`)
Jan 4, 2007

By Cash

1000.00

1000.00

Jan 11, 2007

By Cheque

3000.00

4000.00

Feb 3, 2007

By Cash

2500.00

6500.00

Feb 7, 2007

To Cheque

2000.00

4500.00

March 3, 2007

By Cash

5000.00

9500.00

May 25, 2007

By Cash

2000.00

11500.00

June 7,2007

By Cash

3500.00

15000.00

Aug 29,2007

To Cheque

1000.00

14000.00

Volume 1 of 2

Universal Tutorials X ICSE Maths

33

34

Rate of interest paid by the bank is 4.5% per annum. Mr. Mishra closes his account on 30th
October, 2007. Find the interest he receives.
[2009]
8) Mrs. Goswami deposits `1,000 every month in a recurring deposit account for 3 years at 8%
interest per annum. Find the matured value.
[2009]
9) David opened a Recurring Deposit Account in a bank and deposited `300 per month for two
years. If he received `7725 at the time of maturity, find the rate of interest per annum.
[2008]
10) Mr.Dhoni has an account in the Union Bank of India. The following entries are from his pass
book:
Date
Particular
Withdrawals (`)
Deposits (`)
Balance (`)
Jan, 3, 07

B/F

Jan, 16

To self

March, 5

By Cash

April, 10

To self

April, 25

By Cheque

June, 15

By Cash

2642.00

640.00

2002.00

850.00

2852.00

1130.00

1722.00

650.00

2372.00

577.00

1795.00

Calculate the interest from January 2007 to June 2007 at the rate of 4% per annum.

[2008]

11) Saloni deposited `150 per month in her bank for eight months under the Recurring Deposit
Scheme. What will be the maturity value of her deposit, if the rate of interest is 8% per annum
and the interest is calculated at the end of every month?
[2007]
12) Mrs. Kumar has an account with The Bank of India. The following entries are from his pass book:
Date
Particular
Withdrawals (`)
Deposits (`)
Balance (`)
08.02.06

B/F

18.02.06

To self

12.04.06

By Cash

15.06.06

To self

08.07.06

By cash

8500.00

4000.00

2238.00

5000.00

6000.00

Compute the above page of her pass book and calculate the interest for the six months,
February to July 2006, at 4.5% per annum.
[2007]
13) Mohan deposits `80 per month in a cumulative deposit account for six years. Find the amount
payable to him on maturity, if the rate of interest is 6% per annum.
[2006]
14) Mr. Ashok has an account in the Central Bank of India. The following entries are from her pass
book:
Date
Particular
Withdrawals (`)
Deposits (`)
Balance (`)
01.01.05
B/F

1200.00
07.01.05
By Cash

500.00
1700.00
17.01.05
To Cheque
400.00

1300.00
10.02.05
By cash

800.00
2100.00
25.02.05
To Cheque
500.00

1600.00
20.09.05
By cash

700.00
2300.00
21.11.05
To Cheque
600.00

1700.00
05.12.05
By cash

300.00
2000.00
If Mr.Ashok gets `83.75 as interest at the end of the year where the interest is compounded
annually, calculate the rate of interest paid by the bank in his Savings Bank Account on 31st
December, 2005.
[2006]
15) Mr. R. K. Nair gets `6,455 at the end of one at the rate of 14% per annum in a recurring deposit
account. Find the monthly instalment.
[2005]
34

Universal Tutorials X ICSE Maths

Volume 1 of 2

Chapter 04: Banking (Saving and Recurring Account)

35

16) A page from the passbook of Mrs. Rama Bhalla is given below:
Date
Particular
Withdrawals (`)
Deposits (`)
Balance (`)
Jan, 1
B/F

2,000.00
Jan, 9
By Cash

200.00
2200.00
Feb, 10
To Cheque
500.00

1700.00
Feb, 24
By Cheque

300.00
2000.00
July 29
To Cheque
200.00

1800.00
Nov, 7
By cash

300.00
2100.00
Dec, 8
By cash

200.00
2300.00
Calculate the interest due to Mrs.Bhalla for the period from January 2004 to December 2004, at
the rate of 5% per annum.
[2005]
17) The entries in a Saving Bank passbook are as given below:
Date
Particular
Withdrawals (`)
Deposits (`)
Balance (`)
01.01.03
B/F

14000.00
01.02.03
By Cash

11500.00
25500.00
12.02.03
To Cheque
5000.00

20500.00
05.04.03
By cash

3750.00
24250.00
15.04.03
To Cheque
4250.00

20000.00
09.05.03
By cash

1500.00
21500.00
04.06.05
By cash

1500.00
23000.00
Calculate the interest for six months (January to June) at 4% per annum on the minimum
balance on or after the tenth day of each month.
[2004]

MISCELLANEOUS EXERCISE:
1) A page from a pass book of Saving Bank Account is given below:
Date
Particulars Withdrawals (`)
Deposits (`)
Balance (`)
09.08.1999 By Cash

10,000.00
10,000.00
11.08.1999 By Cheque

5,000.00
15,000.00
05.10.1999 To Cheque
12,000.00

3,000.00
10.10.1999 By Cash

17,000.00
20,000.00
27.11.1999 By Cheque
5,000.00

15,000.00
29.11.1999 By Cash

3,000.00
18,000.00
nd
The account is closed on 2 Jan., 2000. Find the amount received, if the rate of interest is 5%.
2) Calculate maturity value of a R.D. Account of:
i) `800 per month for 2 years at 8% p.a.
ii) `2,000 per month for 2.5 years at 10% p.a.
3) Ritu has a recurring deposit account in a bank and deposits `80 per month for 18 months. Find
the rate of interest paid by the bank if the maturity value of this account is `1,554.
4) Mohit holds a Saving Bank Account in State Bank of India for the months of June and July.
Following entries are noticed in his pass book.
Date Particulars Withdrawals (`)
Deposits (`)
Balance (`)
June 5 By Cash

2,400.00
2,400.00
June 9 By Cash

1,600.00
4,000.00
July 3 By Cheque

3,890.00
7,890.00
July 14 To Cheque
1,890.00

6,000.00
Calculate the interest earned upto 31st July at the rate of 5% per annum.
Volume 1 of 2

Universal Tutorials X ICSE Maths

35

36

5) John had a S.B. Account in Bank of Baroda. His passbook had the following entries:
Date Particulars Withdrawals (`)
Deposits (`)
Balance (`)
Jan. 1 By Balance

9,600.00
Jan. 8 By Cash

6,000.00
15,600.00
Feb. 18 To Cheque
10,500.00

5,100.00
May 19 By Cash

6,300.00
11,400.00
July 15 To Self
2,400.00

9,000.00
Oct. 7 By Cash

3,600.00
12,600.00
On October 30, 2000 he received his Transfer order and closed the account. Find the interest he
received on closing the account on 30th Oct. 2000; when the rate of interest was 5% per annum.
6) Pramod deposits `600 per month in a recurring deposit account for 4 years. If the rate of interest
is 8% per year; calculate the maturity value of his account.
7) The maturity value of a R.D. Account is `16,176. If the monthly installment is `400 and the rate
of interest is 8%; find the time (period) of this R.D. Account.
8) Mr. Bajaj needs `30,000 after 2 years. How much money (in multiple of `5) should he deposit
every month in a recurring deposit account to get required money after 2 years, the rate of
interest being 8% p.a.?
9) Rishabh has a recurring deposit account in a post office for 3 years at 8% p.a. simple interest. If
he gets `9,990 as interest at the time of maturity, find:
i) the monthly instalment
ii) the amount of maturity
10) Gopal has a cumulative deposit account and deposits `900 per month for a period of 4 years. If
he gets `52,020 at the time of maturity, find the rate of interest
11) Seep has a 4 year recurring deposit account in a bank and deposits `1,800 per month. If she
gets `1,08,450 at the time of maturity, find the rate of interest.
12) Mr. Verma opened a Savings Bank Account with the State Bank of India on 5th April, 2007 with
`8,500. For the financial year 20072008, the other transaction with the bank are as given
below:
ii) 27072007; withdrew `2,400
i) 15052007; deposited `3,700
iii) 06092007; withdrew `1,600
iv) 18122007; deposited `4,500
vi) 23032008; account closed.
v) 29012008; deposited `1,900
Taking the rate of interest as 6% per annum, find the amount that Mr.Verma gets on closing the
account.
13) A and B both opened recurring deposit accounts in a bank. If A deposited `1,200 per month for 3
years and B deposited `1,500 per month for 2 years; find, on maturity, who will get more
amount and by how much? The rate of interest paid by the bank is 10% per annum.

ANSWER TO UNSOLVED EXERCISE:


CW Exercise 4.1
1) (a) `1,500 (b) `550
3) 6%
5) `33.85

6) `6,391.95

HW Exercise : 4.1
1) (i) Nil (ii) `1,800 (iii) `1,300 (iv) 1,300
4) i) 70.86 ii) `70.75
5) `377.40

36

2) `5,000; `5,000
4) i) `81.71 ii) `81.74
7) `535

2) `249.71
6) `398.33

Universal Tutorials X ICSE Maths

3) `301.46

Volume 1 of 2

Chapter 04: Banking (Saving and Recurring Account)

CW Exercise 4.2
1) (i) `930 (ii) `4,338.56 (iii) `23,036.40
4) `1,236
5) 10 months
HW Exercise 4.2
1) 13,050

2) `1,000

37

2) `44,064

3) `56.67

3) 10%

4) 9%

Previous Year Board Question:


2) 4.5%
3) `657
1) `8421
4) (i) `6,250 (ii) 10%
5) i) `7,500 ii) per annum = 12%
8) `345
9) 7%
10) `42.48
12) `111.43
13) `6811.20
14) 5%
16) `97.92
17) `398.33
Miscellaneous:
1) `18.325
4) `41.67
8) `1,155
12) 15,144.50

Volume 1 of 2

2) (i) `20,800 (ii) `67,750


5) `310
6) `33,504
9) i) `2250 (ii) 90,990 10) 10%
13) B, `952.50

Universal Tutorials X ICSE Maths

6) `682
11) `1236
15) `500

7) `40,440

3) 10% p.a.
7) 3 years
11) 12.5%

37

38

Chapter 05: Shares and Dividend


Shares:
To establish a big company, a large sum of money is required. It is sometimes not possible for
an individual to invest such a big amount. Then some persons, interested in the company, join
together. They divide the estimated value into small parts ranging from Re 1 to `100. Each part
so obtained is called a share and the value fixed for each share is called its original or
Nominal Value (N.V.). The persons who purchase shares are called share-holders.
The price of a share at any particular time is called its Market Value (M.V.).
The market value of a share can be the same, more or less the nominal value of the share
depending upon the performance and profits of the company.
z If the market value of a share is the same as its nominal value, the share is said to be at
par.
z If the market value of a share is more than its nominal value, the share is said to be above
par or at premium.
z If the market value of a share is less than its nominal value, the share is said to be below
par or at discount.
The profit, which a shareholder gets (out of the profits of the company) from his investment
from the company, is called dividend.
The dividend is always expressed as a percentage of the nominal value of the share.
The percentage dividend is always calculated on the nominal value of the share certificate
regardless of the market value.

Formulae:
z

Sum invested = No. of shares bought M.V. of 1 share

Note: If the share is at par, Market Value = Nominal value i.e. M.V. = N.V.
Total investment
[when shares are allotted by the company]
Nominal Value of a Share

No. of shares =

Also, No. of shares =

Annual dividend or annual income per share = Rate of dividend Nominal value of a share

Rate of income on investment =

Total annual income = Annual income per share number of shares.

Total investment
[when shares are purchased from market]
Market Value of a Share
Dividend 100
Investment

SOLVED EXAMPLES 5.1:


1) By investing `7,500 in a company paying 10 percent dividend, an annual income of `500
is received. What price is paid for each of `100 share?
Sol: Let the price paid for each `100 share be `x
Total amount invested = `7,500
7,500
No. of shares bought =
x
Dividend paid = 10%
38

Universal Tutorials X ICSE Maths

Volume 1 of 2

Chapter 05: Shares and Dividend

39

7,500
7,500
10
75,000
share =

100 =
x
x
100
x
75,000
750
But amount received = `500
500 =
x=
= `150
x
5
Price of each share is `150
2) Jay Invested `6.250 in shares of a company paying 6% dividend per annum. If he bought
`25 shares for `31.25 each, find his income from the investment.
Sol: Since, the market value of each share = `31.25 and the sum invested is `6,250
6,250
= 200
No. of shares bought by Jay =
31 .25

Dividend paid on

No. of shares bought =

sum invested
M.V .of 1share

Income (dividend) on one share = 6% of N.V. =

3)

Sol:

4)

Sol:

6
`25 = 1.50
100

Therefore, his total income = 200 `1.50 = 300


Directly: Income = No. of shares Rate of div Face value (N.V.) of each share
= 200 6% 25 = 300
`67,200 are invested in `100 shares which are quoted at `120. Find the income if 12%
dividend is declared on the shares.
Sum invested = `67,200 and M.V of each share = `120
67,200
= 560
No of shares bought =
120
Given: dividend (income) on 1 share = 12% of N.V = 12% of `100 = 12
Total income from the shares = 560 12 = 6,720
A man wants to buy 62 shares available at `132 (par value being `100)
i) How much he will have to invest?
ii) If the dividend is 7.5%, what will be his annual income?
iii) If he wants to increase his annual income by `150, how many extra shares should he
buy?
i) He will have to invest = 62 `132 = `8,184
ii) Dividend on 1 share = 7.5% of `100 = 7.50 = His annual income = 62 7.50 = 465
iii) The man wants to increase his income by `150 and the income on one share = `7.50
150
= 20
The no. of extra shares he must buy =
7.50

UNSOLVED EXERCISE 5.1:


CW Exercise:
1) How much money will be required to buy 200, `25 shares at a premium of `2?
2) A person buys 120 shares at a nominal value of `40 each, which he sells at `42.50 each. Find
the profit and profit percent.
3) A man invests `800 in buying `5 shares and when they are selling at a premium of `1.15, he
sells all the shares. Find his profit and profit percent.
4) A man invests `3,072 in a company paying 5% per annum, when its `10 share can be bought for
`16 each. Find:
i) his annual income.
ii) his percentage income on his investment.
Volume 1 of 2

Universal Tutorials X ICSE Maths

39

40

5) A man buys `50 shares of a company, paying 12 per cent dividend, at a premium of `10. Find:
i) the market value of 320 shares;
ii) his annual income;
iii) his profit percent.
6) A man has 300, `50 shares of a company paying 20% dividend. Find his net income after paying
3% income tax.
7) A man invests `8,800 in buying shares of a company of face value of rupees hundred each at a
premium of 10%. If he earns `1,200 at the end of the year as dividend, find:
i) the number of shares he has in the company ii) the dividend percent per share.
HW Exercise
1) How much money will be required to buy 125, `30 shares of a discount of `3?
2) Find the cost of 85 shares of `60 each when quoted at `63.25.
3) Find the annual income derived from 250, `60 shares paying 5% dividend.
4) A man invests `7,770 in a company paying 5 per cent dividend when a share of nominal value of
`100 sells at a premium of `5. Find:
i) the number of shares bought
ii) annual income
iii) percentage income
5) A man buys `75 shares at a discount of `15 of a company paying 20% dividend. Find:
i) the market value of 120 shares;
ii) his annual income; iii) his profit percent
6) A company pays a dividend of 15% on its ten rupee shares from which it deducts income tax at
the rate of 22%. Find the annual income of a man who own thousand shares of this company.
7) A man invests `1,680 in buying shares of nominal value `24 and selling at 12% premium. The
dividend on the shares is 15% per annum. Calculate:
i) the number of shares he buys;
ii) the dividend he receives annually.

SOLVED EXAMPLES 5.2:


1) A man invests `20,020/- in buying shares of nominal value `26/- at 10% premium. The
dividend on the shares is 15% per annum. Calculate:
i) The number of shares he buys.
ii) The dividend he receives annually.
iii) The rate of interest he gets on his money.
10
Sol: Nominal value of each share = `26/
Premium = 10% =
26 = `2.60
100
Market value (purchase price) of each share = `26 + 2.60 = `28.60
Amount invested = `20,020/
Amount invested
20,020
=
= 700
i) The number of shares bought =
purchase price/ share
28.60

Number of shares bought = 700


ii) Dividend per annum = 15%
15
700 26 = 2,730
Dividend on 700 shares =
100
Dividend he receives annually = `2,730
Dividend received annually
100
iii) The rate of interest he gets annually =
Total invested
2,730
=
100 = 13.64%
20,020
Rate of interest on his money = 13.64%

40

Universal Tutorials X ICSE Maths

Volume 1 of 2

Chapter 05: Shares and Dividend

41

2) How much should a man invest in `50 shares selling at `60 to obtain an annual income of
`900, if the dividend declared is 15 per cent?
Sol: Dividend on 1 share = 15% of `50 = 7.50
900
Total dividend
=
= 120
Number of shares bought =
Div .on 1share
7.50

Since market value of each share = `60


Sum invested by the man = 120 60 = `7,200
3) A dividend of 9% was declared on `100 share selling at a certain price. If the rate of return
is 7.5%, calculate:
i) The market value of the share;
ii) The amount to be invested to obtain an annual dividend of `630.
Sol: i) Let M.V. of a share = ` X
Since, Rate of return M.V. = Rate of dividend N.V
7 .5
9
`x=
`100 x = 120 i.e. M.V of a share = `120

100
100
ii) Annual income on 1 share = 9% of `100 = `9
630
Total annual income
=
= 70
Since, No. of shares bought =
Annual income on 1share
9

4)
Sol:

5)

Sol:

and, the amount to be invested = No. of shares bought M.V of 1 share


= 70 120 = `8,400
Which is a better investment: 12% at 120 or 8% at 90?
12% at 120 means; M.V. of 1 share is `120, N.V. of 1 share is `100 & dividend paid is 12%
Similarly, 8% at 90 means;MV of 1 share is `90;NV of 1 share is `100 & dividend paid is 8%
In the first case: Income on `120 = 12% of `100 = 12
12
= `0.10
Income on Re.1 =
120
In the second case: Income of `90 = 8% of `100 = 8
8
= `0.09
The first investment is better.
Income on Re 1 =
90
Mr.Ram Gopal invested `8,000 in 7% `100 shares at `80. After a year he sold these shares
at `75 each and invested the proceeds (including his dividend) in 18%, `25 shares at `41.
Find:
i) his dividend for the first year
ii) his annual income in the second year
iii) the percentage increase in his return on his original investment.
[2006]
Given: investment = `8,000, div% = 7%, N.V. = `100 and M.V. = `80
8,000
Investment
=
= 100
i) No. of shares =
M .V .of each share
80
Div. on 1 share = 7% of `100 = ` 7
His dividend for the first year = `7 100 = `700
ii) Since, each share is sold for `75
The proceeds (including dividend) = 100 75 + 700 = `8,200
N.V. of each share = `25
Now, the sum invested = `8,200
M.V. of each share = `41 and, dividend = 18%

Volume 1 of 2

Universal Tutorials X ICSE Maths

41

42

8,200
= 200
41
18
Div. on 1 share = 18% of `25 =
25 = `4.50
100
Annual dividend (income) in the second year = 200 4.50 = `900
iii) Since, increase in return = 900 700 = `200
200
100% = 2.5%
Percentage increase in return (on the original investment) =
8,000

No. of shares bought =

UNSOLVED EXERCISE 5.2:


CW Exercise:
1) A man buys 75, `100 shares of a company which pays 9 per cent dividend. He buys shares at
such a price that he gets 12% of his money. At what price did he buy the shares?
2) Hundred rupee shares of a company available in the market at a premium of `20. Find the rate
of dividend given by the company, when a mans return on his investment is 15% per cent.
3) A company declares 8 per cent dividend to the shareholders. If a man received `2,840 as his
dividend, find the nominal value of his shares.
4) A company declares a dividend of 11.2% to all its share-holders. If its `60 share is available in
the market at a premium of 25%, find how much should Rakesh invest, in buying the shares of
this company, in order to have an annual income of `1,680?
5) A man buys 400, twenty rupee shares at a discount of 20% and receives a return of 12% on his
money. Calculate:
i) the amount invested by him
ii) the rate of dividend paid by the company.
6) A lady holds 1800, `100 shares of a company that pays 15% dividend annually. Calculate her
annual dividend. If she had bought these shares at 40% premium, what is the return she gets as
percent on her investment (give your answer to the nearest integer).
7) Mr.Sharma has 60 shares of nominal value `100 and decides to sell them when they are at a
premium of 60%. He invests the proceeds in shares of nominal value `50, quoted at 4%
discount, and paying 18% dividend annually. Calculate:
i) the sale proceeds
ii) the number of shares he buys and
iii) his annual dividend from the shares.
8) Which is the better investment: 16% at 80 or 20% at 120?
9) A man bought 360, ten-rupee shares paying 12 percent per annum. He sold them when the price
rose to `21 and invested the proceeds in five-rupee shares paying 4.5 per cent per annum at
3.50 per share. Find the annual change in his income.
10) Two brothers A and B invest `16,000 each in buying shares of two companies. A buys 3%
hundred-rupee shares at 80 and B buys ten-rupee shares at par. If they both receive equal
dividends at the end of the year, find the rate per cent of the dividend received by B.
11) Divide `50,760 into two parts such that if one part is invested in 8% `100 shares at 8% discount
and the other in 9% `100 shares at 8% premium, the annual incomes from both the investments
are equal.
1
12) Mr.Shameem invested 33 % of his savings in 20% `50 shares quoted at `60 and the
3
remainder of the savings in 10% `100 shares quoted at `110. If his total income from these
investments is `9,200; find:
i) his total savings
ii) the number of `50 shares.
iii) the number of `100 shares.
42

Universal Tutorials X ICSE Maths

Volume 1 of 2

Chapter 05: Shares and Dividend

43

HW Exercise:
1) By purchasing `25 gas shares for `40 each, a man gets 4 per cent profit on his investment.
What rate per cent is the company paying? What is his dividend if he buys 60 shares?
2) `50 shares of a company are quoted at a discount of 10%. Find the rate of dividend given by the
company, the return on the investment on these shares being 20 per cent.
3) How much should a man invest in `100 share selling at `110 to obtain an annual income of
`1,680, if the dividend declares is 12%?
4) A man buys 400 twenty rupee shares on a premium of `4 each and receives a dividend of 12%.
Find:
i) the amount invested by him
ii) his total income from the shares.
iii) percentage return on his money
5) A company with 10,000 shares of `100 each, declares an annual dividend of 5%.
i) What is the total amount of dividend paid by the company?
ii) What should be the annual income of a man who has 72 shares, in the company?
iii) If he received only 4% of his investment, find the price he paid for each share.
6) A man invests `11,200 in a company paying 6 per cent per annum when its `100 shares can be
bought for `140. Find:
i) his annual dividend
ii) his percentage return on his investment.
7) A company with 10,000 shares of nominal value `100 declares an annual dividend of 8% to the
shareholders.
i) Calculate the total amount of dividend paid by the company.
ii) Ramesh had bought 90 shares of a company at `150 per share. Calculate the dividend he
receives and the percentage return on his investment.
8) A man has a choice to invest in hundred-rupee shares of two firms at `120 or at `132. The first
firm pays a dividend of 5% per annum and the second pays dividend of 6% per annum. Find
i) Which company is giving a better return?
ii) If a man invests `26,400 with each firm, how much will be the difference between the annual
returns from the two firms.
9) A man sold 400 (`20) shares, paying 5% of `18 and invested the proceeds in (`10) shares,
paying 7% at `12. How many (`10) shares did him buy and what was the change in his income?

PREVIOUS BOARD QUESTION:


1) A man invests `9,600 on `100 shares at `80. If the company pays him 18% dividend find:
i) the number of shares he buys.
ii) his total dividend
iii) his percentage return on the shares.
[2012]
2) Mr.Parekh invested `52,000 on `100 shares at a discount of `20 paying 8% dividend. At the end
of one year he sells the shares at a premium of `20. Find
i) The annual dividend ii) the profit earned including his dividend.
[2011]
3) Vivek invests `4500 in 8%, `10 shares at `15. He sells the shares when the price rises to `30,
and invests the proceeds in 12% `100 shares at `125. Calculate
i) the sale proceeds
ii) the number of `125 shares he buys
iii) the change in his annual income from dividend
[2010]
4) Amit Kumar invests `36,000 in buying `100 shares at `20 premium. The dividend is 15% per
annum. Find:
i) The number of shares he buys
ii) His yearly dividend
iii) The percentage return on his investment.
Give your answer correct to the nearest whole number.
[2009]
Volume 1 of 2

Universal Tutorials X ICSE Maths

43

44

5) A company with 4000 shares of nominal value of `110 each declares an annual dividend of 15%.
Calculate:
[2008]
i) The total amount of dividend paid by the company.
ii) The annual income of Shahrukh who holds 88 shares in the company.
iii) If he received only 10% on his investment, find the price Shahrukh paid for each share.
6) Ajay owns 560 shares of a company. The face value of each share is `25. The company
declares a dividend of 9%. Calculate:
i) The dividend that Ajay will get.
[2007]
ii) The rate of interest on his investment, if Ajay had paid `30 for each share.
7) Mr.Tiwari invested `29040 in 15% `100 shares quoted at a premium of 20%. Calculate:
i) The number of shares bought by Mr.Tiwari. ii) Mr. Tiwaris income forms the investment.
iii) The percentage return on his investment.
[2005]
8) A man invested `45000/ in 15% `100/ shares quoted at `125/. When the market value of
these shares rose to `140/ he sold some shares, just enough to raise `8400/ calculate:
i) The number of shares he still holds;
ii) The dividend due to him on these remaining shares.
[2004]

MISCELLANEOUS EXERCISE:
1) A man bought `40 shares at a premium of 40%. Find his income, if he invests `14,000 in these
shares and receives a dividend at the rate of 8% on the face value of the shares.
2) A sum of `11,880 is invested in `50 shares available at 12% discount. Find the income, if a
dividend of 12% is given on the shares.
3) Peter invests `5,625 in a company paying 7% per annum when a share of `10 stands for `12.50.
Find Peters income from this investment If he sells 60% of these shares for ` 10 each. Find his
gain or loss in this transaction.
4) A company gives x% dividend on its `60 shares, whereas the return on the investment in these
shares is (x + 3)%. If the market value of each share is `50, find the value of x.
5) A dividend of 10% was declared on shares with face value `60. If the rate of return is 12%,
calculate:
i) the market value of the share
ii) the amount to be invested to get an annual income of `1,200.
6) A man bought `40 shares at a discount of 40%. Find his income, if he invests `12,000 in these
shares and receives a dividend at the rate of 11% on the face value of the shares.
7) Rajat buys `80 shares at 30% premium in a company paying 18% dividend. Find:
i) the market value of 150 shares
ii) Rajats annual income from these share
iii) Rajats percentage return from this investment.
8) Mrs.Sharma buys 85 shares (par value `100) at `150 each.
i) If the dividend is 6.5%, what will be her annual income.
ii) If she wants to increase her income by `260; how much more should she invest?
9) How much should a man investing `100 shares selling at `85 to obtain an annual income of
`1,800; if the dividend declared is 12%? Also, find the percentage return on his investment.
10) Mr.Gupta has a choice to invest in ten-rupee shares of two firms at `13 or at `16. If the first firm
pays 5% dividend and the second firm pays 6% dividend per annum, find:
i) which firm is paying better.
ii) if Mr. Gupta invests equally in both the firms and the difference between the returns from
them is `30; find how much, in all does he invest.

44

Universal Tutorials X ICSE Maths

Volume 1 of 2

Chapter 05: Shares and Dividend

45

11) A man invests a sum of money in `100 shares paying 15% dividend quoted at 20% premium. If
his annual dividend is `540. Calculate,
i) His total investment.
ii) The rate of return on his investment.
12) A man bought some shares of `50 each paying 15% dividend at a premium of 10%. If the annual
dividend is `270. Calculate
i) The number of shares.
ii) The investment made
iii) The % interest earned by him (correct to the nearest integer)
13) A man bought `100 shares at `130 and earned an annual income of `2340 which is 12% of the
rate of return on his money. Calculate
i) The number of shares bought.
ii) The rate of dividend he got.
14) A person invested his savings as follows:
a) 20% of his savings in buying 10% `100 shares of a company A at `160
b) 60% of his savings in buying 6% `50 shares of company B at `60.
c) 20% of his savings in buying 5% `100 shares of company C at `80. Calculate
i) the number of shares of company B and C bought by him, given that he obtained 40
shares of company A.
ii) the total dividend earned by him at the end of the year.
iii) the over all percentage return on his entire investment.
15) A man wishes to invest `25,000, he invests `1,000 in 9% `100 shares at 20% premium and
`7,200 in 8% `100 shares at 10% discount and the remainder in 12%. If the total income from
the investment is 10% at what price did he buy the 12% shares?
16) A man invested `8,000 in 8% `100 shares selling at 20% discount. After a year he sold these
shares at 25% discount and invested the proceeds in `90 shares selling at `10 premium and
giving 12% dividend. Calculate:
a) His income from the first investment.
b) His income from the second investment.
c) The increased percentage return on his original investment.
17) A man buys 400 `20 shares at a discount of `4 each. He receives a return of 12% on his money.
Calculate
a) The amount invested by him.
b) The rate of dividend paid by the company.
18) A person invested 20%, 30%, 25% of his savings in buying shares of three different companies
a, b and c which declares dividends of 10%, 12% and 15% respectively. If his total income on
account of dividends be `2337.5. Calculate
i) His savings.
ii) The amount he invested in buying shares of each company.
19) A person invested `8000 and `10000 in buying shares of two companies which later on declared
dividends of 12% and 8% respectively. He collects the dividends and sells out his shares at a
loss of 2% and 3% respectively on his investment. Find the total earning from the above
transaction.
20) A man invests on his `2160 in shares at `9 each and sells them at `12. Calculate the profit %
from the investment.
21) Mrs. P. Chandra invested `19,200 in 15% `100 shares at 20% discount. After a year, she sold
these shares at `90 each and invested the proceeds (including her dividend) in 20%, `50 shares
at `42. Find:
i) the dividend for the first year.
ii) her annual income in the second year.
iii) the percentage change in her return on her original investment.
22) Govind invested `19,200 in 15% `100 shares at 20% premium. After a year, he sold these
shares at `140 each and invested the proceeds (including his dividend) in 20%, `20 shares at
`16. Find:
i) the dividend for the first year.
Volume 1 of 2

Universal Tutorials X ICSE Maths

45

46

ii) her annual income in the second year.


iii) the percentage change in her return on her original investment.
23) A dividend of 12% was declared on `150 shares selling at a certain price. If the rate of return is
10%, calculate:
i) the market value of the shares.
ii) the amount to be invested to obtain an annual dividend of `1,350.

ANSWER TO UNSOLVED EXERCISE:


CW Exercise 5:1
1) `5,400

2) `300 and 6.25%

5) (i) `19,200 (ii) `1,920 (iii) 10%

3) `184 and 23%

4) (i) `96 (ii) 3.125%

6) `2,910

7) (i) 80 (ii) 15%

3) `750

4)(i)74 (ii)`370 (iii)4.76%

6) 1,170

7) (i) 62.5 (ii) 225

HW Exercise 5:1
1) `3,375

2) `5,376.25

5) (i) `7,200 (ii) `1,800 (iii) 25%


CW Exercise 5:2
1) `75 each 2) 18%

3) `35,500

4) `18,750

5) (i) `6,400 (ii) 9.6%

6) 27,000; 11%

7) (i) 9,600 (ii) 200 (iii) 1,800

8) 16% at 80

9) Increase `54

10) 3.75%

11) `24,840 and `25,920

12) i) `79,200 ii) 440 iii) 480

HW Exercise 5:2
1) 6.4%; `96 2) 18%

3) `15,400

4) (i) `9,600 (ii) `960 (iii) 10%


6) (i) 480 (ii) 4.29%

5) (i) `50,000 (ii) `360 (iii) `125

7) (i) `80,000 (ii) `720 (iii) 5 13 %

8) (i) Second (ii) `100

9) 600; increase in income = `20


Previous Year Board Question:
1) i) 120 (ii) `2,160 (iii) `22.5%

2) i) `5,200 (ii) `31,200

3) i) `9,000 (ii) `72 (iii) `624

4) i) 300 (ii) `4500 (iii) 13%

5) i) `66000 (ii) `1452 (iii) `165

6) i) 1260 ii) 7.5%

7) i) `242 (ii) `3630 (iii) `12.5%

8) i) 300 ii) 4500 `

Miscellaneous:
1) `800

2) 1,620

3) 315; 625 less

5) (i) 50 (ii) 10,000

6) 2,200

7) (i) 15,600 (ii) 2,160 (iii) 13.85%

8) (i) 552.50 (ii) 6,000

9) 12,750; 14.12%

10) (i) first firm (ii) 62,400

11) (i) `4320 (ii) 12.5%

12) (i) 36 (ii) `1980 (iii) 14%

14) (i) 320, 80 (ii) `1760 (iii) 5.5%


1

20) 33 /3%

22) (i) `2,400 (ii) `6,200 (iii) increases = 19

46

13) (i)150 (ii) 15.6%

15) `112.94

16) (a) 800 (b) 810 (c) 1/8%17) a) `6400 (b)9.6%


19) `1300

4) 15

18) `25000, `5000, `7500, `6250


21) i) 3,600 (ii) `6,000 (iii) increases = 12.5%

19
% = 19.792%
24

Universal Tutorials X ICSE Maths

23) (i) `180 (ii) `13,500

Volume 1 of 2

Chapter 06: Linear Inequations (in one variable)

47

Chapter 06: Linear Inequations(In one variable)


Introduction:
The mathematical statement, which says that one quantity is not equal to another quantity, is
called an inequation.
In an inequation, the signs >, <, and are called signs of inequality.
If x and y are two quantities such that xy, then any one of the following four conditions will be true

i) x > y
ii) x y
iii) x < y
Each of the four conditions given above is an inequation.
Eg. x < 8, x 5, x + 4 3, x + 8 > 4 etc.

or

iv) x y.

Replacement Set and Solution Set:


The set, from which the value of the variable x is to be chosen, is called the replacement set and
its subset, whose elements satisfy the given inequation, is called the solution set.
E.g.
Let the given inequation be x < 4
i) The replacement set = N= the set of Natural Numbers
The solution set = {1,2,3}
ii) If the replacement set = W, the set of Whole Numbers
then the solution set = {0, 1, 2, 3}
iii) If the replacement set = Z or I, the set of all integers,
then the solution set = {, 2, 1, 0, 1, 2, 3}
iv) If the replacement set = R, the set of all Real Numbers,
then the solution set = {x : x R and x < 4}

Solving a Linear Inequation Algebraically:


z
z

To solve a given linear inequation means to find the value or values of the variable
used in it.
Rule 1: On transferring a positive term (the term in addition) from one side of an inequation
to its other side, the sign of the term becomes negative.

Eg. 2x + 3 > 7 2x > 7 3


Rule 2: On transferring a negative term (the term in subtraction) from one side of an
inequation to its other side, the sign of the term becomes positive.
Eg. 2x 3 > 7 2x > 7 + 3
Rule 3: If each term of an inequation be multiplied or divided by the same positive number,
the sign of inequality remains the same.
Case I: if p is positive and x < y
x<y

x
y
and <
p
p

x
y
and >
p
p
Rule 4: If each term of an inequation be multiplied or divided by the same negative number,
the sign of inequality reverses.

x>y

px < py

Volume 1 of 2

px > py

Universal Tutorials X ICSE Maths

47

48

Case II: if p is negative


x
y
and >
p
p
Rule 5: If sign of each term on both the sides of an inequation is changed, the sign of
inequality gets reversed.

x<y

px > py

Eg. x > 5 x < 5


Rule 6: If both the sides of an inequation are positive or both are negative then on taking
their reciprocals, the sign of inequality reverses.
i.e. if x and y both are either positive or both are negative then,
1
1
x>y
<
x
y

Note: Always reverse the sign of inequality when the inequation is multiplied or divided by a ve
number.

Point to Remember:
z
z
z
z
z

x < y and y > x represent the same inequation.


The sign of the inequality remains the same if the same number is added or substrated from
both sides of an inequation.
The sign of the inequality remains the same if the same positive is multiplied or divided from
both sides of an inequation.
The sign of the inequality reverses if the same negative number is multiplied or divided from
both sides of an inequation.
The sign of the inequality reverses if we take the reciprocal of both sides of the gives
inequality. Hence if x, y, z R and if x > y then,
x+z>y+z
xz>yz
xz>yz
if z > 0
xz<yz
if z < 0
x
y

>
if z > 0
z
z
x
y

<
if z < 0
z
z
1
1

<
y
x

Graphical Representation:
A real number line can be used represent the solution set of an inequation.
The convention is that o (a hollow circle) marks the end of a range with a strict inequality
(i.e.< or >) & (a darkend circle) marks the end of a range involving an equality as well (i.e. or )
For example: x < 2 and x R
3

48

x 2, and x R
3

Universal Tutorials X ICSE Maths

Volume 1 of 2

Chapter 06: Linear Inequations (in one variable)

49

SOLVED EXAMPLES 6.1:


x
1
1
1

1 <
: x R. Graph the values of x on the real number line.
3
2
3
6
Sol: The given inequation has two parts:
1
1
1
1
1
x
x

1 <
+1
3
2
3
6
3
3
2
x
1
2x
2
1
1
1
4
9
9
x
x
x
And
1 <

<
+

<
x<
2x<3
2
3
6
2
6
3
2
6
6
On simplifying, the given inequation reduces to 2 x < 3
1
0
1
2
3
and the required graph on number line is:
2) Solve and graph the solution set of 3x + 6 9 and 5x > 15; where x R.
Sol: 3x + 6 9
3x 9 6
3x 3
x1
5 x
15
<
x<3
And, 5x > 15

5
5
Graph for x 1 :
1
0
1
2
3
4
5

1) Simplify,

Graph for x < 3 :

Graph of solution set of x 1 and x < 3


1
0
1
2
Graph of points common to both x 1 and x < 3
3) Solve and graph the solution set of 2 < 2x 6 or 2x + 5 13; where x R.
Sol: 2 < 2x 6
2x 6 > 2
2x > 4
x>2
2x + 5 13
2x 8
x 4
Graph for x > 2 :
1
0
1
2
3
4
5
6

Graph for x 4 :
6
5
4
3
2
1
0
1
2
3
Graph of solution set of x > 2 and x 4
= Graph of points which belong to x > 2 and x 4 or both
6

4) Given, P = {x : 5 < 2x 1 11, x R}; Q = {x : 1 3 + 4x < 23, x I};


where R = {real numbers} and I = {integers}.
Represent P and Q on two different number lines. Write down the elements of P Q.
Sol: For P : 5 < 2x 1 11, where x R
5 < 2x 1 and 2x 1 11
3 < x and x 6
i.e. 3 < x 6; x R

P=
For Q :

1 3 + 4x < 23, x I
1 x and x < 5

1 3 + 4x and 3 + 4x < 23
i.e. 1 x < 5; x I

4
3
2
1
0
1
2
3
4
5
Q=
Hence, P Q = {elements common to both P and Q} = {4}

Volume 1 of 2

Universal Tutorials X ICSE Maths

49

50

UNSOLVED EXERCISE 6.1:


CW Exercise:
1) If the replacement set is the set of whole numbers, solve:
1
iii) 18 3x 2
i) 3x 1 > 8
ii) 7 3x
2
2) If 25 4x 16, find,
i) the smallest value of x, when x is a real number
ii) the smallest value of x, when x is an integer
3) Represent the following inequalities on real number lines:
i) 2x 1 < 5
ii) 4 < x < 4
iii) 8 x > 3
4) For each graph given below, write an inequation taking x as the variable:

i)

ii)

5) For the following inequations, graph the solution set on the real number line,
i) 4 3x 1 < 8
ii) x 1 < 3 x 5
6) List the elements of the solution set of the inequation 3 < x 2 9 2x; x N.
1
2x
5
7) Find the values of x, which satisfy the inequation, 2

1 , x N.
[2007]
2
3
6
Graph the solution on the number line.
8) If 5x 3 5 + 3x 4x + 2, express it as a x b and then state the values of a and b.
9) Solve and graph the solution set of:
i) 3x 2 > 19 or 3 2x 7; x R
ii) 5 > p 1 > 2 or 7 2p 1 17; p R.
10) The diagram represents two inequations A and B on real number lines:
A=

B=
5
4
3
2
1
0
1
2
3
4
5
i) Write down A and B in set builder notation
ii) Represent A B and A B on two different number lines
11) P is the solution set of 7x 2 > 4x + 1 and Q is the solution set of 9x 45 5(x 5); where x
R. Represent:
i) P Q
ii) P Q
iii) P Q on different number lines
1 x
2
1
12) Find the range of values of x, which satisfy: + 1 < 5 .
3 2
3
6
Graph, in each of the following cases, the values of x on the different real number lines:
i) x W
ii) x Z
iii) x R
13) Find the three consecutive largest positive integers such that the sum of one-third of first, onefourth of second and one-fifth of third is atmost 20.
HW Exercise
1) If the replacement set is the set of whole numbers, solve:
3 3
i) x + 7 11
ii) 8 x > 5
iii) x < x
2 2
2) Solve the inequation, 3 2x x 12 given that x N.
50

Universal Tutorials X ICSE Maths

Volume 1 of 2

Chapter 06: Linear Inequations (in one variable)

51

3) Represent the following inequalities on real number lines:


i) 3x + 1 5
ii) 2(2x 3) 6
iii) 2 x < 5
4) For each graph given below, write an inequation taking x as the variable:
i)

ii)

5) Represent the solution of each of the following inequalities on the real number line:
i) 4x 1 > x + 11
ii) x +3 2x + 9
iii) 1 + x 5x 11
6) x (real numbers) and 1 < 3 2x 7, evaluate x and represent it on number line.
2
1
1
7) Find the range of values of x which satisfies, 2 x +
< 3 , x R.
[2007]
3
3
3
Graph these values of x on the number line.
8) Use real number line to find the range of values of x for which,
i) x > 3 and 0 < x < 6
ii) x < 0 and 3 x < 1
iii) 1 < x 6 and 2 x 3
9) Given A = {x : 1 < x 5, x R} and B = {x : 4 x 3, x R}. Represent on different number
lines:
i) A B
ii) A B
iii) A B
10) Given A = {x : 8 < 5x + 2 17, x I} and B = {x : 2 7 + 3x < 17, x R}. Where R = {real
numbers} and I = {Integers}. Represent A and B on two different number lines. Write down the
elements of A B.
11) x and y are positive integers that satisfy:
i) 1 x 4
ii) 3x + 2y 13
List the possible values of y.
12) State whether the following statements are true or false:
i) If a < b, then a c < b c
ii) If a > b, then a + c < b + c
a
b
<
iii) If a < b, then ac > bc
iv) If a > b, then
c
c
v) If a c < b d, then a + d < b + c
vi) If a < b, and c > 0, then a c > b c
Where a, b, c and d are real numbers and c 0

PREVIOUS BOARD QUESTION:


1) Solve the inequation and represent the solution set on the number line
3x
2
2 < + x, where x R.
4x 19 <
5
5
2) Solve the inequation and represent the solution set on the number line
2x 5 5x + 4 < 11, where x I.
3) Solve the inequation and represent the solution set on the number line
1 2x 5
, x R.
3 <
2
3
6
4) Solve the inequations and represent the solution set on the number line
8x
14
+2
+ 2x, where x I.
3+x
3
3
Volume 1 of 2

Universal Tutorials X ICSE Maths

[2012]

[2011]

[2010]

[2009]
51

52

5) Solve the given inequation and graph the solution on the number line.
2y 3 < y + 1 4y + 7; y R.
[2008]
6) Solve the inequation and graph the solution on the number line.
2
1
1
[2007]
2 x + < 3 ; x R.
3
3
3
7) Given that x R, solve the following inequality and graph the solution 1 3 + 4x 23 on the
number line.
[2006]
8) A = {x: 11x 5 > 7x + 3, x R} and B = {x: 18x 9 15 + 12 x, x R}.
Find the range of set A B and represent in on a number line.
[2005]
9) Given that x I, solve the inequation and graph the solution on the number line:
x4 x
+ 2
[2004]
3
2
3

MISCELLANEOUS EXERCISE:
1) Represent the solution of each of the following inequalities on the real number line:
2 3x > 7 5x
2) Solve and graph the solution set of:
i) 2x 9 < 7 and 3x + 9 25; x R
ii) 2x 9 7 and 3x + 9 25; x I
iii) x + 5 4(x + 1) and 3 2x < 7; x R
3) If P = {x : 7x 4 > 5x + 2, x R} and Q = {x : x 19 1 3x, x R}; find the range of set P Q
and represent it on a number line.
4) Given x {real numbers}, find the range of value of x for which 5 2x 3 < x + 2 and represent
it on a real number line.
5) Illustrate the set {x : 3 x < 0 or x > 2; x R} on a real number line.
6) Solve 2 2x 3 5, x R and mark it on a number line.
7) Solve the following inequation and graph the solution set on the number line,
2x 3 < x + 2 3x + 5; x R.
8) Plot on the number line.
i) 3 < x < 4, x Z
ii) 4.5 < x < 3.2, xZ iii) 2 x < 3, x R
iv) 7 < x 1, x R
v) 5 x 8, x R
vi) 4.5 x 3.2, xR vii) 4 > x > 7, x R
viii 4 x > 0, x R
ix) 3 x 3, x R
x) 5 x > 3, x Z.
9) Solve and plot on the number line.
i) 3 + 2x > 4x 2 14, x R
ii) 4 > 2x 4 > 10, x N
iii) x 6 4x + 15 18 + x, x R
iv) 25 + x > 4 6x > x 24, x R
v) 8 2x 5x 1 10 2x, x R
vi) 1 < 3 2x 7, x R
vii) 3 2x x 12, x N
viii) x {3, 4, 5, 6} 9 1 2x, x = ?
ix) x and y are positive integers 1 x 4, 3x + 2y 13. Find the values of y.
x) State true or false.
a) If a > b then a c > b c
b) If a < b then ac < bc
a, b, c, d R
a
b
>
c0
c) If a > b then
c
c
d) If a c < b d then a + d < b + c
Where a, b, c, d R and c 0
10) P = {x : 5 < 2x 1 11, x R}
Q = {x : 1 3 + 4x < 23, x I}
Represent P and Q on the number line. Write down the elements of P Q.
11) R = {(x, y) : 2x + 3y < 10, x, y N} Write down R as ordered pairs.
12) f = {(x, y) : x, y N, y = 3x 2 and x 4} List f.
52

Universal Tutorials X ICSE Maths

Volume 1 of 2

Chapter 06: Linear Inequations (in one variable)

13)
14)
15)
16)
17)
18)
19)
20)
21)
22)

53

30 4 (2x 1) > 8, x {positive integers}. Solve and plot on the number line.
2 2x/3 15/6, x N. Solve and plot on the number line.
1/3 x/2 11/3 < 1/6, x R. Solve and graph the values on the real number line.
22/3 x + 1/3 < 31/3, x R. Solve and graph the values on the real number line.
2/3 < x/3 + 1 2/3, x R. Solve and graph the values on the real number line.
A = {x: 1 < x 5, x R} B = {x: 4 x < 3, x R}. Represent (i) A B (ii) A B on different
number lines.
1 < 2x + 1 7, x R. Represent the solution on a number line.
Solve and represent on a number line: 12 + 15/6x 5 + 3x, x R.
2x + 3 < 4x + 2 3x + 5, x R. Solve and plot on a number line.
List the solution set: 7m + 8 < 5m 4 < 10, m Z.

ANSWERS TO UNSOLVED EXERCISE:


CW Exercise 6.1:
1) i) {4, 5, 6, }
2) i) 2.25 or 9/4
3) i)

ii) {0, 1, 2}

iii) {7, 8, 9, }
ii) 3

ii)
iii)
4) i) x < 1, x R
5) i)
3

6)
7)
8)
9)
10)

ii) 4 x < 3; x R
1

{1, 2, 3}
{x N, 2 x 3.75} = {1, 2, 3}
3 x 4; a = 3 and b = 4
0
i) x > 7 or x 5
i) A = {x R; 2 x < 5}
ii) A B =
3
2
1
0
A B =

11) i) P Q = x 5

1
3

10

B = {x R; 4 x < 3}

.3

ii) P Q = 1 < x < 5

ii)

3
5

iii) P Q is same as ii
12) i) x W x < 7

ii) x Z 4 x < 7
iii) x R 4 x < 7

13) 24, 25, 26


Volume 1 of 2

Universal Tutorials X ICSE Maths

53

54

HW Exercise 6.1:
1) i) {0, 1, 2, 3, 4} (ii) {0, 1, 2} (iii) {0, 1}
3) i)
3

2) {x N, x 5} = {1, 2, 3, 4, 5}
ii)
3

iii)
4)
6)
7)
8)

i) x > 2; x R (ii) 1 < x 5; x R


2 x < 2
3
2
1
0
1
3 x < 3
i) 3 < x < 6 (ii) 3 x < 0 (iii) 1 < x 3

10) A =

5) i) x > 4 (ii) x 1 (iii) x 6


2

9) i) 1 < x < 3 (ii) 4 x 1 (iii) 3 x 5

B=

A B = {1, 0, 1, 2, 3}
11) 1, 2, 3, 4, or 5
Previous Board Question:
1)
2)

12) i) True (ii) True (iii) False (iv) False (v) True (vi) False

3) 2 x < 3.75

Solution on the number line


5

4) {x : 3 x 4; x I}

6) 3 x < 3

7) {x : 1 x 5, x R}

8) A B = {x: x 4, x R}

10

9) 6 x 4.8

5) 2 y < 4

Miscellaneous Exercise:
2) i) x 5
3) x 5
1

4) 1 x < 5
5)

6) 5/2 x 4

ii) {6, 7, 8}

3
0

2
1

1
2

iii) empty set


4

54

3.75

4
1

5
2

6
3

7
4

Universal Tutorials X ICSE Maths

Volume 1 of 2

Chapter 06: Linear Inequations (in one variable)

55

7) 1.5 x < 5
5
3
9) (i) x 4 (ii) 4 < x > 7 (iii) 7 < x
(iv) x < 4 (v) 3 x 3 (vi) 2 < x 2 (vii) 5 x (viii) x = 4
2
4
(ix) y = {1, 2, 3, 4, 5} (x) (a) y (b) y
10) 4
11) (1, 2), (2, 2), (3, 2)
12) (1, 1), (2, 4), (3, 7), (4, 10)
13) { }
14) { }
15) 2 x < 3
16) 3 x < 3
17) 5 < x 1
18) {1, 0, 1, 2}
19) {0, 1, 2, 3}
20) x 6
21) < x 3
22) x > 1

Volume 1 of 2

Universal Tutorials X ICSE Maths

55

56

Chapter 07: Quadratic Equations


Introduction:
An equation with one variable, in which the highest power of the variable is two, is known as
quadratic equation.
The standard form of a quadratic equation is ax2 + bx + c = 0, where a, b and c are all real
3 2
2
x 8x + 3 = 0,
numbers and a 0. Eg. x2 7x + 12 = 0,
3x +2 2x2 3=0
4
The value which satisfies a quadratic equation is the roots of a quadratic equation. Every
quadratic equation can have a maximum of two roots.

Zero product Rule:


If a and b are two numbers or expressions such that ab = 0 then either a = 0 or b = 0 or both a = 0
and b = 0.
Whenever the product of two expressions is zero; at least one of the expressions is zero.
Eg. If (x + 3) (x 2) = 0 then,
x + 3 = 0 or x 2 = 0 i.e. x = 3 or x = 2

Solving Quadratic Equation by factorisation:


z
z
z
z

Clear all fractions and brackets, if necessary.


Transpose all the terms to the left hand side to get an equation in the form ax2 + bx + c = 0.
Factorise the expression on the left hand side.
Put each factor equal to zero and solve.

SOLVED EXAMPLES 7.1:


1) Solve, 2x2 7x = 39.
Sol: 2x2 7x = 39
2x2 7x 39 = 0
x(2x 13) + 3(2x 13) = 0
2x 13 = 0
2) Solve,

or x + 3 = 0

2x2 13x + 6x 39 = 0
(2x 13) (x + 3) = 0
13
x=
or x = 3
2

x
x 1
1
+
=2 .
x 1
x
2

x 2 + (x 1)
x
x 1
1
5
+
=2

=
2(x2 + x2 2x + 1) = 5(x2 x)
x (x 1)
x 1
x
2
2
4x2 4x + 2 = 5x2 5x x2 x 2 = 0
[changing sign of each term]
x2 + x + 2 = 0
(x 2)(x + 1) = 0
[On factorizing]
x 2 = 0 or x + 1 = 0
[Zero Product Rule]
x = 2 or x = 1
3) Find the quadratic equation whose solution set is (2, 3).
Sol: Since, solution set = {2, 3}
x = 2 or x = 3
x + 2 = 0 or x 3 = 0
(x + 2) (x 3) = 0
x2 3x + 2x 6 = 0
x2 x 6 = 0
2

Sol:

56

Universal Tutorials X ICSE Maths

Volume 1 of 2

Chapter 07: Quadratic Equations

57
2

4) Use the substitution x = 3y + 1 to solve for y, if 5(3y + 1) + 6(3y + 1) 8 = 0.


5x2 + 6x 8 = 0
(Putting 3y + 1 = x)
Sol: 5(3y + 1)2 + 6(3y + 1) 8 = 0
4
(x + 2) (5x 4) = 0 (On factorizing)
x = 2 or x =
5
4
4
1
When x = 2 3y + 1 = 2 y = 1 and when x =
3y + 1 =
y=
5
5
15
1
y = 1 or y =
15

UNSOLVED EXERCISE 7.1:


CW Exercise:
1) Solve:
i) x2 10x 24 = 0

ii) x2 16 = 0

9
x = 5 + x2
vi) (2x 3)2 = 49
2
viii) x2 (a + b)x + ab = 0

v)

x)
xii)

x 3 x +3
1
+
=2
x +3 x 3
2
2x 2 x =

iii) 2x2

1
x=0
2

iv) x(x 5) = 24

vii) (x + 1) (2x + 8) = (x + 7) (x + 3)
ix) 4(2x 3)2 (2x 3) 14 = 0

1
1
7

xi) 1 +
1
=
x
x
+
1

1
8

x2 + 6

2) Find the quadratic equation, whose solution set is: (i) {3, 5} (ii) {2, 3}
3) Find the value of x, if a + 1 = 0 and x2 + ax 6 = 0.
4) Use the substitution y = 2x + 3 to solve for x, if 4(2x + 3)2 (2x + 3) 14 = 0.
HW Exercise:
1) Solve:
6
3x + 1
i)
=1+x
ii) x =
x
4x
1
= 2.5
iv) 2(x2 6) = 3(x 4)
iii) x +
x
3x 2 3x 8
vi)
=
v) (x + 3)2 4(x + 3) 5 = 0
2x 3
x+4
100 100
4
1
4
vii)

=1
viii)

=
x
x +5
x + 2 x + 3 2x + 1
5
3
4
x) 3x 2 + x + 5 = x 3
ix)

=
x 2 x +6 x
2) Find the quadratic equation, whose solution set is:

8
2

ii) 3,
iii) 0,
5
9

2
3) Find the value of x, if a + 7 = 0; b + 10 = 0 and 12x = ax b.
i) {5, 4}

Volume 1 of 2

Universal Tutorials X ICSE Maths

57

58

Solving Quadratic Equations using the formula:


z

The roots of the quadratic equation ax2 + bx + c = 0, where a 0 can be obtained by using
the formula, x =

b b 2 4ac
.
2a

Proof:
ax2 + bx + c = 0
4a2x2 + 4abx + 4ac = 0
(2ax)2 + 2 2ax b + b2 b2 + 4ac = 0
(2ax + b)2 b2 + 4ac = 0
2ax + b = b 2 4ac
x=

(Given)
(Multiplying by 4a)
(2ax + b)2 = b2 4ac
2ax = b b 2 4ac

b b 2 4ac
2a

Discriminant:
z
z

For the quadratic equation ax2 + bx + c = 0, a 0, b2 4ac is called discriminant and is in


general, denoted by D.
Thus, discriminant D = b2 4ac

Nature of Roots:
z

The nature of the roots of a quadratic equation depends entirely on the value of its
discriminant b2 4ac.

Case I: If a, b and c are real numbers and a 0, then discriminant:


i) b2 4ac = 0 the roots are real and equal.
ii) b2 4ac > 0 the roots are real and unequal.
iii) b2 4ac < 0 the roots are imaginary (not real)
Case II: If a, b and c are rational numbers and a 0, then discriminant:
i) b2 4ac = 0 the roots are rational and equal.
ii) b2 4ac > 0 and b2 4ac is a perfect square,
the roots are rational and unequal.
2
iii) b 4ac > 0 and b2 4ac is not a perfect square
the roots are irrational and unequal.
2
iv) b 4ac < 0 the roots are imaginary.

SOLVED EXAMPLES 7.2:


1) Solve each of the following equations by using the formula,
i) 5x2 2x 3 = 0.
Sol: Comparing 5x2 2x 3 = 0 with ax2 + bx + c = 0, we get: a = 5, b = 2 and c = 3

and so, x =

2 ( 2)2 4 5 3
25

x=

b b 2 4ac
2a

2 64
28
2+8
28
3
=
=
and
= 1 and
10
10
10
10
5
ii) x2 = 18x 77.
Sol: x2 = 18x 77 = 0 x2 18x + 77 = 0
=

58

Universal Tutorials X ICSE Maths

Volume 1 of 2

Chapter 07: Quadratic Equations

59

Comparing with ax + bx + c = 0, we get: a = 1, b = 18 and c = 77


x=

18 ( 18)2 4 1 77
2 1

=
iii)

Sol:

x=

b b 2 4ac
2a

18 16
18 + 4
18 4
=
and
= 11 and 7
2
2
2

3 x + 11x + 6 3 = 0

3 x2 + 11x + 6 3 = 0 a =

x=

3 , b = 11 and c = 6 3

11 (11)2 4 3 6 3

x=

2 3

11 49

2 3
2
3

3
3

11 + 7
2 3

and

9
3

and

3
3

11 7
2 3

b b 2 4ac
2a

4
2 3

and

18
2 3

(Rationalizing the denominators)

2 3
and 3 3
3
2) Without solving, examine the nature of the roots of the equations,
i) 5x2 6x + 7 = 0
Sol: For equation 5x2 6x + 7 = 0; a = 5, b = 6 and c = 7
=

Discriminant = b2 4ac = (6)2 4 5 7 = 36 140 = 104 (Which is negative)


The roots are not real i.e. the roots are imaginary.
ii) x2 + 6x + 9 = 0
Sol: For equation x2 + 6x + 9 = 0;
b2 4ac = (6)2 4 1 9 = 0
iii) 2x2 + 6x + 3 = 0
Sol: For equation 2x2 + 6x + 3 = 0

The roots are real and equal

b2 4ac = (6)2 4 2 3 = 36 24 = 12; which is positive

The roots are irrational and unequal.


iv) 3x2 5x + 2 = 0
Sol: For equation 3x2 5x + 2 = 0
b2 4ac = (5)2 4 3 2 = 25 24 = 1; which is a perfect square

The roots are rational and unequal.


3) Solve each of the following equations for x and give, in each case, your answer correct to
2 decimal places,
i) x2 10x + 6 = 0
Sol: x2 10x + 6 = 0
a = 1, b = 10 and c = 6
2
2
b 4ac = (10) 4 1 6 = 0 = 100 24 = 76

b 2 4ac =

x=

Volume 1 of 2

76 = 8.718

b b 2 4ac
10 8.718
[ x =
]
2a
2 1
= 9.359 and 0.641 = 9.36 and 0.64
Universal Tutorials X ICSE Maths

59

60

ii) 3x2 + 5x 9 = 0
Sol: a = 3, b = 5 and c = 9

b 2 4ac =

b2 4ac = (5)2 4 3 9 = 25 + 108 = 133

133 = 11.533

b b 2 4ac
5 11.533
Qx =
2a
23
5 + 11.533
5 11.533
and
= 1.089 and 2.756
=
6
6
= 1.09 and 2.76
Correct to 2 decimal places
4) Find the value of m, if the roots of the following quadratic equation are equal:
(4 + m)x2 + (m + 1)x + 1 = 0
Sol: For the given equation (4 + m)x2 + (m + 1) x + 1 = 0;
a = 4 + m, b = m + 1 and c = 1
Since, the roots are equal
b2 4ac = 0 (m + 1)2 4(4 + m) 1 = 0
m2 + 2m + 1 16 4m = 0
m2 2m 15 = 0
On solving, we get: m = 5 or m = 3
x=

UNSOLVED EXERCISE 7.2:


CW Exercise:
1) Without solving, comment upon the nature of roots of each of the following equations:

i) 7x2 9x + 2 = 0

ii) x2 + 2 3 x 9 = 0

iii) x2 ax b2 = 0

2) Solve each of the following equations using the formula:


2
1
1
i) x2 6x = 27
ii) 3x2 + 2x 1 = 0
iii) x = x 2
iv) 6 x 2 4 x 2 6 = 0
3
6
3
2x
2x 5
4
5
1
+
=8
vi)
v) 3 =
x 4 x 3
x
2x + 3
3
3) Find the value of p, if the quadratic equation has equal roots: 4x2 (p 2)x + 1 = 0
HW Exercise:
1) Without solving, comment upon the nature of roots of each of the following equations:
ii) 25x2 10x + 1 = 0 iii) 2x2 + 8x + 9 = 0
i) 6x2 13x + 4 = 0
2) Solve each of the following equations using the formula:
ii) x2 + 6x 10 = 0
iii) x2 + 2x 6 = 0
iv) 2x2 + 7x + 5 = 0
i) x2 10x + 21 = 0
1 2 5 2
2x + 3 x + 4
x 1 x 3
1
=
x + = x
vi) x2 6 = 2 2 x
vii)
viii)
v)
+
=3
x +3
x+2
15
3 3
x2 x4
3
3) The equation 3x2 12x + (n 5) = 0 has equal roots. Find the value of n.
4) Find the value of m, if the equation has equal roots: (m 2)x2 (5 + m) x + 16 = 0

Equations Reducible to Quadratic Equation:


Type I:
z
z

60

(ax4 + bx2 + c = 0)
2x4 5x2 + 3 = 0
Let x2 = y
Universal Tutorials X ICSE Maths

Volume 1 of 2

Chapter 07: Quadratic Equations


4

61

2x 5x + 3 = 0 2y 5y + 3 = 0
(On factorizing)
(y 1) (2y 3) = 0
3
y = 1 or y =
2
When y = 1 x2 = 1 x = 1 and when y =

3
3
3
x2 =
x=
2
2
2

3
3
,
2
2
2
2
2
Type ii): (x + 3x) (x + 3x) 6 = 0
Let x2 + 3x = y
(x2 + 3x)2 (x2 + 3x) 6 = 0 y2 y 6 = 0
(y 3) (y + 2) = 0
(On factorizing)
y = 3 or y = 2
y = 3 x2 + 3x = 3 x2 + 3x 3 = 0

Required solution = 1, 1,

3 ( 3 ) 2 4 1 3
3 21
x=
2
2 1
2
2
And y = 2 x + 3x = 2 x + 3x + 2 = 0

x=

3 (3)2 4 1 2
3 1
3 + 1
3 1

=
and
= 1 and 2
2 1
2
2
2

x=

Required solution is:


z

Type iii):

Let

x
1 x
1
+
= 2 , x 0 and x 1
1 x
x
6

x
=y
1 x

1
1 x
=
y
x

1
13
=
y
6

6y2 + 6 = 13y

(2y 3) (3y 2) = 0
3
2
y=
or y =
2
3

(On factorizing)

y+

When y =
And y =

6y2 13y + 6 = 0

x
3
x
9
9
=

=
4x = 9 9x x =
1 x
2
1 x
4
13
x
2
x
4
4
=

=
9x = 4 4x x =
1 x
3
1 x
9
13

Required solution is:


z

3 + 21 3 21
,
, 1, 2
2
2

9
4
,
13 13

1
1

Type iv): 9 x 2 + 2 9 x + 52 = 0
x
x

Let x +

Volume 1 of 2

1
=y
x
Universal Tutorials X ICSE Maths

61

62

1
1
+ 2 = y2 x 2 + 2 = y2 2
x2
x
Substituting we get, 9(y2 2) 9y 52 = 0
9y2 30y + 21y 70 = 0
9y2 9y 70 = 0
3y(3y 10) + 7(3y 10) = 0
(3y 10) (3y + 7) = 0

Squaring we get, x 2 +

3y 10 = 0

or

3y + 7 = 0

10
3

y=

or

7
3

10
3
3x2 + 3 = 10x

10
1
=
x
3
3x2 10x + 3 = 0

3x(x 3) 1(x 3) = 0

(3x 1) (x 3) = 0

x+

Now, y =

And y =

7
3

3x2 + 7x + 3 = 0

x+

7
1
=
x
3

x=

7 49 36
6

Hence the roots of the given equation are 3,

x 2 + 1 10
=
x
3
3 x 2 9x x + 3 = 0
1
x=
or 3
3
x2 + 1
7
=
x
3
x=

7 13
6

1 7 + 13 7 13
,
,
6
6
3

1
1
1

Note: When the equation is of the form a x 2 + 2 + b x + c = 0 take x


= y and
x
x
x

1
2
x 2 + 2 = y + 2, proceed in the above manner.
x

UNSOLVED EXERCISE 7.3:


CW Exercise:
1) Solve each of the following equation for x and give, in each case, your answer correct to one
decimal place:
ii) 5x2 + 10x 3 = 0
i) x2 8x + 5 = 0
2) Solve each of the following equation for x and give, in each case, your answer correct to two
decimal place:
ii) x2 3x 9 = 0
i) 2x2 10x + 5 = 0
3) Solve each of the following equation for x and give, in each case, your answer correct to three
decimal place, 3x2 12x 1 = 0
4) Solve, x4 2x2 3 = 0
5) Solve, (x2 5x)2 7(x2 5x) + 6 = 0

6) Solve, (i)

62

x
+
x 3

x 3 5
=
x
2

3x + 1 x + 1 5
ii)
+
=
x + 1 3 x + 1 2

7) Solve:

3x + 9
2x
1
3
+
+
= 0; x 3, x
( x 3)(2x + 3)
x 3
2x + 3
2

8) 2(x2 +

1
1
) (x +
) = 11
x2
x

Universal Tutorials X ICSE Maths

Volume 1 of 2

Chapter 07: Quadratic Equations

63

x
x
9) Solve:
+ 12 = 0; x 2
7
x + 2
x +2
10) Find the value of m so that equation has equal roots. Also, find the solution.
(m 3)x2 4x + 1 = 0
HW Exercise:
1) Solve each of the following equation for x and give, in each case, your answer correct to one
decimal place,
ii) 3x2 + 5x 9 = 0
i) 2x2 7x 2 = 0
2) Solve each of the following equation for x and give, in each case, your answer correct to two
decimal place:
i) 4x2 + 13x + 6 = 0
ii) x2 + 3x 9 = 0
3) Solve each of the following equation for x and give, in each case, your answer correct to three
decimal place:
ii) 2x2 + 11x + 4 = 0
i) x2 16x + 6 = 0
4
2
4) Solve, i) x 10x + 9 = 0
ii) x4 = 25x2 144
2
2
2
ii) (x2 3x)2 16(x2 3x) 36 = 0
5) Solve, i) (x x) + 5(x x) + 4 = 0
2x 3 x 1
6) Solve, i)
=3
4
x 1 2x 3
7) Solve: x +

ii) 3

x
5
+3
= 10
5
x

4
= -4; x 0
x

8) Solve: (x2 +

1
1
) 3 (x ) 2 = 0
2
x
x

1 1 1
1
1 1 1

1
+ + =
= 0
take + +
p q x x + p+q
p q x x +p+ q

10) Find the value of m so that the equation has equal roots. Also, find the solution.
x2 (m + 2) x + (m + 5) = 0

9) Solve:

PREVIOUS BOARD QUESTION:


1) Without solving the quadratic equation, find the value of m for which the given equation has real
[2012]
and equal roots. x2 + 2(m 1)x + (m + 5) = 0.
2) Solve the equation and give your answer correct to 3 significant figures: 5x2 3x 4 = 0. [2012]
18
3) Solve the equation x
= 6. Give your answer correct to two significant figures.
[2011]
x
4) Without solving the following quadratic equation, find the value of p for which the roots are
[2010]
equal Px2 4x + 3 = 0
5) Solve the following quadratic equation and give the answer correct to two significant figures
[2009]
4x2 7x + 2 = 0
6) Solve the quadratic equation 5x(x + 2) = 3 for x and give your answer correct to two decimal
places.
[2008]
7) Solve the quadratic equation x2 3x 9 = 0 for x and give your answer correct to two decimal
places.
[2007]
1
= 7. Write your answer correct to two decimal places.
[2006]
8) Solve the equation 2x
x
9) Solve the equation x2 5x 10 = 0 and give your answer up to two decimal places.
[2005]
[2004]
10) Solve the equation 3x2 x 7 = 0 and give your answer correct to two decimal places
Volume 1 of 2

Universal Tutorials X ICSE Maths

63

64

MISCELLANEOUS EXERCISE:
Solve each of the following equations:
1) a2x2 b2 = 0

2) x4 2x2 3 = 0

1
1

3) 2 x 2 + 2 3 x = 4
x
x

2
2
5) Solve, abx + (b ac)x bc = 0

6) 2y2 + y + 1 = 0

7) 5y2 2y 2 = 0

9) 4x2 + 12x + 9 = 0

11)

4) (3x2 5x + 2) (3x2 5x 2) = 21

8) z2 6z + 4 = 0

7 y2 6y 13 7 = 0

10) z2 +

1
z3=0
2

12) (x + 4) (x + 5) = 3(x + 1) (x + 2) + 2x

13) (2x + 3)2 = 81

14) x2

11
15
x+
= 0 15) 2x4 5x2 + 3 = 0
4
8

16) (x2 + 5x + 4) (x2 + 5x + 6) = 120

17)

4 x 2 19 = 9

18) Solve each of the following equation for x and give, in each case, your answer correct to two
decimal place, x2 5x 10 = 0
1
1
1
19)

=
20) 3y2 + 9y + 4 = 0
21) z2 + 2z 8 = 0
22) 6x2 + x 2 = 0
x 3
x +5
6
23) 7z2 + 8z + 2 = 0

24) 3x2+ 2 5 x 5 =0

1
2
3
4

x +1 x + 2 x + 4 x + 4
27) Find the value of m so that the equation has equal roots. Also, find the solution:
3x2 + 12x + (m + 7) = 0

25) 3y2 + (6 + 4a)y + 8a = 0

26)

ANSWER TO THE UNSOLVED EXERCISE:


CW Exercise 7.1:
1) i) 12, 2
5

ii) (4, 4)

iii) (0, 1/4)

iv) (8, 3)

v) ( /2, 2)

vi) (5, 2)

vii) 13

viii) (a, b)

ix) (5/8, 5/2)

x) (9, 9)

xi) (5, 5)

xii) (3, 2)
1 19
4) ,
2
8

2) (i) x2 8x + 15 = 0, (ii) x2 x 6 = 0

3) (3, 2)

HW Exercise 7.1:
1) i) (2, 3)
v) (2, 4)
ix) (12, 2)
2

ii) (1, 1/4)

iii) (2, )

iv) (0, 3/2)

vi) (1, 102/3)

vii) 20, 25

viii) (2, 7/2)

x) (, 4)
2

2) (i) x x 20 = 0 (ii) 5x + 17x + 6 = 0 (iii) 9x2 + 8x = 0

3) (2/3, 5/4)

CW Exercise 7.2:
1) (i) Rational and unequal (ii and iii) Irrational and unequal
2) (i) (9, 3) (ii) ( 1, 1/3) (iii) ( 2, 2 ) (iv)

( 6, ) (v) ( 2, 1) (vi) (6, 3


6
3

1
13

3) P = 6 or 2
64

Universal Tutorials X ICSE Maths

Volume 1 of 2

Chapter 07: Quadratic Equations

65

HW Exercise 7.2:
1) (i) irrational and unequal (ii) real and equal (iii) imaginary
2) (i) 3, 7 (ii) 3, 19 (iii) 1, 7 (iv)( 1, 5/2) (v) (5, 5) (vi) (3 2 , 2 ) (vii) 6 (viii) 5, 5/2
3) n = 17

4) m = 51 or 3

CW Exercise 7.3:
1) (i) 7.3,0.7 (ii) 0.3, 2.3 2) (i) (4.46, 0.56) (ii) 4.85 and 1.85
4) 3
8) 2,

5) 6, 1,

5 29
2

1 3 5
,
= 2.62 or 0.38
2
2

6) (i) 4, 1 (ii) 1, -1/5


9) 3 or 2

HW Exercise 7.3:
1) (i) 3.8, 0.3 (ii) 1.1, 2.8
3) (i) 15.616, 0.384 (ii) 0.381, 5.108
5) No real
6) , 4/3 (ii) 5/9, 45

3 13
= 3.30 or 0.30
2
10) m = 4, for m = 4 sol 3, for m = 4 sol = 1
8) 1, 1,

2
3

3) 4.082 and 0.082


7) 1
10) m = 7; solution =

1
2

2) (i) 0.56, 2.69 (ii) 4.85, 1.85


4) (i) 3, 1 (ii) 4, 3
7) 2
9) p or q

Previous Board Question:


1) m = 4 or m = 1

2) 0.643, 1.243

3) 8.2 or 2.2

5) 1.4 or 0.36
9) x = 6.53 or x=1.53

6) 2.26
7) 4.85 and 1.85
10) x = 1.703 or 1.37

4
3
8) 3.64 or 0.14

4) p =

Miscellaneous:
1)

b
b
,
a
2

2) 2)

3 , 3

3) 2 ,

1
,1
2

5)

c b
,
b a

6) 1,

1
2

8) 3

10) 3

3
2

11) 7 ,

13

5 3
,
4 2

15) 1,

18) 6.53, 1.53

19) (9, 7)

1 2
22) ,

2 3

23)

14)

26) x = 0,

Volume 1 of 2

5
2

12)

3 29
2

3
= 1.22 16) 6, 1
2

4 2
7

20)

3 33
2

24)

5
, 5
3

5 85
=2.37or0.70
6
3
9)
2
4)

13) 3, 6
17) 5, 5
21) (2, 4)

4a

,2
25)
3

27) m = 5 and soln is 2

Universal Tutorials X ICSE Maths

65

66

Chapter 08: Solving Problems (Based on Quadratic Equations)


Introduction:
For solving a word problem based on Quadratic Equation adopt the following steps:
Represent the unknown quantity of the problem by a variable such as x or y.
Translate the given statement to form an equation in terms of the variable x or y
Solve the equation.
Measure of length, area, volume, weight, capacity etc. are always taken as positive.

Problems based on Numbers:


If the sum of two numbers is 7 then the numbers are x and 7 x.
If the difference between two numbers is 7 then the smaller number is x and larger is 7 + x.
If the product of two numbers is 7 then the numbers are x and

7
.
x

SOLVED EXAMPLES 8.1:


1) Find two natural numbers which differ by 3 and the sum of whose squares is 117.
Sol: Let the natural numbers be x and x + 3.
x2 + (x + 3)2 = 117 x2 + x2 + 6x + 9 = 117 2x2 + 6x 108 = 0
x2 + 3x 54 = 0
(Dividing each term by 2)
(x + 6) (x 6) = 0
(Factorising)
x + 9 = 0 or x 6 = 0
x = 9 or x = 6
One number = 6
(Since, 9 is not a natural number)
And other number = 6 + 3 = 9
Numbers are 6 and 9
2) Five times a certain whole number is equal to three less than twice the square of the
number. Find the number.
Sol: Let the number be x
Given, five times the number = 3 less than twice the square of the number.
5x = 2x2 3
2x2 5x 3 = 0
(x 3) (2x + 1) = 0
(Factorising)
1
x 3 = 0 or 2x + 1 = 0
x = 3 or x =
2
Required whole number is 3.
8
3) Divide 8 into two parts such that the sum of their reciprocals is
.
15
Sol: Let the two parts be x and 8 x.
8x + x
1
1
8
8

+
=

=
120 = 8(8x x2)
x (8 x )
x
8x
15
15
x = 5 or x = 3
(On solving)
x2 8x + 15 = 0
x = 5 one part = 5 and other part = 8 5 = 3 or x = 3 one part = 3 and other part = 8 3 = 5
Required pars are 3 and 5.
66

Universal Tutorials X ICSE Maths

Volume 1 of 2

Chapter 08: Problems based on Quadratic Equations

67

Problems based on Time and Work:


4) For the same amount of work, A takes 6 hours less than B. If together they complete the
work in 13 hours 20 minutes; find how much time will B alone take to complete the work.
Sol: If B alone takes x hours then A alone takes (x 6) hours for the same work.

1
1
3
+ =
x 6 x 40

20
40

13 hrs. 20 min. = 13 +
hrs.
hrs. =
60
3

x + x 6
3
=
(x 6)x 40

3x2 18x = 80x 240

3x2 98x + 240 = 0 3x2 90x 8x + 240 = 0


8
(x 30) (3x 8) = 0 x = 30 or x =
3
B alone will take 30 hrs. to complete the work.
x = 30

UNSOLVED EXERCISE 8.1:


CW Exercise:
1) The product of two consecutive integers is 56. Find the integers.
2) The sum of the square of two consecutive natural numbers is 41. Find the numbers.
3) The sum of a number and its reciprocal is 4.25. Find the number.
4) The square of a number added to onefifth of it, is equal to 26. Find the number.
5) The sum of the squares of two consecutive positive even numbers is 52. Find the numbers.
1 1 1
6) Three positive numbers are in the ratio : : . Find the numbers; if the sum of their squares
2 3 4
is 244.
7) Three consecutive natural numbers are such that the square of the middle number exceeds the
difference of the squares of the other two by 60.
Assume the middle number to be x and form a quadratic equation satisfying the above
statement. Hence; find the three numbers.
8) Out of three consecutive positive integers, the middle number is p. If three times the square of
the largest is greater than the sum of the squares of the other two numbers by 67; calculate the
value of p.
9) A can do a piece of work in x days and B can do the same work in (x + 16) days. If both working
together can do it in 15 days; calculate x.
10) One pipe can fill a cistern in 3 hours less than the other. The two pipes together can fill the
cistern in 6 hours 40 minutes. Find the time that each pipe will take to fill the cistern.
HW Exercise:
1) Find the two natural numbers which differ by 5 and the sum of whose square is 97.
7
2) Two natural numbers differ by 3. Find the numbers, if the sum of their reciprocals is
.
10
3
3) Divide 15 into two parts such that the sum of their reciprocals is
.
10
4) Find two consecutive positive odd numbers, the sum of whose squares is 74.
5) Two numbers are in the ratio 3:5. Find the numbers if the difference between their square is 144.
6) Divide 20 into two parts such that 3 times the square of one part exceeds the other part by 10.
7) A takes 6 days less than the time taken by B to finish a piece of work. If both A and B together
can finish it in 4 days, find the time taken by B to finish the work.
Volume 1 of 2

Universal Tutorials X ICSE Maths

67

68

Problems based on Geometrical Figures:


SOLVED EXAMPLES 8.2:
1) The hypotenuse of a right triangle is 13 cm and the difference between the other two
sides is 7 cm. Taking x as the length of the shorter of the two sides, write an equation in
x that represents the above statement and also solve the equation to find the two
unknown sides of the triangle.
Sol: Since, the shorter side = x cm
Longer side = (x + 7) cm
13
x
Using Pythagoras Theorem, we get, x2 + (x + 7)2 = 132
x2 + x2 + 14x + 49 = 169
2x2 + 14x 120 = 0
x+7
[Dividing each term by 2]
x2 + 7x 60 = 0

On solving, it gives, x = 12 or x = 5
Since, the side of triangle cannot be negative, therefore, x = 5
One side of the triangle = x = 5 cm and other side of the triangle = (x + 7) = (5 + 7) = 12 cm
2) The length of a verandah is 3 m more than its breadth. The numerical value of its area is
equal to the numerical value of its perimeter.
i) Taking x as the breadth of the verandah, write an equation in x that represents the
above statement.
ii) Solve the equation obtained in (i) above and hence find the dimensions of the
verandah.
Sol: Since, breadth = x m
Length = (x + 3) m
i) Given: Area of verandah = its perimeter
[Numerically]
i.e. length breadth = 2 (length + breadth)
(x + 3) x = 2(x + 3 + x) x2 + 3x = 4x + 6
x2 x 6 = 0
ii) x2 x 6 = 0
(x 3) (x + 2) = 0
[On factorizing]
x = 3 or x = 2
Since, breadth cannot be negative,
x=3
Hence, length of verandah = (x + 3) = (3 + 3) = 6m and its breadth = x = 3 m

UNSOLVED EXERCISE 8.2:


CW Exercise:
1) The sides of a rightangled triangle containing the right angle are 4x cm and (2x 1) cm. If the
area of the triangle is 30 cm2, calculate the length of its sides.
2) The hypotenuse of a rightangled triangle is 26 cm and the sum of other two sides is 34 cm.
Find the length of its sides.
3) The length of a rectangular plot exceeds its breadth by 12 m and its area is 1260 sq.m. Find its
length and breadth.
4) A footpath of uniform width runs round the inside of a rectangular field 32 m long and 24 m wide.
If the path occupies 208 m2, find the width of the footpath.
5) An area is paved with square tiles of a certain size and the number required is 600. If the tiles
had been 1 cm smaller each way, 864 tiles would have been needed to pave the same area.
Find the size of the larger tiles.
6) A farmer has 70 m of fencing, with which he encloses three sides of a rectangular sheep pen;
the fourth side being a wall. If the area of the pen is 600 sq. m, find the length of its shorter side.
7) The area of a big rectangular room is 300 m2. If the length were decreased by 5 m and the
breadth increased by 5 m; the area would be unaltered. Find the length of the room.
68

Universal Tutorials X ICSE Maths

Volume 1 of 2

Chapter 08: Problems based on Quadratic Equations

69

HW Exercise:
1) The sides of a rightangled triangle are (x 1), 3x and (3x + 1) cm. Find:
ii) the length of its sides
iii) its area
i) the value of x
2) The hypotenuse of a rightangled triangle p exceeds one side by 1 cm and the other side by 18
cm; find the length of the sides of the triangle.
3) The perimeter of a rectangle is 104 m and its area is 640 m2. Find its length and breadth.
4) Two squares have sides x cm and (x + 4) cm. The sum of their areas is 656 sq. cm. Express this
as an algebraic equation in x and solve the equation to find the sides of the squares.
5) The length of a rectangular board exceeds its breadth by 8 cm. If the length was decreased by 4
cm and the breadth doubled, the area of the board would be increased by 256 sq.cm. Find the
length of the board.
7
that of
6) A square lawn is bounded on three sides by a path 4 m wide. If the area of the path is
8
the lawn, find the dimensions of the lawn.

Problems based on Distance Speed and Time:


z

Speed =

Distance
Distance
, Time =
and distance = Speed Time
Speed
Time

SOLVED EXAMPLES 8.3:


1) A man travels 200 km with a uniform speed of x km/hr. The distance could have been
covered in 2 hrs. less, had the speed been (x + 5) km/hr. Calculate the value of x.
Distance
200
Sol: When speed is x km/hr, time =
hrs.
Since, Time =
Speed
x

And when speed is (x + 5) km/hr, time =

200
hrs.
x+5

200
200

=2
x
x +5
On solving, it gives x = 20 or x = 25
Since, speed cannot be negative, x = 20 km/hr
2) Car A travels x km for every litre of petrol, while car B travels (x + 5) km for every litre of
petrol.
i) Write down the number of litres of petrol used by car A and car B in covering a
distance of 400 km.
ii) If car A uses 4 litres of petrol more than car B in covering the 400 km, write down an
equation in x and solve it to determine the number of litres of petrol used by car B for
the journey.
400
400
Sol: i) No. of litres of petrol used by car A =
litre and by car B =
litre.
x
x +5
400 x + 2000 400 x
400
400
ii) Given:

=4

=4
x (x + 5 )
x
x +5
x2 + 5x 500 = 0
4(x2 + 5x) = 2000
[On solving]
x = 25 or x = 20
[ It cannot be negative]
x = 20
400
400
No. of litres of petrol used by car B =
=
= 16 litres.
(x + 5) 20 + 5

According to the problem,

Volume 1 of 2

Universal Tutorials X ICSE Maths

69

70

Problems on C.P. and S.P.:


z

If loss, S.P. = C.P. Loss

Loss% =

Loss
100
C.P.

Pr ofit
100
C.P.
3) By selling an article for `24, a trader loses as much percent as the cost price of the
article. Calculate the cost price.
Sol: Let C.P. of the article = `x
z

If profit, S.P. = C.P. + Profit

Loss = x% of C.P. =

Profit% =

x2
x
x=`
100
100

x2
= 24
(C.P. Loss = S.P.)
100
x2 100x + 2400 = 0
100x x2 = 2400
On solving, we get: x = 60 and x = 40
C.P. of the article is `60 or `40
x

UNSOLVED EXERCISE 8.3:


CW Exercise:
1) The speed of an ordinary train is x km per hr. and that of an express train is (x + 25) km per hr.
i) Find the time taken by each train to cover 300 km.
ii) If the ordinary train takes 2 hrs. more than the express train; calculate speed of the express
train.
2) If the speed of a car is increased by 10 km per hr. it takes 18 minutes less to cover a distance of
36 km. Find the speed of the car.
3) A train covers a distance of 300 km between two stations at a speed of x km/h. Another train
covers the same distance at a speed of (x 5) km/h.
i) find the time which each train takes to cover the distance between the station.
ii) if the second train takes 3 hrs more then the first train, find the speed of each train.
4) A car made a run of 390 km in x hours. If the speed had been 4 km/hour more, it would have
taken 2 hours less for the journey. Find x.
5) A man bought an article for `x and sold it for `16. If his loss was x percent, find the cost price of
the article.
6) By selling a chair for `75, Mohan gained as much per as its cost. Calculate the cost of the chair.
7) The total cost price of a certain number of identical articles is `4,800. By selling the articles at
`100 each, a profit equal to the cost price of 15 articles is made. Find the number of articles
bought.
HW Exercise:
1) If the speed of an aero plane is reduced by 40 km per hr. it takes 20 minutes more to cover 1200
km. Find the speed of the aeroplane.
2) A girl goes to her friends house, which is at a distance of 12 km. She covers half of the distance
at a speed of x km/hr. and the remaining distance at a speed of (x + 2) km/hr. If she takes 2 hrs.
30 minutes to cover the whole distance; find x.
3) A goods train leaves a station at 6 p.m., followed by an express train which leaves at 8 p.m. and
travels 39 km/hour faster than the goods train. The express train arrives at a station, 180 km
away, 36 minutes before the goods train. Assuming that the speeds of both the trains remain
constant between the two stations; calculate their speeds.
70

Universal Tutorials X ICSE Maths

Volume 1 of 2

Chapter 08: Problems based on Quadratic Equations

71

4) A trader bought an article for `x and sold it for `52, thereby making a profit of (x 10) per cent
on his outlay. Calculate the cost price.
z Two digit number = 10 Tens place + Unit place.
z Speed of boat on up stream = Speed of boat in still water Speed of the stream
z Speed of boat in down stream = Speed of boat in still water + Speed of the stream.

SOLVED EXAMPLES 8.4:


1) The sum S of first n natural numbers is given by the relation: S =

1
n (n + 1). Find n, if the
2

sum is 276.

Sol: Given: S = 276

2)

Sol:

3)

Sol:

4)

Sol:

1
n(n + 1) = 276
2

n2 + n 552 = 0

(On factorizing)
(n + 24) (n 23)
n = 24 or n = 23
(Zero product rule)
Since n is a natural number, reject n = 24
n = 23
A two digit number is such that the product of its digits is 12. When 36 is added to this
number; the digits interchange their places. Find the number.
Let the required two digit number be 10x + y
Given: xy = 12 and 10x + y + 36 = 10y + x
10x + y + 36 = 10y + x 9y = 9x + 36
i.e. y = x + 4
Now, xy = 12 x(x + 4) = 12
x2 + 4x 12 = 0
x = 6 or x = 2
(On solving)
Taking x = 2, we get, y = x + 4 = 2 + 4 = 6
The required two digit number = 10x + y = 10 2 + 6 = 26
The product of Ramus age (in years) five years ago and his age (in years) nine years later
is 15. Determine Ramus present age.
Let Ramus present age = x years
His age 5 years ago = (x 5) years and 9 years later = (x + 9) years
Given: (x 5) (x + 9) = 15 x2 + 4x 60 = 0
x = 10 or x = 6 (On solving)
Ramus present age = 6 years
A motorboat, whose speed is 9 km/h in still water, goes 12 km downstream and comes
back in a total time of 3 hours. Find the speed of the stream.
Let the speed of the stream = x km/h
Speed of boat downstream = (9 + x) km/hr and speed of boat upstream = (9 x) km/hr
Also, time taken to go 12 km downstream =
And, time taken to come back =
Given:

12
12
+
=3
9+x
9x

12
hrs.
9+x

Time =

Distance
Speed

12
hrs.
9x

108 12x + 108 + 12x


=3
(9 + x )(9 x )

3(81 x2) = 216


81 x2 = 72 and x2 = 9 i.e. x = 3
Hence, the speed of the stream = 3 km/hr
5) A piece of cloth costs `200. If the piece was 5 m longer and each metre of cloth costs `2
less; the cost of the piece would have remained unchanged. How long is the piece and
what is the original rate per metre?
Sol: Let the length of the piece be x metres
Volume 1 of 2

Universal Tutorials X ICSE Maths

71

72

200
x

Since, the cost of x metres cloth = `200

Cost of each metre of cloth = `

New length of cloth = (x + 5)m

New cost of each metre of cloth = `

Given:

200
200

=2
x
x +5

200
x +5

200 x + 1000 200 x


=2
x (x + 5 )

i.e. 2(x2 + 5x) = 1000 x2 + 5x = 500


i.e. x2 + 5x 500 = 0 x = 25 or x = 20
The length of the piece = x = 20 m
200
And, the original rate per meter = `
= `10
20

UNSOLVED EXERCISE 8.4:


CW Exercise:
1) The sum S of n successive odd numbers starting from 3 is given by the relation: S = n(n + 2).
Determine n, if the sum is 168.
2) The product of the digits of a two digit number is 24. If its units digit exceeds twice its tens digit
by 2; find the number.
3) The ages of two sisters are 11 years and 14 years. In how many years time will the product of
their ages be 304?
4) The speed of a boat in still water is 15 km/hr. It can go 30 km upstream and return downstream
to the original point in 4 hours 30 minutes, find the speed of the stream.
5) Mr.Mehra sends his servant to the market to buy oranges worth `15. The servant having eaten
three oranges on the way, Mr.Mehra pays 25 paise per orange more than the market price.
Taking x to be the number of oranges which Mr.Mehra receives, form a quadratic equation in x.
Hence, find the value of x.
6) `250 is divided equally among a certain number of children. If there were 25 children more, each
would have received 50 paise less. Find the number of children.
HW Exercise:
1) A stone is thrown vertically downwards and the formula d = 16t2 + 4t gives the distance, d
metres, that it falls in t seconds. How long does it take to fall 420 metres?
2) One year ago, a man was 8 times as old as his son. Now his age is equal to the square of his
sons age. Find their present ages.
3) An employer finds that if he increases the weekly wages of each worker by `3 and employs one
worker less, he reduces his weekly wage bill from `816 to `781. Taking the original weekly wage
of each worker as `x; obtain an equation in x and then solve it to find the weekly wages of each
worker.
4) A trader bought a number of articles for `1,200. Ten were damaged and he sold each of the
remaining articles at `2 more than what he paid for it, thus getting a profit of `60 on the whole
transaction. Taking the number of articles he bought as x, form an equation in x & solve it.

PREVIOUS BOARD QUESTION:


1) A car covers a distance of 400 km at a certain speed. Had the speed been 12 km/h more, the
time taken for the journey would have been 1 hour 40 minutes less. Find the original speed of
the car.
[2012]
2) `480 is divided equally among x children. If the number of children were 20 more then each
[2011]
would have got `12 less. Find x.

72

Universal Tutorials X ICSE Maths

Volume 1 of 2

Chapter 08: Problems based on Quadratic Equations

73

3) A positive number is divided into two parts such that the sum of the squares of the two parts is
208. The square of the larger part is 18 times the smaller part. Taking x as the smaller part of the
two parts, find the number
[2010]
4) The speed of an express train is x km/h and the speed of an ordinary train is 12 km/h less than
that of the express train. If the ordinary train takes one hour longer than the express train to
cover a distance of 240 km, find the speed of the express train.
[2009]
5) Some students planned a picnic. The budget for the food was `480. As eight of them failed to
join the party, the cost of the food for each member increased by `10. Find how many students
went for the picnic
[2008]
6) Five years ago, a womans age was the square of her sons age. Ten years later her age will be
twice that of her sons age. Find:
[2007]
i) The age of the son five years ago.
ii) The present age of the woman.
7) A shopkeeper buys a certain number of books for `720. if the cost per book was `5 less; the
number of books that could be bought for `720 would be 2 more. Taking the original cost of each
book to be `x, write an equation in x and solve it.
[2006]
8) By increasing the speed of a car by 10 km/hr, the time of journey for a distance of 72 km is
reduced by 36 minutes. Find the original speed of the car.
[2005]

MISCELLANEOUS EXERCISE:
1) The distance by road between two towns A and B is 216 km and by rail it is 208 km. A car
travels at a speed of x km/hr and the train travels at a speed which is 16 km/hr. faster than the
car. Calculate:
i) The time taken by the car to reach town B from A, in terms of x;
ii) The time taken by the train to reach town B from A, in terms of x;
iii) If the train takes 2hours less than the car, to reach town B, obtain an equation in x and solve it
iv) Hence, find the speed of the train.
2) A trader buys x articles for a total cost of `600
i) Write down the cost of one article in terms of x. If the cost per article were `5 more, the
number of article that can be bought for `600 would be four less.
ii) Write down the equation in x for the above situation and solve it for x.
3) A hotel bill for a number of people for overnight stay is `4,800/. If there were 4 people more the
bill each person had to pay would have reduced by `200/. Find the number of people staying
overnight.
4) An aeroplane travelled a distance of 400 km at an average speed of x km/hr. On the return
journey, the speed was increased by 40 km/hr. Write down an expression for the time taken for:
i) the onward journey
ii) the return journey
iii) If the return journey took 30 minutes less than the onward journey, write down an equation in
x and find its value.
5) `6,500 were divided equally among a certain number of persons. Had there been 15 persons
more, each would have got `30 less. Find the original number of persons.
6) The products of the digits of a two digit number are 24. The units digit exceeds twice the tens
digit by two. Find the number.
7) Two train leave a railway station at the same time. The first train travels due west and the
second train due north. The first train travels 5 km/hr faster than the second train. If after 2
hours, they are 50 km apart, find the average speed of each train.
8) The sum S of first n even natural numbers is given by the relation S = n(n + 1). Find n, if the sum
is 420.
9) The sum of the ages of a father and his son is 45 years. Five years ago, the product of their
ages (in years) was 124. Determine their present ages.

Volume 1 of 2

Universal Tutorials X ICSE Maths

73

74

10) In an auditorium, seats were arranged in rows and columns. The number of rows was equal to
the number of seats in each row. When the number of rows was doubled and the number of
seats in each row was reduced by 10, the total number of seats increased by 300. Find:
i) the number of rows in the original arrangement.
ii) the number of seats in the auditorium after rearrangement.
11) Divide 29 into 2 parts so that the sum of the squares of the parts is 425.
12) Find the whole no. which when decreased by 20 is equal to 69 times the reciprocal of the no.
13) There are 3 consecutive integers such that the square of the first increased by the product of the
other 2 gives 154. What are the integers?
14) A two digit number is four times the sum and three times the product of its digits. Find the
number.
15) The sides in cm of a right angled triangle are (x 1), x and (x + 1). Find the lengths of the sides
of the triangle.
16) A person on tour has `360 for his daily expenses. If he exceeds his tour programme by 4 days,
he must cut down his daily expenses by `3 per day. Find the number of days of his tour
programmed.
17) A shopkeeper buys a no. of books for `80/- If he had bought 4 more for the same amount, each
book would have cost Re 1 less. How many books should he buy?
18) A farmer prepares a rectangular vegetable garden of area 180 sq metres. With 39 metres of
barbed wire, he can fence the three sides of the garden, leaving one of the longer sides
unfenced. Find the dimensions of the garden.
19) An aeroplane takes 1 hour less for a journey of 1200km, if its speed is increased by 60km/h from
its usual speed. Find the usual speed of the plane.
20) A plane left 30 minutes later than the schedule time and in order to reach its destination 1500 km
away in time it has to increase its speed by 250 km/hr from its usual speed. Find its usual speed.
21) A journey of 192 km by a fast train takes 2 hours less than by a slow train. If the average speed
of the slow train be 16km/h less than that of the fast train, find the average speed of each train.
22) In a flight of 600km, an aircraft was slowed down due to bad weather. Its average speed for the
trip was reduced by 200km/h and the time of flight increased by 30 minutes. Find the duration of
flight.
23) 2 pipes running together can fill the cistern in 31/13 min. If one pipe takes 3 min. more than the
other to fill the cistern, find the time in which each pipe would fill the cistern
24) The speed of a boat in still water is 11km/h. It can go 12 km upstream and return downstream to
the original point in 2 hours 45 minutes. Find the speed of the stream.
25) The speed of a boat in still water is 8km/hr. It can go 15km upstream and 22km downstream in 5
hours. Find the speed of the stream.
26) The length of the hypotenuse of a right-angled triangle exceeds the length of the base by 2cm
and exceeds twice the length of the altitude by 1 cm. Find the length of each side of the triangle.
27) A train covers a distance of 600 km at x km/hour. Had the speed been x + 20 km/hour the time t
cover the distance would have been reduced by 5 hours. Write down an equation in x & solve it.
28) The total surface area of a hollow metal cylinder, open at both ends, of external radius 8cm and
height 10 cm is 338 cm2. Taking r to be the inner radius, write down an equation in r and use
it to state the thickness of the metal in the cylinder. [Hint 2Rh + 2rh + 2(R2 r2) where R is
the external radius]
29) The perimeter of a rectangular plot is 180cms and its area is 1800 m2 of the length is x ms,
express its breadth in terms x. Hence form an equation in x. Solve the equation and find the
length and breadth of the rectangle.
30) A rectangular garden 10ms by 16ms is to be surrounded by a concrete walk of uniform width.
Given that the area of the walk is 120m2. Assuming the width of the walk to be x, form an
equation in x and solve it to find the value of x.

74

Universal Tutorials X ICSE Maths

Volume 1 of 2

Chapter 08: Problems based on Quadratic Equations

75

31) An express train makes a run of 240km at a certain speed. Another train whose speed is 12
km/hour less takes an hour longer to make the same trip. Find the speed of the express train in
km/hour.
32) A train covers a distance of 90km at a uniform speed. Had the speed been 15 km/hour more it
would have taken 30 minutes less for the journey. Find the original speed of the train.
33) A rectangle of area 105 cm2 has its length equal to x cm. Write down its breadth in terms of x
given that its perimeter is 44cms, write down an equation in x and get the dimensions if the
rectangle.
34) The sum of two numbers is 9 and the sum of their squares is 41. Taking one number as x, form
an equation in x and solve it to find the numbers.
35) One number is x. Another number is five more than twice this number. If the product of the two
numbers is 25, find x.

ANSWER TO THE UNSOLVED EXERCISE:


CW Exercise 8.1:
4) 5,

9) x = 24

1
4
10) 12, 15

3) 5, 10

4) 5, 7

5) 9, 15, or 9, 15

4) 2 m

5) 6 cm

2) 5, 4

7) 9, 10, 11

8) p = 5

HW Exercise 8.1:
1) 4, 9
2) 2, 5

26
5

3) 4,

1) 7, 8 or 7, 8

CW Exercise 8.2:
1) 12, 5, 13
2) 10, 24
3) 42, 30
7) 20 when the longer side is 30 m
HW Exercise 8.2:
1) i) 8 ii) 7, 24, 25 iii) 84 sq.cm
CW Exercise 8.3:
300 300
,
1) i)
ii) 30 km/hr
x x + 25
4) 15
5) 20 or 80 6) 50

2) 25, 24, 7 3) 20, 32

2) 30 km/hr 3) (i)

6) 3, 17

7) 12

6) 15 longer side is 40 m

4) 16, 20

5) 24 cm

6) 16 m

7) 60

3) 36 km/hr 75 km/hr

4) `40

CW Exercise 8.4:
1) 12
2) 38

3) 5 years

4) 5 km/hr

5) 12

HW Exercise 8.4:
1) 5 sec
2) 49, 7

3) 68

4) 100

Volume 1 of 2

6) 12, 8, 6

300
300
hrs,
hrs (ii) 25km/hr 20 km/hr
x
x 5

HW Exercise 8.3:
1) 400 km/hr 2) 4

Previous Board Question:


1) 48 km/hr
2) 20
3) 20
6) Son 5 years; Woman 30 years.

5) 4, 6

6) 100

4) 60 km/hr 5) 16
7) x = 45
8) 30 km/hr

Universal Tutorials X ICSE Maths

75

76

Miscellaneous:
216
208
600
600 600
(ii)
(iii) 36 (iv) 52 km./hr 2) i)
(ii)

= 5, x = 24 3) 8
1) i)
x
x + 16
x
x4
x
400
400
4) i)
(ii)
(iii) x = 160
5) 50
6) 38
7) 20 km/hr, 15 km/hr
x
x + 40
8) 20
9) 36, 9
10) i) 30 (ii) 1200
11) 16, 13 12) 23
19 17
15
13) 8,9,10 (or)
,
,
14) 24
15) 3, 4, 5 16) 20
17) 16
2
2
2
18) b = 12 or 15/2; l = 15 or 24
19) 60 km/hr.
20) 750km/hr
21) 32km/hr
22) 1 hr
23) faster 5 min, slower 8 min
24) 5 km/hr 25) 3 km/hr
27) 40km/hr
28) r2 10r + 25 = 0; 3cm
29) 60m, 30m
30) 2m
31) 60 km/hr
32) 45km/hr
33) 15cms, 7cms
34) 4, 5
35) (5/2, 5)

76

Universal Tutorials X ICSE Maths

Volume 1 of 2

Chapter 09: Coordinate Geometry: Reflection

77

Chapter 9: Co-ordinate Geometry: Reflection


Introduction:
Co-ordinate Geometry:
z

It is the branch of geometry in which two numbers, called coordinates, are used to locate
the position of a point in a plane.

Coordinate Axes:

The two mutually perpendicular number lines intersecting


each other at their zeroes, are called rectangular axes or
coordinate axes of reference.
As shown in the adjacent figure, the horizontal number
line XOX is called the xaxis; the vertical number line
YOY is called the yaxis and their point of intersection,
O, is called the origin.

4
yaxis

Y
5
3
2
1
X

X
5 4 3 2 1

3 4 5
xaxis

2
3
4
5
Y

Co-ordinates:
z

The position of a point in a plane is expressed by a pair of two numbers (one concerning x
axis and the other concerning yaxis) called coordinates.
Y

Consider a point P (x, y)

A(3, 4)

Here (x, y) is a pair of two numbers, which gives the


abscissa
P(x, y)
3
x
coordinates of point P.
2
y
The first number x of the pair (x, y) is the distance of
1
the point P from yaxis and is called xcoordinate or X
X
5 4 3 2 1
1 2 3 4 5
abscissa.
1
2
The second number y of the pair (x, y) is the distance
3
of the point P from xaxis and is called the yco
B(2, 3)
4
ordinate or ordinate.
5
Suppose the coordinates of point A are (3, 4), then
Y
its abscissa = 3 and ordinate = 4.
And, if for a point B, abscissa = 2 and ordinate = 3, then its coordinates are (2, 3).
ordinate

Note:
1)

In stating the coordinates of a point the abscissa precedes the ordinate. The two coordinates
are separated by a comma and are enclosed in a bracket.
Thus, a point with abscissa x and ordinate y is denoted by (x, y).

2)

Coordinates of origin O = (0, 0).

3)

Coordinates of a point on the xaxis = (x, 0) and

4)

Coordinates of a point on the yaxis = (0, y)

Volume 1 of 2

Universal Tutorials X ICSE Maths

77

78

Reflection:
z
z

When an object is placed before a plane mirror, the image formed is


at the same distance behind the mirror as the object is in front of it.
There, to find the image of a point P in a line AB, consider AB as the
plane mirror and point P as the object. Now, find a point P on the
other side of AB, such that P is at the same distance from AB as P is
from it.

P(x, y)
B
A

Thus, point P is the image of point P in line AB and line AB, which is
also the perpendicular bisector of PP, is said to be the mirror line or
mediator of segment PP.

The transformation which maps a point P to P is called reflection.


The reflection can be denoted in several ways, but here it will be denoted by Ml, where M
denotes reflection and l is the line or point in which the reflection takes place.
Thus, Mx represents reflection in the xaxis;
My represents reflection in the yaxis;
and Mo represents reflection in the origin.

P(x, y)

Reflection in xaxis (Mx):


z
z
z

Let P(x, y) be any point in the coordinate plane then, the point
P(x, y) is the reflection of the point P in the xaxis.
If a point is reflected in the xaxis, its abscissa (x) remains the
same but the sign of ordinate (y) is changed.
Hence, Mx (x, y) = (x, y). E.g. Mx (2, 3) = (2, 3).

Y
P(x, y)
X

P(x, y)
Y

Reflection in yaxis (My):


z
z
z
z

Let P(x, y) be any point in the coordinate plane then, the point
P(x, y) is the reflection of the point P in the yaxis.
If a point is reflected in the yaxis, then the sign of its abscissa (x) is
changed but its ordinate (y) remains the same.
Hence, My (x, y) = (x, y)
E.g. My (5, 2) = (5, 2)

P(x, y)

P(x, y)
X

Reflection in the Origin (M0):


z
z
z

Let P(x, y) be any point in the coordinate plane then, the point
P(x, y) is the reflection of the point P in the origin.
If a point is reflected in the origin, then the sign of both abscissa (x) and
ordinate (y) is changed.
Hence, M0 (x, y) = (x, y). E.g. M0 (7, 3) = (7, 3)

P(x, y)
0

P(x,y)
y

Note: The combination of the reflections is always commutative, i.e.

i) Mx . My = My . Mx = Mo

ii) Mo . Mx = Mx . Mo = My

iii) Mo . My = My . Mo = Mx and so on

Properties of reflection:
z
z
z

78

If three points A, B and C is clockwise, then their image A, B and C are anticlockwise.
The image of a figure under reflection is congruent to the original figure.
Reflection in the xaxis, followed by the reflection in the yaxis is equivalent to reflection in
the origin.
Universal Tutorials X ICSE Maths

Volume 1 of 2

Chapter 09:
0 Coordina
ate Geometry: Reflection

79

SOLVED
D EXAMPL
LES 9.1:
1) Th
he triangle A(1,
A 2) B(4, 4)
4 and C(3, 7)
7 is first refflected in th
he line y = 0 onto triang
gle ABC
an
nd then trian
ngle ABC is
i reflected in the origin
n onto triang
gle ABC..
Write down th
he coordina
ates of, (i) A,
A B and C (ii) A, B and
a C.
Sol: No
ote: The line y = 0 meanss xaxis and the line x = 0 means ya
axis)
Re
eflection in y = 0 means reflection
r
in xaxis.
x
i) Since, refle
ection in the xaxis
x
is give
en by MX(x, y)
y = (x, y)
A = refle
ection of A(1,, 2) in the x
axis = (1, 2
2)
Similarly, B
B = (4, 4) an
nd C = (3, 7
7)
ii) Since, refle
ection in the origin
o
is given by M0(x, y)) = (x, y)
A = refle
ection of A(1
1, 2) in the origin = (1, 2)
Similarly, B
B = (4, 4) and
a C = (3,, 7)
2) A point P is reflected
r
in the xaxis. Coordinattes of its im
mage are (8, 6). (i) Find
d the co
orrdinates of P (ii) Find the coordina
ates of the im
mage of P under
u
reflecttion in the yaxis.
y
Sin
Sol: i) P = (8, 6)
nce, Mx (8, 6) = (8, 6)
ii) Coordinate
es of the imag
ge of P unde
er reflection in the yaxis = (8, 6)
3) Pe
erform the operations
o
Mx My & My Mx on the point (3,4). State whetther Mx My =My Mx.
If yes,
y
then sttate whetherr it is always
s true.
Sol: Mx My (3, 4) = Mx [My (3, 4)]
My Mx (3, 4) = My [Mx (3, 4)]
= Mx (3, 4) = (3, 4)
= My (3, 4) = (
3, 4)
Mx My = My Mx
Ye
es, it is alwayys true.

Inv
variant Poiint:
z

Any point that remainss unaltered under


u
a given
n transforma
ation is called
d an invariant.

Note:: In case of an invariant point, the point


p
is its ow
wn image i.e
e. reflection o
of a point in the point
itself is
i invariant e.g.
e reflection
n of origin in origin etc.
Simila
arly, every po
oint in a line is invariant under the re
eflection in th
he same line i.e. if a point P in line
AB is reflected in AB itself, the
e point is inva
ariant.
Pointss P(x, 0), (0, y), (0, 0) are
e invariant po
oints for the reflection
r
in Mx, My and M0 respective
ely.
4) Th
he point P(5
5, 1) and Q(2, 2) are
e reflected in line x = 2.
2 Use grap
ph paper to find the
im
mages P and
d Q of pointts P and Q respectively
r
y in line x = 2.
2 Take 2 cm
m equal to 2 units.
Sol: Th
he graph of liine x = 2 is the
t straight line AB, as shown below, which is pa
arallel to yax
xis and is
at a distance of
o 2 units from
m it.
Ma
ark P(5, 1) and Q(2, 2)) on the sam
me graph pap
per.
To
o find P the image
i
of P:
Ma
ark P at th
he same disstance behin
nd AB as P is
be
efore it. Sincce P is 3 units before AB
B, its image
e P
will be 3 units behind
b
AB.
Clearly, the co
oordinates of
o P = (1, 1)
In the same way,
w
since Q(2,
Q
2) is 4 units befo
ore
AB
B, its image Q
Q will be 4 units
u
behind AB.
A
On
n marking po
osition of Q, we find: Q = (6, 2)
5) Us
se a graph paper
p
for thiis question. (Take two division
d
= 1 unit on both the axis).
Plot the pointts P(3, 2) an
nd Q(3, 2)). From P and Q, draw perpendicu
ulars PM an
nd QN on
the xaxis. (a
a) Write the
e coordinates of points M and N.
N (b) Name
e the image of P on
refflection in the
t
origin (c
c) Assign th
he special name
n
to geo
ometrical fig
gure PMQN and find
its
s area (d) Write the coordinates off the point to
t which M is
i mapped o
on reflection
n in (i) x
ax
xis (ii) yaxis
s (iii) origin..
Volume 1 of 2

Universal Tu
utorials X IC
CSE Maths

79

80

Sol: a) Coordinate of M = (3, 0)


0 and Coo
ordinates of N = (3, 0)
2
b) Image of P (3, 2) in orig
gin = (3, 2)) = Q
1
c) PMQN is a parallelogram,
Area of PM
MQN = 2(Area
a of PMN)0
01
2 1
1
1
2
= 2( 6 2) sq.
s units = 12
2 sq. units
1
d) i) M(3, 0) re
eflected in x
axis gives (3
3, 0)
ii) M(3, 0) re
eflected in y
axis gives (3,
(
0)
2
iii) M(3, 0) reflected
r
in origin
o
gives (
3, 0)
6) Fin
nd the coorrdinates of the
t
images of the follow
wing points
s under refle
ection in the xaxis,
(i) (2, 7) (ii) (5
5, 6) (iii) (2, 1)
( Mx = (2,, 1) (2, 1)
Sol: (i) Mx = (2, 7) (2, 7) (ii) Mx = (5, 6) (5, 6) (iii)
nd the coorrdinates of the
t
images of the follow
wing points
s under refle
ection in the yaxis,
7) Fin
(i) (3, 4) (ii) (7
7, 8) (iii) (5, 6)
6) (5, 6)
Sol: (i) My = (3, 4) (3, 4) (ii) My = (7, 8) (7, 8) (iii)) My = (5, 6
nd the coorrdinates of the
t
images of the follo
owing points
s under refllection in th
he origin,
8) Fin
(i) (1, 2) (ii) (2
2, 3) (iii) (4, 5)
3 (2, 3) (iii) M0 = (4, 5) (4, 5
5)
Sol: (i) M0 = (1, 2) (1, 2) (ii) M0 = (2, 3)
9) Th
he point P(a
a, b) is first reflection
r
in
n the origin to
t P and then reflected
d in the yax
xis to P.
If P
P has coorrdinates (3, 4) evaluate
e (a, b).
Sol: Th
he point P(a, b) is first refflected in orig
gin and then reflected in yaxis,
M0 = P(a, b)) P(a, b
b)
My = P(a, b)) P(a, b)
ut given P(3
3, 4)
P(a, b) = P(3,
P
4) a = 3 and b = 4
Bu
10) A(4, 2) is the image of A when reflected in the
e line x = 0, B(5, 6) is tthe image off B when
refflected in th
he y = 0. Find the coordinates of A and B. Also
o calculate le
ength of AB
B.
Sol: Given, A(4, 2
2) is the imag
ge of A; B(5, 6) is the image of B.
he point A is reflected in the
t line x = 0,
0 i.e. in the yaxis,
y
My = A(4,
A
2) A
A(4, 2)
Th
Th
he point B is reflected in the
t line y = 0,
0 i.e. in the xaxis,
x
Mx = B(5, 6) B
B(5, 6)
Distance, AB =

[5 ( 4)]2 + [ 6 ( 2)]2

97 = 9.85 units

11) Pe
erform Mx My and My Mx on the po
oint (4, 6). State whether Mx My = My Mx. If yes,
y
then
sta
ate whetherr it is always
s true.
Sol: Given a point P(4,
P
6)
No
ow, My Mx : P(4,
P
6) P(4,
P
6) P(4,
P
6) and
d Mx My : P(
4, 6) P(4
4, 6) P(4, 6)
My Mx = Mx My
Th
his is always true. Because any pointt reflected in xaxis or vicce versa is a
always equiv
valent to a
sin
ngle reflectio
on in the origiin.
12) A point P(8, 1) is reflected in the xaxis
x
to the
e point P. The
T
point P is then refflected in
own the co
oordinates of
o P (ii) W
Write down a single
the origin to point P. (i) Write do
ansformatio
on that maps
s P into P.
tra
Sol: Given, a point P(8, 1) whiich is reflecte
ed in xaxis followed
f
by reflection
r
in o
origin.
i) Mx : P(8, 1)
1 P(8, 1)

M0 : P(8, 1)
P(8, 1)
ii) My : P(8, 1)
1 P(8, 1)
Reflectio
on of a point in xaxis followed by reflection in orig
gin is same a
as reflection in yaxis.
M0 Mx My

80

Universal Tu
utorials X IC
CSE Maths

Volu
ume 1 of 2

Chapter 09:
0 Coordina
ate Geometry: Reflection

81

13) Fin
nd the coorrdinates of the image off P(5, 4) aftter
i) reflection
r
in
ny=0
ii) reflection in
n the line y = 4
Sol: Given a point P(5,
P
4)
i) reflection
r
in y = 0 i.e. in xaxis
x
Mx : P(5, 4) P(5, 4)
ii) reflection in y = 4 i.e. in y = 4
Th
he point P(5
5, 4) is on the
e line y = 4.
Coordinatess of its image
e is also (5
5, 4) i.e. the coordinates
rem
main unchan
nged.
Point P(5, 4) is said to
o be invarian
nt under refle
ection in the
line y = 4.
he points (4,, 1), (4, 1), (4, 1) and (4, 1) are the vertices
s of a rectan
ngle. If the rectangle
r
14) Th
is reflected in
n the line x = 5, find the coordinattes of the re
eflected recttangle, also
o find the
area and perim
meter of the
e reflected rectangle.
erimeter of ABCD
A
= AB
B + B C + C
C D + DA
Sol: Pe
= 2 + 8 + 2 + 8 = 20 units
Arrea = L B = B C C D
D =82
= 16 sq. units.

UNSOLV
VED EXER
RCISE 9.1:
CW Exerrcise
1) Re
efer the diagram,
i) Mark
M
the reflection, in the
e yaxis, of each
e
given point;
p
ii) state the co
ordinates of each image
e obtained;
iii) name the po
oints invarian
nt under refle
ection in ya
axis.

2) A point P is its own image under the re


eflection in a line l. Describe the posittion of the po
oint P with
resspect to the line l.
3) State the coo
ordinates of the following points under reflection in
n xaxis
i) (3,
ii) (5, 4)
( 2)
iii) (0, 0)
4) State the coo
ordinates of the following points under reflection in
n the line y = 0
iii) (1, 3)
i) (3,
ii) (8, 5)
(
0)
5) A point P is refflection in the
e origin. Co
ordinates off its image arre (2, 7) (i) Find the co
ordinates
of P (ii) Find th
he coordinates of the im
mage of P und
der reflection
n in the xaxis.
6) Th
he point P(x, y) is first refflected in the
e xaxis and then reflecte
ed in the orig
gin to P. If P
P has co
ord
dinates (8, 5); evaluate x and y.
7) Th
he triangle AB
BC, where A is (2, 6) B iss (3, 5) and C is (4, 7) iss reflected in
n the yaxis to
t triangle
AB
BC. Trianglle ABC is th
hen reflected
d in the origin
n to triangle ABC.
A
i) Write
W
down the
t coordina
ates of A, B
B and C
ii) Write down a single transformation th
hat maps tria
angle ABC onto triangle A
ABC.
Volume 1 of 2

Universal Tu
utorials X IC
CSE Maths

81

82

8) P and Q have coordinates (2, 3) and (5, 4) respectively. Reflect P in the xaxis to P and Q in
the yaxis to Q. State the coordinates of P and Q.
9) Find the image of point (4, 6) under the following operations:
(i) Mx My (ii) My Mx (iii) M0 Mx (iv) Mx M0 (v) M0 My (vi) My M0.
Write down a single transformation equivalent to each operation given above.
State whether: (a) M0 Mx = Mx M0 (b) My M0 = M0 My.
10) Point A(4, 1) is reflected as A in the yaxis. Point B on reflection in the xaxis is mapped as
B(2, 5). Write the coordinates of A and B.
11) Use graph paper for this question, the point P(5, 3) was reflected in the origin to get the image P
a) Write down the coordinates of P
b) If M is the foot of the perpendicular from P to the xaxis, find the coordinates of M
c) If N is the foot of the perpendicular from P to the xaxis, find the coordinates of N
d) Name the figure PMPN
e) Find the area of the figure PMPN.
12) Use a graph paper for this question. A(1, 1), B(5, 1), C(4, 2) and D(2, 2) are the vertices of a
quadrilateral. Name the quadrilateral ABCD. A, B, C and D are reflected in the origin onto A, B,
C and D on the graph sheet and write their coordinates. Are D, A, A and D collinear?
13) Use a graph paper for this question. (Take 10 small division = 1 unit on both axis) P and Q have
co-ordinates (0, 5) and (2, 4)
i) P is invariant when reflected in an axis. Name the axis
ii) Find the image of Q on reflection in the axis found in (i)
iii) (0, K) on reflection in the origin is invariant. Write the value of K
iv) Write the coordinates of the image of Q, obtained by reflecting it in the origin followed by
reflection in xaxis.
14) The points P(4, 1) and Q(2, 4) are reflected in line y = 3. Find the coordinates of P, the image
of P and Q, the image of Q.
HW Exercise
1) State the coordinates of the following points under reflection in yaxis
i) (6, 3)
ii) (1, 0)
iii) (8, 2)
2) State the coordinates of the following points under reflection in origin
i) (2, 4)
ii) (2, 7)
iii) (0, 0)
3) State the coordinates of the following points under reflection in the line x = 0
i) (6, 4)
ii) (0, 5)
iii) (3, 4)
4) A point P is reflection in the xaxis. Coordinates of its image are (4, 5)
i) Find the coordinates of P
ii) Find the coordinates of the image of P under reflection in the yaxis.
5) The point P(a, b) is first reflected in the origin and then reflected in the yaxis to P. If P has
coordinates (4, 6); evaluate a and b.
6) The point A(3, 2) is reflected in the xaxis to the point A. Point A is then reflected in the origin
to point A.
i) Write down the coordinates of A
ii) Write down a single transformation that maps A onto A
7) The point A(4, 6) is first reflected in the origin to the point A. Point A is then reflected in the y
axis to point A.
i) Write down the coordinates of A
ii) Write down a single transformation that maps A onto A.
82

Universal Tutorials X ICSE Maths

Volume 1 of 2

Chapter 09: Coordinate Geometry: Reflection

83

8) On a graph paper, plot the triangle ABC, whose vertices are at the points A(3, 1), B(5, 0) and
C(7, 4). On the same diagram, draw the image of the triangle ABC under reflection in the origin
O(0, 0).
9) Attempt this question on graph paper,
i) Plot A(3, 2) and B(5, 4) on graph paper. Take 2 cm = 1 unit on both the axis.
ii) Reflect A and B in the xaxis to A and B respectively. Plot these points also on the same
graph paper
iii) Write down:
a) the geometrical name of the fig. ABBA
b) the measure of angle ABB
c) the image A of A, when A is reflected in the origin
d) the single transformation that maps A to A.
10) Points (3, 0) and (1, 0) are invariant points under reflection in the line L1; points (0, 3) and (0, 1)
are invariant points on reflection in line L2.
i) Name or write equations for the lines L1 and L2
ii) Write down the images of points P(3, 4) and Q(5, 2) on reflection in L1. Name the images
as P and Q respectively
iii) Write down the images of P & Q on reflection in L2. Name the image as P & Q respectively
iv) State or describe a single transformation that maps P & P.
11) i) Point P(a, b) is reflected in the xaxis to P(5, 2). Write down the values of a and b.
ii) P is the image of P when reflected in the yaxis. Write down the coordinates of P
iii) Name a single transformation that maps P to P.
12) A point P(a, b) is reflected in the xaxis to P(2, 3). Write down the values of a and b. P is the
image of P, reflected in the yaxis. Write down the coordinates of P. Find the coordinates of
P, when P is reflected in the line, parallel to yaxis, such that x = 4.
13) The point P(3, 4) is reflected to P in the xaxis and O is the image of O (the origin) when
reflected in the line PP. Using graph paper, give,
i) the coordinates of P and O
ii) the length of the segments PP and OO
iii) the perimeter of the quadrilateral POPO
iv) the geometrical name of the figure POPO.
14) A triangle P(2, 1) Q(4, 4) R(7, 3) is first reflected in the origin into PQR and then PQR is
reflected in the line x = 0 onto PQR. Write down the coordinates of
i) P, Q and R
ii) P, Q and R
15) Perform the operation Rx Ry and Ry Rx on the point (2, 3). State whether Rx Ry = Ry Rx. If
yes then state whether it is always true.

PREVIOUS BOARD QUESTION:


1) Use graph paper for this question
A(0, 3), B(3, 2) and O(0, 0) are the vertices of triangle ABO
i) Plot the triangle on a graph sheet taking 2 cm = 1 unit on both the axes.
ii) Plot D the reflection of B in the Y axis, and write its coordinates.
iii) Give the geometrical name of the figure ABOD
iv) Write the equation of the line of symmetry of the figure ABOD.
2) Use graph paper to answer this question]
i) Plot the points A(4, 6) and B(1, 2)
ii) A is the image of A when reflected in Xaxis.
iii) B is the image of B when B is reflected in the line AA.
iv) Give the geometrical name for the figure AB AB.
Volume 1 of 2

Universal Tutorials X ICSE Maths

[2010]

[2009
83

84

3) Use a graph paper for this question.


[2007]
i) The point P(2, 4) is reflected about line x = 0 to get the image Q. Find the coordinates of Q.
ii) Point Q is reflected about the line y = 0 to get image R. Find the coordinates of R.
iii) Name the figure PQR
iv) Find the area of figure PQR
4) Use graph paper for this question.
[2006]
The points A (2, 3), B (4, 5) and C (7, 2) are the vertices of ABC
i) Write down the coordinates of A, B, C if ABC is the image of ABC, when reflected in the
origin.
ii) Write down the coordinates of A, B, C if ABC is the image of ABC, when reflected in
the xaxis.
iii) Mention the special name of the quadrilateral BCC B and find its area.
5) Use a graph paper for this question. (Take 10 small divisions = 1 unit on both axes). P and Q
have coordinates (0, 5) and (2, 4).
[2005]
i) P is invariant when reflected in an axis. Name the axis.
ii) Find the image of Q on reflection in the axis found in (i)
iii) (0, k) on reflection in the origin is invariant. Write the value of k.
iv) Write the coordinates of the image of Q, obtained by reflecting it in the origin followed by
reflection in xaxis.
6) Use a graph paper for this question A(1, 1), B(5, 1), C (4, 2) and D (2, 2) are the vertices of a
quadrilateral. Name the quadrilateral ABCD. A, B, C and D are reflected in the origin onto A1, B1,
C1 and D1 on the graph sheet and write their coordinates. Are D, A, A1 and D1 collinear? [2004]

MISCELLANEOUS EXERCISE:
1) The points A(4, 11), B(5, 3), C(2, 15) and D(1, 1) are the vertices of a parallelogram is reflected
in the y-axis and then in the origin, find the coordinates of the final images. Whether it remains a
parallelogram or not? Write a single transformation that brings the above change.
2) Find the images of (0, 0), (1, 2) and (2, 1) in the line (i) x = 1 (ii) y = 5.
3) A point P is reflected to P in the x-axis. The coordinates of its image are (2, 3). Find,
i) the coordinates of P
ii) the coordinates of the image P of P under reflection in the y-axis
iii) the coordinates of the image Q of the point Q(1, 2) in the line PP.
4) i) Points (5, 0) and (2, 0) are invariant points under reflection in the line L1. Points (0, 5) and
(0, 2) are invariant points on reflection in L2
ii) Write down the images of points P (5, 8) and Q (3, 4) on reflection L1. Name the images as
P and Q respectively
iii) Write down the images of P and Q on reflection in L2. Name the images as P and Q
respectively
iv) State or describe a single transformation that maps P onto P.
5) State the coordinates of the following points after reflection in the xaxis (y = 0)

3 1
,
iii)

2 2
6) The point P(3,4) is reflected to P in the x-axis and O is the image of O(0, 0) in the line PP. Find
i) the coordinates of P and O
ii) the length of segments PP and OO
iv) the geometrical name of the figure POPO
iii) the perimeter of the quadrilateral POPO.
7) State the coordinates of the following points after reflection in the origin (x = 0)

84

i) (2, 2)

ii) (4, 5)

i) (8, 3)

ii) (6, 8)

5 3
,
iii)

2
2

Universal Tutorials X ICSE Maths

Volume 1 of 2

Chapter 09: Coordinate Geometry: Reflection

85

8) The point P(3, 2) on reflection in x-axis is mapped on P. Then P on reflection in the origin is
mapped as P. Find the coordinates of P and P. Write down a single transformation that maps
P onto P.
9) The point P(3, 5) on reflection in y-axis is mapped on P. The point P on reflection in the origin
is mapped as P. Find the coordinates of P and P. Write down a single transformation that
maps P onto P.
10) i) Plot A(2, 3) and B(4, 5) on a graph paper
ii) Reflect A, B in the y-axis to A, B. Plot these points on the same graph
iii) Write down:
a) the geometrical name of the figure ABBA
b) the measure of the angle ABB
c) the image B to B, when B is the reflected in the point (0, 0).
11) Write down the coordinates of the image of the point (3, 2) when
i) reflected in the x-axis
ii) reflected in the y-axis
iii) reflected in the x-axis followed by the reflection in the y-axis
iv) reflected in the origin
12) In the following question, the coordinates of the point and their image are given. State the mode
of transformation in each case,
i) Point (3, 2) Image (3, 2)
ii) Point (5, 2) Image (5, 2)
iii) Point (1, 2) Image (1, 2)
iv) Point (3, 2) Image (3, 4)
v) Point (1, 2) Image (3, 2)
13) In the following question, the coordinates of the image of a point P and their mode of
transformation is given. Write the coordinates of the given point P
i) Image (5, 2) Mode of transformation = y-axis
ii) Image (3, 2) Mode of transformation = y-axis = x-axis
iii) Image (5, 6) Mode of transformation = origin
iv) Image (5, 6) Mode of transformation = origin.
14) Find the image of the point (10, 15) under a reflection in the line x + 2y 5 = 0.
15) Give the images of the following points under reflection in the line y + 2 = 0
i) (1, 1)
ii) (5, 2)
iii) (3, 3)
iv) (3, 3)
16) A point P(a, b) becomes (3, c) after reflection in the x-axis and P becomes (d, 6) after reflection
in the origin. Find the values of a, b, c and d.
17) My(x1, y2) (x2, 2) and M0(x2, 2) (5, y3). Find x1, y1, x2 and y3.
18) Mx(m, n) (2, p) and M0(m, n) (q, 5). Find the value m, n, p and q.
19) Find the reflection of the points (5, 2), (2, 3), (4, 5) and (6, 2) in the line y = x.
20) Find the images of (0, 0), (1, 2) and (2, 1) in the line (i) x = 1 (ii) y = 5.
21) The point P(3, 2) on reflection in x-axis is mapped as P. Then P on reflection in the origin is
mapped as P, Find the co-ordinates of Pand P.
22) The point P(2, 6) is reflected to the following points. Draw the axis of the reflection in each case,
and find its equation.
b) R (2, 8)
c) S(6, 6)
a) Q (2, 2)
23) Using graph paper for,
i) Plot the points A(3, 5) and B(-2, -4). Use 1 cm = 1 unit on both axis.
ii) A is the image of A when reflected in the x-axis. Write down the co-ordinates of A and plot it
on the graph paper.
Volume 1 of 2

Universal Tutorials X ICSE Maths

85

86

24)

25)

26)

27)

28)
29)
30)

31)
32)

iii) B is the image of B when reflected in the y-axis, followed by reflection in the origin. Write
down the co-ordinates of B and plot it on the graph paper.
iv) Write down the geometrical name of the figure AA BB.
v) Name two invariant points under reflection in the x-axis.
The vertices of a triangle are, A(1, 1), B(1, 2), C(3, 4).
a) Find the co-ordinates of its image when reflected in the x-axis.
b) Find the co-ordinates of its image when reflected in the y-axis.
c) Find the co-ordinates of its image when reflected in the origin.
d) Find the co-ordinates of its image when reflected in the line y = 2.
e) Find the co-ordinates of its image when reflected in the line x = 3.
The image of triangle OXY under reflection in the origin, is the triangle OX1Y1, where X1(3, 4)
is image of X and Y1(0, 5) is the image of Y.
i) Write down the co-ordinates of X and Y.
ii) What kind of figure is the quadrilateral XYX1Y1? Give a reason for your answer, State, with a
reason, whether the figure XYX1Y1 has any lines of symmetry.
B, C have co-ordinates (3, 2) and (0, 3). Find
i) the image B of B under reflection in the x-axis;
ii) the image C of C under reflection in the line BB;
iii) calculate the length of B C.
A(2, 3), B(0, 5) and C(0, 5) are the vertices of a triangle in the co-ordinate plane.
i) Name the two points which are invariant under reflection in the y-axis.
ii) Name the image of triangle ABC by reflection in the y-axis. What type of figure is formed by
triangle ABC and this image? Give reasons for your answer.
If N is the foot of the perpendicular from A to the yaxis and A is the image of A(3, 1) when
reflected in the origin. Find the Coordinates of N.
The point A(3, 1) is reflected, such that its image is A(3, 4), find the equation of the mirror line.
If A is the image of A, when reflected in the x-axis, B is the image of B, when reflected in the yaxis, followed by reflection in the origin, then write down the geometrical name of the figure
AAB. Also write down two invariant points on the figure, under reflection in the x-axis.
Mx(m, n) (2, p) and M0 (m, n) (q, 5). Find the values of m, n, p and q.
A point P(a, b) becomes (3, c) after reflection in the xaxis, and P becomes (d, 6) after
reflection in the origin. Find the values of a, b, c and d.

ANSWER OF UNSOLVED EXERCISE:


CW Exercise 9.1:
1) (ii) A(2, 1); B(3, 0); C(5, 3); D(4, 2); E(5, 4); F(4, 0); G(0, 3); H(0, 4) (iii) G and H
2) point P lies in the line l
3) (i) (3, 2) (ii) (5, 4) (iii) (0, 0)
4) (i) (3, 0) (ii) (8, 5) (iii) (1, 3)
5) (i) (2, 7) (ii) (2, 7) 6) x = 8; y = 5
7) (i) (2,6);(3,5);(4,7) (ii)Reflection in x-axis 8) P(2, 3); Q(5, 4)
9) (i) (4, 6); reflection in origin (ii) (4, 6); reflection in origin (iii) (4, 6); reflection in yaxis
(iv) (4, 6); reflection in yaxis (v) (4, 6); reflection in xaxis (vi) (4, 6); reflection in xaxis
(a) True (b) True
10) (4, 1) and (2, 5)
11) (a) (5, 3) (b) (5, 0) (c) (5, 0) (d) Parallelogram (e) 30 sq. units
12) collinear
13) (i) yaxis (ii) (2, 4) (iii) K = 0 (iv) (2, 4)
14) (4, 5); (2, 2)
HW Exercise 9.1:
1) (i) (6, 3) (ii) (1, 0) (iii) (8, 2)
86

2) (i) (2, 4) (ii) (2, 7) (iii) (0, 0)

Universal Tutorials X ICSE Maths

Volume 1 of 2

Chapter 09: Coordinate Geometry: Reflection

3)
5)
7)
9)
10)
11)
13)
14)
15)

87

(i) (6, 4) (ii) (0, 5) (iii) (3, 4)


4) (i) (4, 5) (ii) (4, 5)
a = 4; b = 6
6) (i) (3, 2) (ii) Reflection in yaxis
(i) (4, 6) (ii) Reflection in xaxis
(iii) (a) Isosceles trapezium (b) 45 (c) (3, 2) (d) Reflection in yaxis
(i) L1 is xaxis & L2 is yaxis (ii) P(3,4); Q(5,2) (iii) P(3,4)Q(5,2) (iv)reflection in origin
(i) a = 5, b = 2 (ii) (5, 2) (iii) reflection in origin 12) (i) a = 2; b = 3, P(2, 3), P(6, 3)
(i) P(3, 4) O (6, 0) (ii) PP = 8 unit; OO = 6 units (iii) 20 units (iv) Rhombus
(i) P(2, 1) Q(4, 4) R(7, 3) (ii) P(2, 1) Q(4, 4) R(7, 3)
Rx Ry = Ry Rx, Yes, it is always true

Previous Board Question:


1) (ii) (3, 2) (iii) (it may be a quadrilateral) iii) x = 0
2) A (4, 6), B (7, 2), kite
3) (i) Q (2, 4) (ii) R(2, 4) (iii) right angled triangle (iv) Area PQR 16 sq unit
4) (i) A (2, 3) B (4, 5) C(7, 2) (ii) A(2, 3) B(4, 5) C(7, 2)
(iii) Isosceles trapezium Area = 21 sq. unit.
5) (i) y axis (ii) Q(2, 4) (iii) k = 0 (iv) Q(2, 4)
6) A(1,1), B(5,1), C(4,2) D(2, 2), yes
Miscellaneous Exercise:
1) (4, 11), (5, 3), (2, 15), (1, 1)
2) (0, 10); (1, 12) and (2, 11)
3) (i) (2, 3) (ii) (2, 3) (iii) (3, 2)
4) (i)L1 is x-axis;L2 is y-axis (ii)P(5,8)Q(3,4) (iii)P(5,8)Q(3,4) (iv)P is reflection; P in origin
5) (3/2, 1/2)
6) (i) (3, 4), (6, 0) (ii) 8 * 6 (iii) 20 ( iv) rhombus
7) (5/2, 3/2)
8) P(3, 2) P(3, 2)
9) P(3, 5) P(3, 5)
10) (iii)(a) isosceles trapezium (b) 45 (c) (4,5)
11) (i) (3, 2) (ii) (3, 2) (iii) (3, 2) (iv) (3, 2)
12) i) yaxis ii) origin iii) yaxis iv) y=1 v)x=2
13) (i) (5, 2) (ii) (2, 3) (iii) (3, 5) (iv) (5, 6)
14) (4, 13)
15) (i) (1, 5) (ii) (5, 2) (iii) (3, 1) (iv) (3, 7)
16) a = 3, b = 6, c = 6, d = 2
17) x1 = 5, y1 = 2, x2 = 5, y3 = 2
18) m = 2, n = 5, p = 5, q = 2
19) (2, 5) (3, 2) (5, 4) (2, 6)
20) (i) (2, 0) (1,2)(4,1)(ii)(0,10)(1,12)(2,11)
21) P (3, 2), P (3, 2)
22) (a) y = 4 (b) y = 1 (c) a = 3, b = 4
23) ii) A(3. 5) (iii) (2, 4) (iv) Isosceles trapezium (v) (4, 0); (2, 0)
24) (a) (1, 1), (1, 2), (3, 4) (b) (1, 1), (1, 2), (3, 4) (c) (1, 1), (1, 2), (3, 4)
(d) (1, 5), (1,6), (3,8) (e) (5, 1) (7, 2), (3, 4)
25) i) X(3, 4), Y (0, 5) (ii) Parallelogram (diagonals bisect each other); No.
26) i) B (3, 2) ii) C(6, 3) iii) 3 5

27) (i) B & C (ii) Kite (adj. sides are equal)

28) N(0, 1)
29) y = 2.5
30) AAB - An isosceles triangle B and B are invariant pts.
31) [2, 5, 5, 2]
32) a = 3, b = 6, c = 6, d = 3

Volume 1 of 2

Universal Tutorials X ICSE Maths

87

88

Chapter 10: Ratio and Proportion


Ratio:
The relation of one type of quantity to another quantity of the same type is called ratio.
If a and b are two quantities of the same kind and with same units such that b 0; then the
a
is called the ratio between a and b, which is denoted by a : b. (read as a is to b)
quotient
b
The quantities a and b are called terms of the ratio.
The first quantity a is called the first term or the antecedent and the second quantity b is called
the second term or consequent of the ratio a:b.
The second term of a ratio cannot be zero.
A ratio must always be expressed in its lowest terms.
A ratio is in its lowest terms, if HCF of its both the terms is 1 (Unity).
Ratios a : b and b : a cannot be equal unless a = b i.e., a : b b : a unless a = b
In other words, the order of the terms in a ratio are important.

Properties:
z

If both the terms a and b of the ratio are multiplied or divided by the same quantity then, the
ratio remains unchanged.
a
ma
=
a:b is the same as ma:mb
b
mb
a b
a
am
:
=
a:b is the same as
m m
b
bm

Increase (or decrease) in a ratio:


Let the price of an article decrease from `24 to 20; we say that the price has decreased in the
ratio 24: 20 = 6 : 5
5
times of its original price
6
In General : If a quantity increases or decreases in the ratio a: b

The new price of the article =

The new (resulting) quantity =

b
times of the original quantity.
a

Commensurable and Incommensurable quantities:


If the ratio between any two quantities of the same kind and having the same unit, can be
expressed exactly by the ratio between two integers, the quantities are said to
commensurable, otherwise incommensurable.
Eg.: i) The ratio between 2

1
1
7 7 7
2
and 3 = : =
= 2: 3, which is the ratio between two
3
2
3 2 3
7

integers 2 and 3.
Therefore, 2
88

1
1
and 3 are commensurable quantities.
3
2
Universal Tutorials X ICSE Maths

Volume 1 of 2

Chapter 10: Ratio and Proportion

ii) The ratio between

89

3 and 5 i.e.

two integers, therefore,

3 :5 which can never be expressed as the ratio between

3 and 5 are incommensurable quantities.

Comparison of Ratios:
First Method:
z

By making the second terms i.e. denominators, equal. We can compare the ratios. Eg.
which ratio is greater 3 : 8 or 5: 12.

Since, ratio are 3 : 8 and 5 : 12 i.e.

3
5
and
, and L.C.M. of their second terms 8 and 12
8
12

is 24; therefore:
3 33
9
5
52
10
=
=
=
=
and
8 8 3 24
12 12 2 24
Making second term of each ratio equal to L.C.M. 24

Since,

10
9
5
3
>
> or ratio 5 : 12 is greater than ratio 3: 8
therefore
24 24
12 8

Second Method:
z

For any two ratios a : b and c : d.


a c
< ad < bc
b d
a c
ii) (a:b) = (c:d) =
ad = bc
b d
a c
iii) (a:b) > (c:d) > ad > bc
b d

i) a: b < c : d

Composition of Ratios:
Compound Ratio:
z When two or more ratios are multiplied term-wise, the ratio thus obtained is called
compound ratio i.e. compound ratio of (a:b) and (c:d) is (ac:bd).
Duplicate Ratio:
z It is the compound ratio of two equal ratios i.e. Duplicate ratio of a : b = Compound ratio of
a : b and a: b = a2 : b2.
2
2
z Eg. duplicate ratio of 2 : 3 = 2 : 3 = 4 : 9
Triplicate Ratio:
z It is the compound ratio of three equal ratios i.e. Triplicate ratio of a : b = compound ratio of
a : b, a: b and a: b = (a a a): (b b b) = a3 : b3
3
3
z Eg Triplicate Ratio of 2 : 3 = 2 : 3 = 8 : 27
Subduplicate Ratio:
z

The sub-duplicate ratio of a : b is

a : b . i.e. a1/2: b1/2

Eg. sub-duplicate ratio of 9 : 16 =

9 : 16 = 3 : 4

Subtriplicate Ratio:
z

The subtriplicate ratio of a : b is

Volume 1 of 2

a : 3 b . i.e. a1/3: b1/3

Universal Tutorials X ICSE Maths

89

90
z

Eg. Subtriplicate Ratio of 27 : 64 =

27 : 3 64 = 3: 4

Reciprocal Ratio:
1 1
: = b : a.
a b

Reciprocal ratio of a : b is

Eg. Reciprocal ratio of 3 : 5 =

1 1
: =5:3
3 5

SOLVED EXAMPLES 10.1:


1) i) If 2x + 3y : 3x + 5y = 18 : 29, find x : y.
ii) If x : y = 2 : 3, find the value of 3x + 2y : 2x + 5y.

Sol: i) 2x + 3y : 3x + 5y = 18 : 29
58x + 87y = 54x + 90y
x
3

=
i.e. x : y = 3 : 4
y
4
ii) x : y = 2 : 3

2x + 3y
18
=
3 x + 5y
29

4x = 3y

x
2
=
y
3

3 x + 2y
Now, 3x + 2y = 2x + 5y =
=
2x + 5y

x
3 + 2
y
(Dividing each term by y)
x
2 + 5
y

2
+2
x
2
3
=
= 12 : 19
(
= )
2
y
3
2 + 5
3
If a : b = 5 : 3, find (5a + 8b) : (6a 7b).
Let, a : b = 5 : 3
If a = 5x, then b = 3x;
5a + 8b
5 5x + 8 3x
49 x
And
=
=
= 49 : 9
6a 7b
6 5x 7 3x
9x
i) What quantity must be added to each term of the ratio 8 : 15 so that it becomes equal to
3 : 5?
ii) What quantity must be subtracted from each term of the ratio a:b so that it becomes
c:d?
i) Let x be added to each term of the ratio 8 : 15.
3

2)
Sol:

3)

Sol:

Given:

8+x
3
=
40 + 5x = 45 + 3x
15 + x
5

ii) Let x be subtracted, then,

ax
c
=
bx
d

cx dx = bc ad x(c d) = bc ad

90

x= 2

1
2

ad dx = bc cx
x=

bc ad
c d

Universal Tutorials X ICSE Maths

Volume 1 of 2

Chapter 10: Ratio and Proportion

91

4) The work done by (x 3) men in (2x + 1) days and the work done by (2x + 1) men in (x + 4)
days are in the ratio 3 : 10. Find the value of x.
(x 3)(2x + 1) = 3
Sol: Assuming that all the men do same amount of work in one day; we get
(2x + 1)(x + 4) 10

2x 2 + x 6 x 3
2

3
10

2x + 8 x + x + 4
20x2 50x 30 = 6x2 + 27x + 12

5)

Sol:

6)

Sol:

2x 2 5 x 3
2

3
10

2x + 9 x + 4
14x2 77x 42 = 0 2x2 11x 6 = 0
1
(x 6) (2x + 1) = 0 (on factorizing)
x = 6 or x =
2
x = is not possible as it will make number of men (x 3) negative. x = 6
Arrange the ratios 5 : 6, 8 : 9 and 13 : 15 in ascending order of magnitude.
5 8
13
The given ratios are ,
and
and L.C.M. of their second terms 6, 9 and 15 is 90.
6 9
15
5
5 15
75 8
8 10
80
13
13 6
78
=
=
,
=
=
and
=
=

6
6 15
90 9
9 10
90
15
15 6
90
78
80
75
5
13
8
Since,
<
<
i.e.
<
<
5 : 6 < 13 : 15 < 8 : 9
90
90
90
6
15
9
Find the compound ratio of:
i) 3a : 2b, 2m : n and 4x : 3y.
ii) a b : a + b, (a + b)2 : a2 + b2 and a4 b4 : (a2 b2)2.
i) Required compound ratio = (3a 2m 4x) : (2b n 3y)
24amx
=
= 4amx : bny
6bmy
ii) Required compound ratio = [(a b) (a + b)2 (a4 b4)] : [(a + b) (a2 + b2) (a2 + b2)2]
=

(a b )(a + b )2 (a 2 + b 2 )(a 2 b 2 )
(a + b )(a 2 + b 2 )(a 2 b 2 )(a + b )(a b )

= 1:1

7) Find the ratio compounded of the duplicate ratio of 5 : 6, reciprocal ratio of 25 : 42 and the
subtriplicate ratio of 216 : 343.
Sol: Since, duplicate ratio of 5 : 6 = 52 : 62 = 25 : 36
1
1
Reciprocal ratio of 25 : 42 =
:
= 42 : 25
25 42

And subtriplicate ratio of 216 : 343 =

216 : 3 343 = 6 : 7

the required compounded ratio = (25 42 6) : (36 25 7)


25 42 6
=
= 1:1
35 25 7

UNSOLVED EXERCISE 10.1:


CW Exercise:
5a 3b
5a + 3b
2) If (a b) : (a + b) = 1 : 11; find the ratio (5a + 4b + 15) : (5a 4b + 3).

1) If a : b = 5 : 3; find :

3) If

2n 2
m+n
2
= ; find :
.
m + 3n
3
3m 2 + mn

Volume 1 of 2

Universal Tutorials X ICSE Maths

91

92

4) Find

x
: when x2 + 6y2 = 5xy.
y

5) Two numbers are in the ratio 2 : 3. If 5 is added to each number, the ratio becomes 5 : 7. Find
the numbers.
6) Two positive numbers are in the ratio 3 : 5 and the difference between their squares is 400; find
the numbers.
7) The work done by (x 2) men in (4x + 1) days and the work done by (4x + 1) men in (2x 3)
days are in the ratio 3 : 8. Find the value of x.
8) Compare the ratios:
i) 5 : 8 and 7 : 9
ii) 3 : 16 and 5 : 32
9) Arrange the ratios 3 : 5, 7 : 10 and 18 : 25 in descending order of magnitude.
10) Find the compound ratio of:
i) 3 : 5 and 8 : 15

ii)

2 : 1, 3 :

5 and

20 : 9.

11) Find duplicate ratio of:


i) 3 : 4

ii) 3 3 : 2 5

12) Find subduplication ratio of:


i) 9 : 16
ii) (x y)4 : (x + y)6
13) Find the reciprocal ratio of:
i) 5 : 8
ii) x/3 : y/7
14) If m : n is the duplicate ratio of m + x : n + x; show that x2 = mn
15) If 4x + 4 : 9x 10 is the triplicate ratio of 4 : 5, find x.
HW Exercise:
1) If x : y = 4 : 7; find the value of (3x + 2y) : (5x + y).
4a + 3b
2) If a : b = 3 : 8, find the value of
.
6a b
yx
y
3
= ; find the value of .
3) If
x
8
x
4) If the ratio between 8 and 11 is the same as the ratio of 2x y to x + 2y; find the value of

7x
.
9y

5) What quantity must be subtracted from each term of the ratio 9 : 17 to make it equal to 1 : 3?
6) What quantity must be added to the each terms of the ratio m + n : m n to make it equal to
(m + n)2 : (m n)2?
7) Arrange the ratios 5 : 8, 7 : 10 and 17 : 24 in ascending order of magnitude.
8) Find the compound ratio of:
i) 2 : 3, 9 : 14 and 14 : 27
ii) 2a : 3b, mn : x2 and x : n
9) Find triplicate ratio of:
m n
:
i) 1 : 3
ii)
2 3
10) Find subtriplicate ratio of:
i) 64 : 27
ii) x3 : 125y3
11) If 3x + 4 : x + 5 is the duplicate ratio of 8 : 15; find x.
12) Find the ratio compounded of the reciprocal ratio of 15 : 28, the subduplicate ratio of 36 : 49
and triplicate ratio of 5 : 4.

92

Universal Tutorials X ICSE Maths

Volume 1 of 2

Chapter 10: Ratio and Proportion

93

Proportion:

The equality of two ratios is called Proportion.


Four nonzero quantities of the same kind a, b, c and d are said to be in proportion if a : b = c : d.
This is often expressed as a : b : : c : d read as a is to b as c is to d.
a and d are called the extreme terms and b and c are called the mean terms.

a c
or ad = bc
=
b d
Product of extremes = Product of means
d is called the fourth proportional.

If the quantities a, b, c, d are in proportion

Continued Proportion:
z

a, b, c, are said to be in continued proportion if

a b
=
b c

a, b, c, d are said to be in continued proportion, if

a b c
= =
b c d

Mean Proportion:
z
z

a b
or b2 = ac or b = ac
=
b c
In this case, b is called the mean proportional or geometric mean between a and c.

Let a, b, c be in continued proportion then

Third Proportion
z

Also, when a:b = b:c, then c is called the third proportional to a and b

b2
Then, a:b = b:c c =
a

Fourth Proportion
z

If a:b = c:d or a:b :: c:d, then d is called the fourth proportional to a, b, c.


To find fourth proportional to a, b, c. Let the fourth proportional to a, b, c be x then,

a:b = c:x

bc
x=

k method:
a
c
=
= k (say). Therefore, a = bk, c = dk.
b
d

If a, b, c, d are in proportion, then

If a, b, c are in continued proportion, a : b = b : c = then

Therefore, a = bk and b = ck a = ck k = ck2

a
b
=
= k (say).
b
c

SOLVED EXAMPLES 10.2:


1) Find:

i) the fourth proportional to 3, 6 and 4.5.


ii) the mean proportional between 6.25 and 0.16.
iii) the third proportional to 1.2 and 1.8.

Volume 1 of 2

Universal Tutorials X ICSE Maths

93

94

Sol: i) Let fourth proportional to 3, 6 and 4.5 be x


3 : 6 = 4.5 : x
3 x = 6 4.5
x=9
ii) Let the mean proportional between 6.25 and 0.16 be x.
6.25, x and 0.16 are in continued proportion 6.25 : x = x : 0.16
x x = 6.25 0.16 x2 = 1
x=1
iii) Let the third proportional to 1.2 and 1.8 be x.
1.2, 1.8 and x are in continued proportion 1.2 : 1.8 = 1.8 : x
1.8 1.8
x=
x = 2.7
1.2
2) If p : q : : q : r, prove that p : r = p2 : q2.
Sol: p : q : : q : r q2 = pr
p2 : q2 =

p2
p2
=
pr
q2

(q2 = pr)

p
= p:r
r
3) If a b and a : b is the duplicate ratio of a + c and b + c, prove that c is the mean
proportional between a and b.

(a + c )
a
=
b
(b + c )2
2

Sol: Given:

a(b2 + c2 + 2bc) = b(a2 + c2 + 2ac)

ab2 + ac2 + 2abc = a2b + bc2 + 2abc


c2 (a b) = ab(a b)

ac2 bc2 = a2b ab2


c2 = ab
c is mean proportional between a and b
4) If q is the mean proportion between p and r, prove that:
1
1
1
p2 q2 + r2 = q4 2 2 + 2 .
q
r
p
Sol: Since, q is the mean proportional between p and r q2 = pr
1
1
1
q4
q4
RHS = q4 2 2 + 2 = 2 q 2 + 2
q
r
p
r
p
p 2r 2

p 2r 2

(q2 = pr)
p2
r2
= r2 q2 + p2
5) Find two numbers whose mean proportional is 24 and third proportional is 1536.
Sol: Let the required numbers be
Therefore, 24 is mean proportional between a and b.
a : 24 = 24 : b
ab = 576
(I)
and, 1536 is third proportional to a and b.
a : b = b : 1536
b2 = 1536a
(II)
=

q2 +

From eq. (I); ab = 576 a =


Substituting, a =
b = 96 and a =
94

576
b

576
576
in eqn. II, we get b2 = 1536
b3 = 1536 576
b
b
576
=6
96

The required numbers are 6 and 96


Universal Tutorials X ICSE Maths

Volume 1 of 2

Chapter 10: Ratio and Proportion

95

UNSOLVED EXERCISE 10.2:


CW Exercise:
1) Find the fourth proportional to:
i) 1.5, 4.5 and 3.5
ii) 3a, 6a2 and 2ab2
2) Find the third proportional to:
2
i) 2 and 4
ii) a b and a2 b2
3
3) Find the mean proportional between:

i) 17.5 and 0.007

iii) a b and a3 a2b

ii) 6 + 3 3 and 8 4 3

4) What least number must be added to each of the numbers 16, 7, 79 and 43 so that the resulting
numbers are in proportion?
5) If q is the mean proportional between p and r, show that pqr(p + q + r)3 = (pq + qr + pr)3.
6) If three quantities are in continued proportion; show that the ratio of the first to the third is the
duplicate ratio of the first to the second.
7) Find two numbers such that the mean proportional between them is 12 and the third proportional
to them is 96.
3a 2 10c 2
3b 2 10d 2

8) If

a
c
= , prove that each of the given ratio is equal to
b
d

9) If

x y z
2 x 3 3 y 3 + 4z 3
2 x 3 y + 4z
= = , prove that:
=

a b c
2a 3 3b 3 + 4c 3
2a 3b + 4c

HW Exercise:
1) Find the third proportional to: (i) 3, 0.6 and 4.5 (ii) 6p, 12p2 and 4pq2
2) What least number must be subtracted from each of the numbers 7, 17 and 47 so that the
remainders are in continued proportion?
3) If y is the mean proportional between x and z; show that xy + yz is the mean proportional
between x2 + y2 and y2 + z2.

4) If y is the mean proportional between x and z, prove that :

x 2 y 2 + z2
x 2 y 2 + z 2

= y4.

5) Given four quantities a, b, c and d are in proportion. Show that:


(a c)b2 : (b d)cd = (a2 b2 ab) : (c2 d2 cd)

x y
+ and x 2 + y 2 .
y x
7) If p : q = r : s; then show that: mp + nq : q = mr + ns : s.
a c
8) If = , prove that each of the given ratio is equal to:
b d
6) Find the third proportional to

i)

5a + 4c
5b + 4d

ii)

13a 8c
13b 8d

8a 3 + 15c 3 3

iii) 3
3
8b + 15d

9) If a, b, c and d are in proportion, prove that:


1

i)

xa 3 5 yb 3 3
4a 2 + 9b 2

=
xc 3 5 yd 3
4c 2 + 9d 2

Volume 1 of 2

ii)

13 a + 17 b
=
13 c + 17d

Universal Tutorials X ICSE Maths

2ma 2 3nb 2
2mc 2 3nd 2

95

96

Some important properties of proportion:


z

If four quantities a, b, c and d form a proportion i.e. If a : b :: c : d, many other proportion


may be obtained using the properties of fractions. Some of these proportion are given
below:

Invertendo:
z

If a : b = c : d then b : a = d : c.
Proof:
a
c
If a : b = c : d
=
b
d
b
d
=
(Taking reciprocal on both sides)

a
c
b:a=d:c

Alternendo:
z

If a : b = c : d then a : c = b : d.
Proof:
a
c
If a : b = c : d
=
ad=bc
b
d

a
b
=
c
d

a:c=b:d

Componendo:
z

If a : b = c : d, then a + b : b = c + d : d.
Proof:
a
c
a
c
If a : b = c : d
=

+1=
+1
(adding 1 on both sides)
b
d
b
d
a+b
c +d
=
a+b:b=c+d:d

b
d

Dividendo:
z

If a : b = c : d, then a b : b = c d : d.
Proof:
a
c
a
c
a:b=c:d
=

1=
1
(subtracting 1 from both sides)
b
d
b
d
ab
c d
=
ab:b=cd:d

b
d

Componendo and Dividendo:


z

96

If a : b = c : d, then a + b : a b = c + d : c d.
Proof:
a:b=c:da+b:b=c+d:d
(By Componendo)
a+b
c +d

=
(i)
b
d
a:b=c:dab:b=cd:d
(By Dividendo)
ab
c d
=
(ii)

b
d
Universal Tutorials X ICSE Maths

Volume 1 of 2

Chapter 10: Ratio and Proportion

97

a+b
c +d
=
ab
c d
Thus a : b = c : d a + b : a b = c + d : c d

Dividing (i) by (ii) we get,

Summary:
If

a
c
=
then
b
d

b
d
=
a
c

(By Invertendo)

a
b
=
c
d

(By Alternendo)

a+b
c +d
=
b
d

(By Componendo)

ab
c d
=
b
d

(By Dividendo)

a+b
c +d
=
ab
c d

(By Componendo and Dividendo)

SOLVED EXAMPLES 10.3:


1) If a : b = c : d, show that 3a + 2b : 3a 2b = 3c + 2d : 3c 2d

Sol: a : b = c : d

a
c
=
b
d

3a
3c
=
2b
2d

3a + 2b
3c + 2d
=
3a 2b
3c 2d

(Multiplying each side by

3
)
2

(By Componendo and Dividendo)

3a + 2b : 3a 2b = 3c + 2d : 3c 2d
2) If, 3a + 5b : 3c + 5d = 3a 5b : 3c 5d show that a, b, c and d are in proportion.
Sol: Applying alternendo: 3a + 5b : 3c + 5d = 3a 5d : 3c 5d
3a + 5b : 3a 5d = 3c + 5d : 3c 5d

3a + 5b
3c + 5d
=
3a 5 b
3c 5d

Now, by componendo and dividendo, we get:

(3a + 5b ) + (3a 5b)


(3a + 5b) (3a 5b )

a
c
=
b
d

3) If p =

Sol: p =

(3c + 5d ) + (3c 5d )
(3c + 5d ) (3c 5d )

6a
6c
=
10b
10d

i.e. a, b, c and d are in proportion

4 xy
p + 2x
p + 2y
, find the value of
+
.
x+y
p 2x
p 2y

4 xy
2y
p + 2x
2y + x + y
P

=
(Applying componendo and dividendo)
x+y
x+y
p 2x
2y x y
2x
=

Again, p =
Volume 1 of 2

x + 3y
yx

4 xy
P
2x

=
x+y
2y
x+y
Universal Tutorials X ICSE Maths

97

98

p + 2y
2x + x + y
=
(Applying componendo and dividendo)
p 2y
2x x y
3x + y
xy

x + 3y
x + 3y
p + 2x
p + 2y
3x + y
3x + y
x + 3y 3 x y
+
=
+
=

=
= 2
p 2x
p 2y
xy
y x
y x
yx
yx

4) If a : b = c : d; prove that, (a2 + ac + c2) : (a2 ac + c2) = (b2 + bd + d2) : (b2 bd + d2)
Sol: a : b = c : d
a
c
=
= k (say)
a = bk and c = dk
b
d

(a2 + ac + c2) : (a2 ac + c2) =


=
=

a 2 + ac + c 2
a 2 ac + c 2

b 2 k 2 + (bk )(dk ) + d 2 k 2

(As a = bk and c = dk)

b 2 k 2 (bk )(dk ) + d 2 k 2

(
(b

)=
)

k 2 b 2 + bd + d 2
k

bd + d

(b2 + bd + d2) : (b2 bd + d2)

5) If x, y and z are in continued proportion, prove that, x2 y2 : x2 + y2 = x z : x + z.

Sol: x, y and z are in continued proportion

x y
=
= k (say)
y z

x = yk, y = zk & x = yk = (zk)k = zk2


x2 y2 : x2 + y2 =
=

(
(k

x +y

)=k
k
+ 1)

y 2 k2 1
y

Also, x z : x + z =
=

x2 y 2

(
(z k

=
2
2

y 2k 2 y 2

( x = yk)

y 2k 2 + y 2

1
+1

(i)

xz
zk 2 z
=
x+z
zk 2 + z

)=k
k
+ 1)

z k2 1

(x = zk2)

1
+1

(ii)
x2 y2 : x2 + y2 = x z : x + z

From (i) and (ii), we get

6) Using the properties of proportion, solve the equation for x,

Sol: Applying componendo and dividendo, we get

(x + 1)3
(x 1)3

x 3 + 3 x 341
=
.
91
3x 2 + 1

x 3 + 3 x + 3 x 2 + 1 341 + 91
=
x 3 + 3 x 3 x 2 1 341 91

x +1 6
432
216
6
=
=
=
250
125
x 1 5
5

Again, applying componendo and dividendo, we get


x + 1+ x 1 6 + 5
2 x 11
=

=
x = 11
x + 1 x 1 6 5
2
1

98

Universal Tutorials X ICSE Maths

Volume 1 of 2

Chapter 10: Ratio and Proportion

99

UNSOLVED EXERCISE 10.3:


CW Exercise:
1) If a : b = c : d, prove that,
i) 5a + 7b : 5a 7b = 5c + 7d : 5c 7d
ii) (9a + 13b) (9c 13d) = (9c + 13d) (9a 13b)
iii) xa + yb : xc + yd = b : d
5 x + 6y
5 x 6y
2) If
=
; then prove that, x : y = u : v.
5u + 6v
5u 6v
3) If (7a + 8b) (7c 8d) = (7a 8b) (7c + 8d), prove that a : b = c : d.
x + 3a
x + 3b
6ab
4) If x =
, find the value of,
+
.
a+b
x 3a
x 3b
a 2b 3c + 4d
a 2b + 3c 4d
5) If
=
, show that 2ad = 3bc.
a + 2b 3c 4d
a + 2b + 3c + 4d

a b
= .
x y

6) If (a2 + b2) (x2 + y2) = (ax + by)2; prove that,

7) If a, b, and c are in continued proportion, prove that,


i)

a 2 + ab + b 2 a
=
b 2 + bc + c 2 c

ii)

a2 + b2 + c 2

(a + b + c )

ab+c
a+b+c

8) Using properties of proportion, solve for x,


i)

x + 5 + x 16
x + 5 x 16

7
3

ii)

x +1 + x 1
x +1 x 1

4x 1
2

HW Exercise:
1) If a : b = c : d, prove that, (6a + 7b) (3c 4d) = (6c + 7d) (3a 4b)
a
c
3a 5b 3c 5d
2) Given,
=
= , prove that,
b
d
3a + 5b 3c + 5d

3) If a =

4 6
2+ 3

, find the value of,

a+2 2
a2 2

a+2 3
a2 3

4) If (a + b + c + d) (a b c d) = (a + b c d) (a b + c d), prove that, a : b = c : d.


5) If (a2 + c2) : (ab + cd) = (ab + cd) : (b2 + d2); prove that, a, b, c and d in proportion.
6) Using properties of proportion, solve for x,

3x + 9x 2 5
3x 9x 2 5

=5.

PREVIOUS BOARD QUESTION:


1) If x=

a +1 + a 1

, using properties of proportion show that x2 2ax + 1 = 0.

[2012]
a +1 a 1
2) The monthly pocket money of Ravi and Sanjeev are in the ratio 5:7. Their expenditures are in
the ratio 3:5. If each save `80 every month, find their monthly pocket money.
[2012]
3) 6 is the mean proportion between two numbers x and y and 48 is the third proportional of x and
y. Find the numbers.
[2011]
4) Using componendo and dividend, find the value of x,

Volume 1 of 2

3x + 4 + 3x 5
3x + 4 3x 5

Universal Tutorials X ICSE Maths

= 9.

[2011]
99

100

5) Given x=

a2 + b2 + a2 b2
a2 + b2 a2 b2

. Use componendo and dividend to prove that b2 =

6) If x, y, z are in continued proportion, prove that


a 3 + 3ab 2

(x + y )2
(y + z )2

x
z

2a 2 x
x2 + 1

[2010]
[2010]

63
. Using Componendo and Dividendo find a : b.
62

[2009]
b + 3a b
8) What least number must be added to each of the numbers 5, 11, 19 and 37 so that they are in
proportion?
[2009]

7) Given that

9) If

a c
8a 5b 8a + 5b
=
, prove that = .
b d
8c 5d 8c + 5d

10) If x =
11) If

a + 3b + a 3b
a + 3b a 3b

[2008]

, prove that 3bx2 2ax + 3b = 0.

3 x + 5y 7
= , find x : y.
3 x 5y 3

[2007]
[2006]

12) What number must be added to each of the numbers 6, 15, 20 and 43 to make them
proportional?
[2005]
13) What number should be subtracted from each of the numbers, 23, 30, 57 and 78, so that the
remainders are in proportion?
[2004]

MISCELLANEOUS EXERCISE:
1)
2)
3)
4)

If a : b = 3 : 5, find (10a + 3b) : (5a + 2b)


If 5x + 6y : 8x + 5y = 8 : 9, find x : y.
If (3x 4y) : (2x 3y) = (5x 6y) : (4x 5y), find x : y.
Find the:
i) duplicate ratio of 2 2 : 3 5
ii)
iii)
iv)
v)
vi)

triplicate ratio of 2a : 3b
subduplicate ratio of 9x2a2 : 25y6b2
subtriplicate ratio of 216 : 343
reciprocal ratio of 3 : 5
ratio compounded of the duplicate ratio of 5 : 6, the reciprocal ratio of 25 : 42 and the sub
duplicate ratio of 36 : 49.
5) Find the value of x, if:
i) (2x + 3) : (5x 38) is the duplicate ratio of

6)
7)
8)
9)
10)

100

5 :

6.

ii) (2x + 1) : (3x + 13) is the subduplicate ratio of 9 : 25.


iii) (3x 7) : (4x + 3) is the subtriplicate ratio of 8 : 27.
Arrange 5 : 6, 7 : 8 and 13 : 16 in descending order of magnitude.
What quantity must be added to each term of the ratio x : y so that it may become equal to c : d?
Two numbers are in the ratio 5 : 7. If 3 is subtracted from each of them, the ratio between 2 : 3.
Find the numbers.
If 15(x2 y2) = 7xy, find x : y.
Find the:
ii) third proportional to a2 b2 and a b
i) fourth proportional to 2xy, x2 and y2
3
2
iii) mean proportional to (x y) and (x x y)
Universal Tutorials X ICSE Maths

Volume 1 of 2

Chapter 10: Ratio and Proportion

101

11) Find two numbers such that the mean proportional between them is 14 and third proportional to
them is 112.
12) If x and y be unequal and x : y is the duplicate ratio x + y and y + z, prove that z is mean
proportional between x and y.
13) If

a2 + c 2
ab + cd
= 2
, prove that a : b = c : d.
ab + cd
b + d2

14) If q is the mean proportional between p and r, prove that:

p3 + q 3 + r 3
2 2 2

p q r

1
p

1
q

r3

15) If a, b and c are in continued proportion, prove that, a : c = (a2 + b2) : (b2 + c2).
x +a x +b
2ab
16) If x =
+
, find the value of
.
a+b
x a x b
17) If (4a + 9b) (4c 9d) = (4a 9b) (4c + 9d), prove that, a : b = c : d.
a
c
18) If
= , show that, (a + b) :(c + d) = a 2 + b 2 : c 2 + d 2 .
b
d
ax by
by cz
cz ax
x
y
z
+
+
=
= , prove that,
= 3.
19) If
(a + b)(x y ) (b + c )(y z ) (c + a)(z x )
a
b
c
20) Two numbers are in the ratio 3 : 5. If 9 is added to each number, the ratio becomes 2 : 3. Find
the numbers.
21) If x =
22) Solve,
23) Solve

2a + 1 + 2a 1
2a + 1 2a 1
a+x + ax
a+x ax

, show that, x2 4ax + 1 = 10.


= b.

(3 x + 2)2 + (3 x 2)2
(3 x + 2)2 (3 x 2)2

5
.
4

ANSWERS TO UNSOLVED EXERCISE:


CW Exercise 10.1:
1) 8/17
5) 20 and 30
9) 3:5 < 7:10 < 18:25
13) i) 8:5 ii) 3y : 7x
HW Exercise 10.1:
1) 26/27

2) 5:1
6) 15 and 25

3) 1/15
7) x = 3.5

4) 3 or 2
8)i) 5:8<7:9 ii)3:16>5:32

10) i) 8:25 ii) 2 2 :3


15) 15

11) i) 9:16 ii) 27:20

12) i) 3:4 ii)(xy)2:(x+ y)3

2) 18/5

3) 11/8

4) 3/2

7) 5:8; 7:10; 17:24

8) i) 2:9 ii) 2am:3bx

11) 580/611

12) 25:8

5) 5
9) i) 1:27 ii) 27m3 :8n3

n m
2m
10) i) 4:3 ii) x:5y
6)

CW Exercise 10.2:
1) i) 10.5 ii) 4a2b2

2) i) 6 ii) (a +b)(a2b2)

4) 5

7) 6 and 24

Volume 1 of 2

3) i) 0.35 ii) 2 3 iii) a(a b)

Universal Tutorials X ICSE Maths

101

102

HW Exercise 10.2 :
3) 2

7) xy

CW Exercise 10.3:
4) 2

8) i) 20 ii) 5/4

HW Exercise 10.3:
3) 2

6) 1

Previous Board Question:


2) 200; 280
8) 2

3) 3 and 12
11) x : y = 25: 6

4) x = 7
12) Required no is 3

Miscellaneous Exercise:
1) 9:5
2) 14:19
3) 1:1
4) (i) 8 : 45 (ii) 8a3:27b3 (iii) 3xa2 :5y3b iv) 6 : 7 v) 5 : 3 vi) 1 : 1
cy dx
7
5
13
5) i) 16 ii) 34 iii) 27
6)
>
>
7)
8
6
16
d c
a+b
xy
9) 5:6
10) i)
ii)
iii) x(x y)
2
ab
2ab
16) 2
20) 27, 45
22) 2
b +1

102

Universal Tutorials X ICSE Maths

7) 3 : 2
13) 6

8) 15 and 21
11) 7 and 28
23)

4 1
,
3 3

Volume 1 of 2

Chapter 11: Remainder and Factor Theorems

103

Chapter 11: Remainder and Factor Theorems


Introduction
Polynomial:
An algebraic expression of the form a0 + a1x + a2x2 + anxn, where n is a positive integer
a0, a1, a2 ..an R, the set of real numbers is called a polynomial of degree n. where an 0
A polynomial in variable x is denoted by f(x), g(x), r(x) etc.
For example 3x2 5x + 6 is a polynomial of degree 2.
If f(x) is a polynomial in x; when f(x) is divided by (x a) the remainder is f(a).

Reminder theorem:
If a polynomial f(x), is divided by (x a); a R the remainder is f(a)
Eg. If f(x) is divided by (x 3), the remainder is f(3).
x 3
Example:
x 2 x 2 5x + 8
f(x) = x2 5x + 8 find f(2)
x 2 2x
2
2
+
f(x) = x 5x + 8 f(2) = (2) 5 2 + 8
3x + 8
= 4 10 + 8 = 2= the remainder
3x + 6
When x2 5x + 8 is divided by x 2
+

It is clear from the example that when f(x) is divided by x 2


2
the remainder is f(2)
The method of finding the remainder without actually performing the process of division is
called Reminder theorem
Note: For finding the remainder, express given divisor in the form (x a); and then remainder = f(a).
Divisor
b
a
x+a

ax + b = a x +
a

Remainder
b
f
a

f(a)
b
f
a

b
f
ax b = a x
a

a
Alternative Method:
Step 1: Put the divisor equal to zero and solve the equation obtained to get the value of its variable.
Step 2: Substitute the value of the variable, obtained in step 1, in the given polynomial and simplify
it to get the required remainder.

Factor Theorem:
When a polynomial f(x) is divided by x a, the remainder is f(a). And, if remainder f(a) = 0;
then x a is a factor of the polynomial f(x).
For example: Let f(x) = x2 5x + 6 be divided by x 3; then remainder = f(3)

= (3)2 5 3 + 6 = 0 Remainder = 0
x 3 is a factor of f(x) = x2 5x + 6
Volume 1 of 2

Universal Tutorials X ICSE Maths

103

104

SOLVED EXAMPLES 11.1:


1) In each case, find the remainder when x2 8x + 4 is divided by,
i) x 1
ii) x + 1
iii) 2x + 1
2
Sol: Here, f(x) = x 8x + 4
i) By remainder theorem, if f(x) is divided by (x 1), the remainder is f(1):
f(1) = 12 8 1 + 4
Remainder = 3
f(x) = x2 8x + 4
=1 8 + 4 = 3
ii) By remainder theorem, if f(x) is divided by (x + 1), the remainder is f(1):
f(1) = (1)2 8 (1) + 4
[x + 1 = x (1)]
Remainder = 13

1
iii) By remainder theorem, if f(x) is divided by (2x + 1), the remainder is f :
2
2

1
1 1
1
f = 8 + 4 = + 4 + 4
2
2
4
2

Remainder = 8

1
4

1
4
What number should be added to 2x3 3x2 + x so that when the resulting polynomial is
divided by x 2, the remainder is 3?
Let the number added be k and the resulting polynomial be f(x).
f(x) = 2x3 3x2 + x + k
Given, when f(x) is divided by x 2, the remainder = 3
i.e. f(2) = 3
x2=0x=2
3
2
2(2) 3(2) + 2 + k = 3
16 12 + 2 + k = 3 i.e., k = 3
The required number to be added = 3
i) if x 2 is a factor of x2 7x + 2a, find the value of a.
ii) Find the value of a if the division of ax3 + 9x2 + 4x 10 by x + 3 leaves a remainder of 5.
i) Here, f(x) = x2 7x + 2a and x 2 = 0 x = 2
Since, x 2 is a factor of f(x) = x2 7x + 2a
Remainder, f(2) = 0 (2)2 7(2) + 2a = 0 a = 5
ii) Here, f(x) = ax3 + 9x2 + 4x 10 and x + 3 = 0 x = 3
Given, remainder is 5; f(3) = 5
a(3)3 + 9(3)2 + 4(3) 10 = 5
27a + 81 12 10 = 5 or a = 2
Given that x + 2 and x 3 are factors of x3 + ax + b; calculate the values of a and b.
Here, f(x) = x3 + ax + b; Given, x + 2 is a factor of f(x) = x3 + ax + b;
f(2) = 0
(x + 2 = 0 x = 2)
3
(2) + a(2) + b = 0 2a + b = 8
(i)
3
Again, given that: x 3 is factor of f(x) = x + ax + b;
f(3) = 0
(x 3 = 0 x = 3)
3
(3) + a(3) + b = 0 3a + b = 27
(ii)
On solving eqn. (i) and (ii), we get a = 7 and b = 6

= 8

2)

Sol:

3)

Sol:

4)
Sol:

104

Universal Tutorials X ICSE Maths

Volume 1 of 2

Chapter 11: Remainder and Factor Theorems

105

UNSOLVED EXERCISE 11.1:


CW Exercise
1) Find, in each case, the remainder when:
ii) 2x3 + 9x2 x 15 is divided by 2x + 3
i) 4x3 3x2 + 2x 4 is divided by 2x + 1
2) Show that:
i) x 2 is a factor of 5x2 + 15x 50
ii) 3x + 2 is a factor of 3x2 x 2
iii) x + 1 is a factor of x3 + 3x2 + 3x + 1
3) i) If 2x + 1 is a factor of 2x2 + ax 3, find the value of a;
ii) Find the value of k, if 3x 4 is a factor of expression 3x2 + 2x k.
4) If 2x + 1 is a factor of (3k + 2)x3 + (k 1), find the value of k.
5) Find the value of m and n so that x 1 and x + 2 both are factors of, x3 + (3m + 1)x2 + nx 18.
6) When x3 + 2x2 kx + 4 is divided by x 2, the remainder is k. Find the value of constants k.
7) If x3 + ax2 + bx + 6 has x 2 as a factor and leaves a remainder 3 when divided by x 3, find the
values a and b.
8) What should be subtracted from x3 + 3x2 8x + 14 so that on dividing it by x 2, the remainder
is 10.
HW Exercise:
1) Find, in each case, the remainder when:
ii) 4x3 + 4x2 27x + 16 is divided by 2x 3
i) x3 + 3x2 12x + 4 is divided by x 2
2) Use the remainder theorem to find which of the following is a factor of 2x3 + 3x2 5x 6
ii) 2x 1
iii) x + 2
iv) 3x 2
v) 2x 3
i) x + 1
3) Find the value of constants a and b when x 2 and x + 3 both are the factors of expression
x3 + ax2 + bx 12.
4) Find the value of a, if the division of ax3 + 9x2 + 4x 10 by x + 3 leaves a remainder 5.
5) The expression 2x3 + ax2 + bx 2 leaves a remainder 7 and 0 when divided by 2x 3 and x + 2
respectively. Calculate the values of a and b.
6) What number should be added to 3x3 5x2 + 6x so that when resulting polynomial is divided by
x 3, the remainder is 8?
7) The polynomials 2x3 7x2 + ax 6 and x3 4x2 + (a + 1) x 12 leave the same remainder when
divided by x 2. Find the value of a.

Application of Remainder Theorem to Factorise the given expression:


z

To factorize a given expression the following steps may be followed:


Find one value for the variable x for which the value of the expression is equal to zero.
Generally this value is a factor of the constant term. Let the value be a
Divide the expression by x a.
Factorize further the quotient, if possible.

SOLVED EXAMPLES 11.2:


1) Using the Factor Theorem, show that (x 3) is a factor of x3 7x2 + 15x 9. Hence,
factorise the given expression.
Sol: x 3 = 0 x = 3
Remainder obtained on dividing x3 7x2 + 15x 9
By x 3 = (3)3 7(3)2 + 15(3) 9 = 27 63 + 45 9 = 0
(x 3) is a factor of x3 7x2 + 15x 9.
Now, dividing (x3 7x2 + 15x 9) by (x 3); we get quotient = x2 4x + 3
x3 7x2 + 15x 9 = (x 3)(x2 4x + 3) = (x 3)(x2 3x x + 3)
= (x 3)[x(x 3) 1(x 3)] = (x 3) (x 3) (x 1)
Volume 1 of 2

Universal Tutorials X ICSE Maths

105

106

2) Factorise, using the Remainder Theorem, each of the following expressions:


i) x2 + 3x 4
ii) x2 2x 15
Sol: i) First of all find one value of the variable x for which the value of the expression x2 + 3x 4 is
equal to zero. Such a value of x is 1.
For x = 1, x2 + 3x 4 = (1)2 + 3 1 4 = 1 + 3 4 = 0 x 1 is factor of x2 + 3x 4.
Now, divide x2 + 3x 4 by x 1.
On dividing x2 + 3x 4 by x 1; we get x + 4.
x2 + 3x 4 = (x 1)(x + 4)
ii) For x = 5, x2 2x 15 = (5)2 2 5 15 = 25 10 15 = 0
x 5 is factor of x2 2x 15
On dividing x2 2x 15 by x 5; we get x + 3 x2 2x 15 = (x 5) (x + 3)
3) Using the Remainder Theorem, factorise the expression 2x3 + x2 2x 1 completely.
Sol: For x = 1, the value of given expression = 2(1)3 + (1)2 2(1) 1
2x 2 + 3 x + 1
3
=2+121=0
x 1 2x + x 2 2x 1
3
2
x 1 is a factor of 2x + x 2x 1
2x 3 2x 2
3
2
2
+
2x + x 2x 1 = (x 1)(2x + 3x + 1)
2
3 x 2 2x 1
= (x 1)(2x + 2x + x + 1)
3x 2 3x
= (x 1)[2x(x + 1) + 1(x + 1)]

+
= (x 1) (x + 1) (2x + 1)
x 1
x 1

+
x

UNSOLVED EXERCISE 11.2:


CW Exercise
1) Using the Factor Theorem, show that:
i) (x + 1) is a factor of x2 x 2. Hence, factorise x2 x 2.
ii) (2x + 3) is a factor of 10x2 + 3x 18. Hence, factorise 10x2 + 3x 18.
2) Factorise, using the Remainder Theorem; each of the following algebraic expression:
ii) 3x2 x 4
iii) 4x2 5x 6
iv) 5x2 + 6x 8
i) 2x2 + x 3
3) Using the Remainder Theorem; factorise the expression 3x3 + 10x2 + x 6. Hence, solve the
equation 3x3 + 10x2 + x 6 = 0.
4) Given that x 2 and x + 1 are factors of f(x) = x3 + 3x2 + ax + b; calculate the values of a and b.
Hence, find all the factors of f(x).
5) If x a is a common factor of expressions f(x) = x2 + px + q and g(x) = x2 + mx + n; show that,
nq
a=
.
pm

6) The polynomials ax3 + 3x2 3 and 2x3 5x + a, when divided by x 4, leave the same
remainder in each case. Find the value of a.
7) Find the number that must be subtracted from the polynomial 3y3 + y2 22y + 15, so that the
resulting polynomial is completely divisible by y + 3.
8) (3x + 5) is a factor of the polynomial (a 1)x3 + (a + 1)x2 (2a + 1)x 15. Find the value of a.
For this value of a, factorise the given polynomial completely.
9) The polynomial px3 + 4x2 3x + q is completely divisible by x2 1; find the values of p and q.
Also, for these values of p and q factorize the given polynomial completely.
106

Universal Tutorials X ICSE Maths

Volume 1 of 2

Chapter 11: Remainder and Factor Theorems

107

HW Exercise
1) Using the Factor Theorem, show that:
i) (x 5) is a factor of 3x2 13x 10. Hence, factorise 3x2 13x 10.
ii) (x 2) is a factor of x3 2x2 9x + 18. Hence, factorise the expression x3 2x2 9x + 18
completely.
iii) (x + 5) is a factor of 2x3 + 5x2 28x 15. Hence, factorise the expression 2x3 + 5x2 28x 15
completely.
iv) (3x + 2) is a factor of 3x3 + 2x2 3x 2. Hence, factorise the expression 3x3 + 2x2 3x 2
completely.
2) Using the Remainder Theorem; factorise each of the following expression completely:
ii) 2x3 + x2 13x + 6
iii) 3x3 +2x2 23x 30 iv) 4x3 + 7x2 36x 63
i) x3 x2 4x + 4
3) Factorise the expression, f(x) = 2x3 7x2 3x + 18. Hence, find all possible values of x for which
f(x) = 0.
4) The expression 4x3 bx2 + x c leaves remainders 0 and 30 when divided by x + 1 and 2x 3
respectively. Calculate the values of b and c. Hence, factorize the expression completely.
5) Find the value of a, if (x a) is a factor of x3 ax2 + x + 2.
6) Find the value of m, if mx3 + 2x2 3 and x2 mx + 4 leave the same remainder when each is
divided by x 2.
7) When the polynomial x3 + 2x2 5ax 7 is divided by (x 1), the remainder is A and when the
polynomial x3 + ax2 12x + 16 is divided by (x + 2), the remainder is B. Find the value of a if
2A + B = 0.

PREVIOUS BOARD QUESTION:


1) Using the Remainder Theorem factorise completely the polynomial: 3x3 + 2x2 19x + 6. [2012]
2) Find the value of k if (x 2) is a factor of x3 + 2x2 kx + 10. Hence determine whether (x + 5) is
also a factor.
[2011]
3) When divided by x 3 the polynomials x3 px2 + x + 6 and 2x3 x2 (p + 3) x 6 leave the
same remainder. Find the value of p
[2010]
[2010]
4) Use the Remainder Theorem to factorise the following expression: 2x3 + x2 13x + 6
3
2
5) Given that x + 2 and x + 3 are factors of 2x + ax + 7x b. Determine the values of a and b.
[2009]
[2008]
6) If (x 2) is a factor of 2x3 x2 px 2.
i) find the value of p.
ii) With the value of p, factorise the above expression completely
7) Show that (x 1) is a factor of x3 7x2 + 14 x 8. Hence, completely factorise the above
expression.
[2007]
8) Show that 2x + 7 is a factor of 2x3 + 5x2 11x 14. Hence factorise the given expression
completely, using the factor theorem
[2006]
3
2
9) (x 2) is a factor of the expression x + ax + bx + 6. When this expression is divided by (x 3),
it leaves the remainder 3. Find the values of a and b
[2005]
[2004]
10) Use the factor theorem to factorise completely, x3 + x2 4x 4.

MISCELLANEOUS EXERCISE:
1) Find the remainder in the following cases, when f(x) is divided by g(x) is each case:
a) f(x) = 2x3 3x2 4x 5;
g(x) = 2x + 1
g(x) = x 1
b) f(x) = x3 6x2 + 9x + 7;
2
5
c) f(x) = 4x3 + 6x2 x + ;
g(x) = 2 3x
3
6
Volume 1 of 2

Universal Tutorials X ICSE Maths

107

108

2) Using factor theorem, find out whether polynomial g(x) is a factor of f(x) or not:
a) f(x) = x3 + x2 17x + 15;
g(x) = x 3
3
b) f(x) = 3x3 2x2 + 5x + 6;
g(x) = 2x +
2
c) f(x) = 3x3 + 6x 9;
g(x) = 4 3x
3) For what values of a the polynomial g(x) is a factor of f(x).
a) f(x) = x3 a2x + x + 2;
g(x) = x a
3
2
b) f(x) = x + a(x + 1) 2x + 4;
g(x) = x + a
4) The polynomial P(x) = kx3 + 9x2 + 4x 8, when divided by the polynomial q(x) = x + 3, leaves the
remainder 20. Find the value of k.
5) The remainder obtained by dividing kx2 3x + 6 by x 2 is twice the remainder obtained by
dividing 3x2 + 5x k by (x + 3). Find the value of k.
6) (x 2) is the factor of the polynomial x3 + ax2 + bx + 6 and when this polynomial is divided by
(x 3) leaves the remainder 3. Find the values of a and b.
7) If g(x) = 2x 3 is a factor of f(x) = 2x3 9x2 + x + p, find the value of p. Hence find all the factors.
8) Find the value of q if the polynomial f(x) = x3 + 2x2 13x + q is divisible by g(x) = x 2. Hence
find all the factors.
9) Factorize 2x3 9x2 + x + 12 and hence, solve 2x3 9x2 + x + 12 = 0
10) Use the factor theorem to factorise completely, x3 + x2 4x 4.
11) Find the number which should be added to x2 + x + 3 so that the resulting polynomial is
completely divisible by (x + 3).
12) Show that (x 1) is a factor of x3 7x2 + 14x 8. Hence, completely factorise the given
expression.
13) Using Remainder Theorem, factorise: 2x3 + 7x2 8x 28 completely.
14) When x3 + 3x2 mx + 4 is divided by x 2, the remainder is m + 3. Find the value of m.
15) What should be subtracted from 3x3 8x2 + 4x 3, so that the resulting expression has x + 2 as
a factor?
16) If (x + 1) and (x 2) are factors of x3 + (a + 1)x2 (b 2) x 6, find the values of a and b. And
then, factorise the given expression completely.

ANSWERS TO UNSOLVED EXERCISE:


CW Exercise 11.1
1) 6
6) 6

1
(ii) 0
4

2
3

3) (i) a = 5 (ii) K = 8

4) 2

7) a = 3 & b = 1

8) 8

5) m = 3 & n = 7

HW Exercise 11.1:
1) (i) 0 (ii) 2

2) (i) x + 1 (iii) x + 2 (v) 2x 3

3) a = 3, b = 4

4) 2

5) a = 3 & b = 3

7) 7

6) 46

CW Exercise 11.2:
1) i) (x + 1) (x 2) (ii) (2x + 3) (5x 6)
2) i) (2x + 3) (x 1) (ii) (3x 4) (x + 1) (iii) (4x + 3) (x 2) (iv) (5x 4) (x + 2)
3) (x + 3) ( 3x 2) (x +1); 3, 2/3, 1
108

4) a = 6; b = 8; (x + 1) (x 2) (x + 4)

Universal Tutorials X ICSE Maths

Volume 1 of 2

Chapter 11: Remainder and Factor Theorems

6) 1

109

8) a = 4; (3x+5) (x23) = (3x+5) (x+ 3 ) (x 3 )

7) 9

9) p = 3 q = -4; (x + 1) (x 1) (3x + 4)
HW Exercise 11.2:
1) i) (x 5) (3x + 2) (ii) (x 2)(x + 3)(x 3) (iii) (x + 5) (2x + 1) (x 3) (iv) (3x + 2) (x + 1) (x 1)
2) i) (x 1) (x + 2)(x 2) (ii) (2x 1)(x 2)(x + 3) (iii) (x 3)(x + 2)(3x + 5) iv) (x +3)(x 3)(4x + 7)
3) (x 2) (x 3) (2x + 3); 2, 3,
5) 2

6)

3
2

3
10

4) b = 8; c = 3; (x + 1) (2x 1) (2x + 3)
7) 4

Previous Board Question:


1) (x 2) (3x 1) (x + 3) 2) k = 13
3) p = 1
4) (x 2) (2x 1) (x + 3)
5) a = 9, b = 6
6) i) P = 5 (ii) (2x + 1) (x + 1) (x 2)
7) (x 1) (x 2) (x 4)
8) (2x + 7) (x + 1) (x 2) 9) a = 3, b = 1
10) (x + 1) (x 2) (x + 2)
Miscellaneous:
1) a) 4 (b) 11 (c) 229/54

2) (a) Yes (b) 135/64 (c) 55/9

3) (a) 2 (b) 4/3

4) 3

5) 14

6) a = 23, b = 39

7) p = 12; factors: 2, (x 4), (x + 1)

8) q = 10, factors: (x 1), (x + 5)

9) 2(x 4)(x + 1)(2x 3); x = 1, 4, 3/2

10) (x 1)(x + 1) (x + 2)

11) 9

12) (x 1) (x 2) (x 4)

13) (x 2) (x + 2) (2x + 7)

14) m = 7

15) 67

16) a = 1; b = 7; (x + 1) (x 2) (x + 3)

Volume 1 of 2

Universal Tutorials X ICSE Maths

109

110

Chapter 12: Matrices


Introduction:
Matrix:
A matrix is a rectangular arrangement of numbers, arranged in rows and columns enclosed by
a pair of brackets [ ]. In the form of horizontal and vertical lines.

5 5 3
Eg. ,
, [5 3 2] etc.
1 1 2
In a matrix, the horizontal lines are called rows and vertical lines are called columns.
Each number or entity in a matrix is called its elements.

Order of a matrix:
z

The order of a matrix = Number of rows in it Number of columns in it.

i.e. if a matrix has m rows and n columns, then its order will be m n (read as m by n) and
its contains mn elements.
Matrix are generally denoted by capital letters A, B, C etc. whereas the elements are
denoted by small letters.
Plural of matrix is matrices.

z
z
z

If A is a matrix with m rows and n columns, then it is written as Amn.


st
1
5 1 row
2
Example: A =
nd

2
7 2 row
3

1st col. 2nd col. 3rd col.


A is a matrix of order 2 3 (read as 2 by 3).

Matrix A has 2 rows and 3 columns, so the number of elements in it = 2 3 = 6 elements.

Note: If a matrix as 6 elements the possible order it can have is 1 6 (1 by 6), 2 3 (2 by 3), 3 2
(3 by 2) or 6 1 (6 by 1).

Types of Matrices:
z

Row Matrix: A matrix which has only one row is called a row matrix.
Eg.: A = [1 2]; B = [3 0 1]

Row matrix is also called a row vector. A is of order 1 2 and B is of order 1 3.


Column Matrix: A matrix which has only one column is called a column matrix.

5
3

Eg.: A = ; B = 8 .
1

2

Order is 2 1

order is 3 1

Column matrix is also called a column vector. A is of order 2 1 and B is of order 3 1.


Square Matrix: A matrix which has equal number of rows and columns is called a
square matrix.

110

Universal Tutorials X ICSE Maths

Volume 1 of 2

Chapter 12: Matrices

111

2 1
Eg.: A =
;B=
3 4

3 1 2

6 4 8 .
2 6 1

A is a square matrix of order 2 2 (or 2) and B is of order 3 3 (or 3).


Rectangular Matrix: A matrix in which the number of rows are not equal to the number
of columns is called a rectangular matrix.

2 4 7
Eg.: A =
;B=
3 0 2

6 2

1 6
3 1

Order is 2 3
Order is 3 2
Zero or Null Matrix: If each element of a matrix is Zero, it is called a zero or null matrix.

0
Eg.: [0]; [0 0]; ;
0

0 0

etc.
0 0
Diagonal Matrix: A square matrix, which has all its elements zero except those on the
leading (or principal) diagonal is called a diagonal matrix.

5 0 0
2 0

Eg.:
; 0 2 0
0
1

0 0 3

Note: In a square matrix, the leading (Principal) diagonal means the diagonal from top left to bottom
right.
z
z

Unit or Identity Matrix: A diagonal matrix, in which each element of its leading diagonal
is unity (i.e. 1) is called a unit or identity matrix. It is denoted by I.
It is a square matrix in which each elements of its leading diagonal is equal to 1, and all
other remaining elements of the matrix are zero.

1 0
Eg.: I =
; I =
0 1

1 0 0

0 1 0
0 0 1

Equality of Matrices: Two matrices are said to be equal if, both the matrices have the
same order, the corresponding elements of both the matrices are equal.

2 3
2 3
Eg.: If A =
and B =
then A = B
1 5
1 5

Operations on Matrices:
Addition of Matrices:
z
z

Two matrices are compatible for addition, if they are of the same order.
To add two matrices of the same order means to add the corresponding elements of both
the matrices.

Volume 1 of 2

2 1
3 2
2 1 3 2 2 + 3 1 + 2 5 3
Eg.: If A =
& B =
; then A + B =
+
=
=

5 6
1 4
5 6 1 4 5 + 1 6 + 4 6 10

Universal Tutorials X ICSE Maths

111

112

Subtraction of Matrices:
z

The same rule and method used in addition of matrices in applied for subtraction of
matrices.

4
5 4
3 0
5 4 3 0 5 3 4 0 2
Eg.: If A =
& B =
; then A B=

=
=

2 1
4 2
2 1 4 2 2 4 1 2 2 1

Notes:
In addition or subtraction of the matrices, the order of the resulting matrix is the same as the order
of the matrices added or subtracted.
If A, B and C are the matrices of the same order, then,
A+B=B+A

(i.e. addition of matrices is commutative)

A + (B + C) = (A + B) + C

(i.e. addition of matrices is associative)

A+X=BX=BA

Additive Identity:
In numbers O is the additive identity i.e. a + 0 = 0 + a = a for any number a.
Similarly, the additive identity of any matrix is the null (zero) matrix of the same order.
z

4 6
0 0
4 6
0 0
4 6
Eg.:
+
=
=
+

0 7
0 0
0 7
0 0
0 7

Additive Inverse:
If A and B are two matrices of the same order such that,
z A + B = B + A = a null matrix, then A is said to be the additive inverse of B and B is said to
be the additive inverse of A.
z The additive inverse of a matrix. A is its negative, i.e. A.
z

4 2
4 2
Eg.: If A =
then the additive inverse of A = A =

3 7
3 7

SOLVED EXAMPLES 12.1:


y
x 2
0 3
=
1) Find the values of x, y, a and b, if a

.
b + 1

1 5
2

Sol: x 2 = 0 x = 2
y=3
(If two matrices are equal, their corresponding elements are also equal)
a/2 = 1 a = 2
b+1=5b=4
x = 2, y = 3, a = 2 and b = 4

2 1
3 2
0 4 and C = 1 0 ; find,

ii) B A
iii) A + B C
5 4
3 2
5 3 4 + 2
2 6
Sol: i) A + C =
+
=
=

3 1
1 0
3 + 1 1 + 0
4 1
5 4
2) If A =
;B=
3 1
i) A + C

112

Universal Tutorials X ICSE Maths

Volume 1 of 2

Chapter 12: Matrices

113

2 1
5 4 2 5 1 4
3 3
ii) B A =

=
=

0 4
3 1 0 3 4 + 1
3 5
5 4
2 1
3 2
iii) A + B C =
+

3 1
0 4
1 0
7 5 3 2
=

3 3 1 0

(Evaluating A + B then subtracting C)

7 + 3 5 2
10 3
=
=

3 1 3 0
2 3
2 1 3
3 2
t
t
3) If matrix A =
and B =

; find transpose matrices A and B . If possible,


4 3 2
7 4
find: (i) A + At (ii) B + Bt.
2 4
3 7

Sol: A = 1 3 and Bt =

2 4
3 2
i) Since, the order of matrix A is 2 3 and that of At is 3 2; A + At is not possible.
Two matrices are compatible for addition or subtraction; only when they have the same order.
ii) Since, the order of matrix B is 2 2 and that of Bt is 2 2; B + Bt is possible.
t

3 2
3 7
3 + 3 2 + 7
6 5
And B + Bt =
+
=
=

7 4
2 4
7 2 4 + 4
5 8
8 6
3 5
4) If matrix A =
and B = 1 0 ; then solve for 2 2 matrix X such that:

2
4

i) A + X = B
ii) X B = A.
3 5
8 6
3 8 5 6
11 1
Sol: i) A + X = B X = B A =

=
=

1
0

2
4
1
+
2
0

4
4

3
8 6
3 5
5 11
ii) X B = A X = A + B =
+
=

2 4
1 0
1 4

UNSOLVED EXERCISE 12.1:


CW Exercise
1) State, whether the following statements are true or false. If false, give a reason.

i) If A and B are two matrices of orders 3 2 and 2 3 respectively; then their sum A + B is
possible.
ii) The matrices A23 and B23 are conformable for subtraction.
iii) Transpose of a 2 1 matrix is a 2 1 matrix.
iv) Transpose of a square matrix is a square matrix.
v) A column matrix has many columns and only one row.

x y + 2
3 1
2) Given:
=
; find x, y and z.
3 z 1
3 1
3) If A = [8 3] and B = [4 5]; find: (i) A + B (ii) B A
Volume 1 of 2

Universal Tutorials X ICSE Maths

113

114

4) Wherever possible, write each of the following as a single matrix,

1 2
1 2
2 3 4
0 2 3
0 1 2
3 4
(i)
+
(ii)

(iii)
+

3 4
1 7
5 6 7
6 1 0
4 6 7
6 8
5 3
t
t
t
5) Given: M =
, find its transpose matrix. M If possible, find: (i) M + M (ii) M M

2
4

2 0
7
6) Write the additive inverse of matrices A, B & C: where A = [6 5]; B =
and C = .
4 1
4
1 0
7) Given: A =
and B =
2 4
i) A + X = B

3 3

; find the matrix X in each of the following:


2 0

ii) A X = B

iii) X B = A

HW Exercise:
1) Solve for a, b and c; if:

2
4 a + 5
b + 4
i)
=

2
c 1
3
3

a b
a
3 1
ii)
=

0
b + c
2 0

2
1
6
2) If A = , B = and C = , find (i) B + C (ii) A C (iii) A + B C (iv) A B + C
5
4
2
3) Find, x and y from the following equations:
2
5
1 x 1
4 7
ii) [8 x] + [y 2] = [3 2]
i)

=

1
y

1
2

3 2
4) Given: A = [2 3], B = [0 2] and C = [1 4]; find the matrix X in each of the following:
ii) A X = B + C
i) X + B = C A
6 4
t
t
t
5) Given: A =
; find its transpose A . Also find (i) A + A (ii) A A

2
3

Multiplication of a matrix by a scalar (real number)


To multiply a matrix by a scalar means to multiply each of its elements by this scalar.

2 1
2 2 2 1
4 2
E.g.: 2
=
=

5 2
2 5 2 2
10 4

SOLVED EXAMPLES 12.2:


1 2
1) Given: A =
,B=
2 3

2 1
and C =
1
2

0 3
2 1 , find: A + 2B 3C.

1 2
2 1
0 3
Sol: A + 2B 3C =
+ 2
3

2
2 3
1
2 1
1 2
4 2
0 9
3 0
0 9
3 9
=
+

=

=

2
3
2
4
6

3
0
7
6

6 10
5
1
2) Given, matrix A = and matrix B = ; find matrix X such that, A + 2X = B.

3

7
114

Universal Tutorials X ICSE Maths

Volume 1 of 2

Chapter 12: Matrices

115

1
5
6
Sol: A + 2X = B 2X = B A = =
7
3
10
X=

1
2

6 21 6
3
= 1 10 =
10
2
5

UNSOLVED EXERCISE 12.2:


CW Exercise
1) Evaluate:

1 2
ii) 7

0 1

i) 3[5 2]

1 0 3 3
3
8
iii) 2
+
iv) 6 2
2 3 5 0
2
1

2) Find x and y if:

1
2
7
ii) x 4 =
2
y
8

i) 3[4 x] + 2[y 3] = [10 0]

4 2
3) If
+ 3A =
4 0

2 2

; find A.
1 3

3 6
t
4) Given A =
and A is its transpose matrix. Find:
0 9
1
1
A At
2
3
5) If I is the unit matrix of order 2 2; find the matrix M, such that:

i) 2A + 3At

ii) 2At 3A

1 0
i) M 2I = 3

4 1
HW Exercise

iii)

iv) At

1
A
3

2 5
ii) 5M + 3I = 4

0 3

1 1
3 1

and C =
; find
0
5 2
0
ii) A + 2C B
4 1
and B =
.
3 2

2 1
1) Given A =
, B =
3 0
i) 2A 3B + C
1 4
2) Given A =

2 3

0 0
ii) find a matrix C such that, C + B =

0 0

i) find the matrix 2A + B

3 x
1 3
z 7
3) If 2
+ 3
=
; find the values of x, y and z.
0 1
y 2
15 8
1 1
2 1
4) Given A =
and B =
. Solve for matrix X:

2
0

1 1
ii) 3X + B + 2A = 0
iii) 3A 2X = X 2B
i) X + 2A = B
0
5) If M = and N =
1

Volume 1 of 2

1
, show that, 3M + 5N =
0

5

3

Universal Tutorials X ICSE Maths

115

116

Multiplication of Matrices:
Two matrices A and B can be multiplied if and only if the number of columns in A is equal to
the number of rows in B. i.e. if A is of order mn & B is of order np then AB is of order m p
To get the product of matrices the following steps may be adopted.
z Multiply every element of first row of matrix A with the corresponding element of first column
of B and add them to get the first element of the first row of the product matrix AB.
z Multiply every element of first row of matrix A with the corresponding element of second
column of B and add them to get the second element of the first row of the product matrix
AB.
z In a similar manner corresponding rows and columns may be multiplied and added to get
the elements.
1 2

0 1
3 4

2 3 5
Eg.: A =
,B=
3 1 2

Since A is of order 2 3 and B is of order 3 2

1 2
2 3 5
2 1 + 3 0 + 5 3 2 2 + 3 1 + 5 4

AB =
0 1 =

3
1
2

3 4
3 1 + 1 0 + 2 3 3 2 + 1 1 + 2 4

AB is of order 2 2.

2 + 0 + 15 4 3 + 20
17 21
=
=

6 1+ 8
3+0+6
9 13

Identity matrix for Multiplication:


z

If I is the unit Matrix and A is any matrix of the same order as that of I then
A I = A = I A.

2 3
E.g Let A =
then A I =
4 6

2 3 1 0 2 1 + 3 0 2 0 + 3 1


=
=
4 6 0 1 4 1 + 6 0 4 0 + 6 1

2 3

=A
4 6

1 0 2 3 1 2 + 0 4 1 3 + 0 6
2 3
And I A =

=
=
=A
0 1 4 6 0 2 + 1 4 0 3 + 1 6
4 6
AI=A=IA
Notes:
In general AB BA i.e. product of matrices is not commutative.
A(BC) = (AB)C i.e. product of matrices is associative.
If A 0 and AB = AC, then it is not necessary that B = C.
If AB = 0, then it is not necessary that A = 0 or B = 0.
If A = 0 or B = 0 then AB = 0 = BA.
i) A (B + C) = AB + AC (in matrices multiplication is distributive over addition.)
ii) (A + B) C = AC + BC.
In the same way, A(B C) = AB AC and (A B) C = AC BC

116

Universal Tutorials X ICSE Maths

Volume 1 of 2

Chapter 12: Matrices

117

SOLVED EXAMPLES 12.3:


2 3
1 2
1) If A =
and B = 3 5 ; find: (i) AB (ii) BA.
4
1

2 3 1 2
2 1 + 3 3 2 2 + 3 5
7 11
Sol: i) AB =

=
=

4 2 + 1 5
4 1 3 5
4 1 + 1 3
7 13
1 2 2 3
1 2 + 2 4 1 3 + 2 1
6 5
ii) BA =

=
=

3 5 4 1
3 2 + 5 4 3 3 + 5 1
14 14
3 2
1 0
2
2
2) Let A =
and B = 1 2 ; find: (i) (A + B) (A B) (ii) A B .
0
5

Is (A + B) (A B) = A2 B2?
3 2
1 0
3 + 1 2 + 0
4 2
Sol: i) Since, A + B =
+
=
=

0 5
1 2
0 + 1 5 + 2
1 7
3 2 1 0
3 1 2 0
2 2
And A B =

=
=

0 5 1 2
0 1 5 2
1 3
4 2 2 2
4 2 + 2 1 4 2 + 2 3
6 14
(A + B) (A B) =

=
=

1 7 1 3
1 2 + 7 1 1 2 + 7 3
5 23
3 2 3 2
3 3 + 2 0 3 2 + 2 5
9 16
ii) Since, A2 = A A =

=
=

0 5 0 5
0 3 + 5 0 0 2 + 5 5
0 25
1 0 1 0
1 1 + 0 1 1 0 + 0 2
1 0
And B2 = B B =

=
=

1 2 1 2
1 1 + 2 1 1 0 + 2 2
3 4
9 16
1 0
8 16
A2 B2 =

=

0 25
3 4
3 21
From the results of parts (i) and (ii) it is clear that: (A + B) (A B) A2 B2
3 8 x
2
3) Given:
y = 8 , find x and y.
9
4



3 8 x
2
Sol:
=
9
4
y


8
3 x 8 y
2

= 3x 8y = 2 and 9x + 4y = 8.
9 x + 4y
8
On solving, we get: x =

2
1
and y = .
3
2

6
3
3) Find the matrix M, such that M
= [2 16]
2 8
Sol: First of all, we must find the order of matrix M.
Let the order of matrix M be a b,
6
3
= [2 16]12
i.e. Mab

2 8 2 2
(1st matrix)
Volume 1 of 2

(2nd matrix)

(resulting matrix)
Universal Tutorials X ICSE Maths

117

118

Since, product of matrices is possible, only when the number of columns in the first matrix is
equal to the number of rows in the second.
b = 2.
Also, the number of rows of product (resulting) matrix is equal to the number of rows of first
matrix.
Let M = [x y]
a = 1 order of matrix M = a b = 1 2.

6
3
[x y]
= [2 16] [3x 2y 6x 8y]

= [2 16] 3x 2y = 2 and 6x 8y = 16
On solving, we get: x = 4 and y = 5
M = [4 5]
8 2
12
4) Given:
X = . Write down: (i) the order of the matrix X (ii) the matrix X.

1 4
10
Sol: i) Let the order of matrix X be a b.
8 2
12
Xab =

1 4 2 2
10 21

a = 2 and b = 1.

The order of the matrix X = a b = 2 1.

8 2 x
12
8 x 2y
12

=
=
1 4 y
10
x + 4y
10

x
ii) Let X =
y
8x 2y = 12 and x + 4y = 10
On solving, we get: x = 2 and y = 2.

x
2
The matrix X = =
y

2

UNSOLVED EXERCISE 12.3:


CW Exercise:
1) Evaluate: If possible,

2 3
ii) [1 2]

1 4
If not possible, give a reason.

2
i) [3 2]
0

6 4 1
iii)

3 1 3

6 4
iv)
[1 3]
3 1

2 1
2
3
5
2) If M =
; find i) M , ii) M and iii) M
1

3) Find x and y, if:


4 3 x 5
y
i)
=
x 2 1
8

2
x 0 1 1
2
ii)

=

3 1 0 y
3 2

2 1

3 2 ;
1 1
i) Write down the product matrix AB

1 2 1
4) If A =
and B =
2 1 3

118

Universal Tutorials X ICSE Maths

Volume 1 of 2

Chapter 12: Matrices

119

ii) Would it be possible to form the product matrix BA? If so, compute BA; if not, give a reason
why it is not possible.

1 2
2
5) If M =
and I is a unit matrix of the same order as that of M; show that, M = 2M + 3I.
2
1

a 0
6) If A =
,B=
0 2

0 b

, M =
1 0

1 1
2

and BA = M , find the values of a and b.


1
1

1 4
1 2
7) If A =
and B =
. Find:
1 3
1 1
ii) A2 + B2
i) (A + B)2
1 4
3 2
8) If A =
,B=
and C =
2
1

4 0

iii) Is (A + B)2 = A2 + B2?

1 0
2

, simplify: A + BC.
0
2

1 1
9) i) Let M
= [1 2] (a) State the order of matrix M (b) Find the matrix M.
0 2
1 4
13
ii) Let
M = (a) State the order of matrix M (b) Find the matrix M.
2 1
5
p
10) Find the positive integers p and q such that: [p q] = [25]
q
HW Exercise:

0 2
1 1
1) If A =
,B=
and I is a unit matrix of order 2 2, find:
5 2
3 2
ii) BA
iii) AI
iv) IB
v) A2
vi) B2A
i) AB
1 3
1 2
4 3
2) If A =
,B=
and C =
, find:
2 4
4 3
1 2
ii) A(BC).
i) (AB)C
Is A(BC) = (AB)C?
0 4 6
3) Given A =
and B =
3 0 1
i) AB

ii) BA

1
0

1 2 , find; if possible:
5 6
iii) A2

4 1
1 0
4) Given A =
and B =
, Find:
2
3

2 1
i) A B
ii) A2

iii) AB

iii) A2 AB + 2B

2 1
1
2
5) Find the matrix A, if B =
and B = B + A.
0
1
2

1 1
2
6) If A =
and A = I; find the values of a and b.
a
b

2 1
2 3
1 4
7) If A =
, B =
and C =
; then show that:
0 0
4 1
0 2
Volume 1 of 2

Universal Tutorials X ICSE Maths

119

120

i) A(B + C) = AB + AC
8) Solve for x and y:

ii) (B A)C = BC AC.

3 1 2
x
ii)
=
2

1
4

2 5 x
7
i)
=
5
2
y


14
iii) [x + y

1 2
x 4]
= [7 11]
2
2

2 x
4 36
2
9) If A =
and B =
; find the value of x, given that: A = B.
0
1
0
1

10) If A and B are any two 2 2 matrices such that AB = BA = B and B is not a zero matrix, what can
you say about the matrix A?

PREVIOUS BOARD QUESTION:


2 1
1) Given
X=
3 4

7
. Write: (i) the order of the matrix X (ii) the matrix X.
6

[2012]

3 1
1 0
2
2) If A =
and B =
, find A 5A + 7I.
1 2
0 1

[2012]

2 5
4 2
t
3) If A =
and B =
and I is the identity matrix of the same order and A is the
1
3

1
3

[2011]
transpose of matrix A, AtB + BI.
3 5
4) If A =
and B =
4 2

2
, is the product AB possible? Give a reason. If yes, find AB.
4

2 cos 60o 4 5
2 cos 0o 5 4

5) Evaluate, 4 sin 30o


sin 90

3 2
6) Given A =
,B=
1 4
2 x x
7) Find x and y, if

y 3y

6
4
, C = and D =
1
5

[2011]
[2010]

2
. Find AB + 2C 4D
2

[2010]

16
3
= .
2
9

[2009]

1 4
3 2
8) If
+ 2M = 3
, find the Matrix M.
2 3
0 3

[2008]

p 0
0 q
9) Given A =
, B =
, C =
0 2
1 0

[2008]

2 2
2

and BA = C . Find the values of p and q.


2
2

3 4
1 y 7 0
10) If 2
+
=
, find the values of x and y.
5 x
0 1 10 5

[2007]

1 0
11) Let A =
, B =
2 1

[2007]

2 3
2
2

Find A + AB + B .

1
0

4 2
12) Let A =
, B =
6 3

0 2
2 3
2

and C =
Find A A + BC.
1
1
1

2 12
4 x
13) Find the value of x given that A2 = B, A =
B=
.
0 1
0 1
120

Universal Tutorials X ICSE Maths

[2006]
[2005]
Volume 1 of 2

Chapter 12: Matrices

121

2 1
3 2
1 0
14) Given A =
, B=
& C=
, find the matrix x. Such that A + x = 2B + C. [2004]
2 0
4 0
0 2

MISCELLANEOUS EXERCISE:
3 2 2x
4
2
1) Find x and y, if:
+ 2 = 4 .
1 4 1
5
y
1 4
2) Find x and y, if: [3x 8]
3[2 7] = 5[3 2y]
3 7
x
2 x
3) If [x y] = [25] and [x y] = [2]; find x and y, if:
y
y
i) x, y W (whole numbers)
ii) x, y Z (integers)
8 2
12
4) Given:
, X = . Write down:
1 4
10
i) the order of matrix X

ii) the matrix X

cos 45 sin 30 sin 45 cos 90


5) Evaluate:

.
2 cos 0 sin 0 sin 90 cot 45
0 1
5
6) If A =
, B = and 3A M = 2B; find matrix M.
4 3
6
a 3
2 b
1 1
5 0
7) If
+

=
, find the values of a, b and c.
4
1
1

2
c

7 3
1 2
2 1
8) If A =
and B =
; find: (i) A(BA)
2 1
1 2

(ii) (AB)B.

x 3 x 2
5
9) Find x and y, if:
=
y 4y 1
12
10) If I is the unit matrix of order 2 2; find matrix M, in each case, given below:
4 5 2 1
i) M 3I =

7 1 3 4

4 3 3 2
ii) 2M + 5I =

0 2 5 4

3 4 2
10
11) If matrix X =
and 2X 3Y = ; find the matrix X and matrix Y.
2 3 2
8
3 0
12) Given A =
,B=
0 4

a b

and that AB = A + B; find the values of a, b and c.


0 c

1 2
1 0
2
2
13) If P =
and Q =
, then compute (i) P Q (ii) (P + Q)(P Q)
2
1
2
1

Is (P + Q)(P Q) = P2 Q2 true for matrix algebra?


4
2 1
3
3 1
14) Given the matrices, A =
,B=
and C =
. Find
4 2
1 2
0 2
i) ABC
ii) ACB. State whether ABC = ACB.

Volume 1 of 2

Universal Tutorials X ICSE Maths

121

122

1 2
6 1
2 3
15) If A =
,B=
&C=
; find each of the following and state if they are equal
1
3 4
1 1
0
ii) A + CB
i) CA + B
2 1
16) If A =
and B =
1 3

; find the matrix X such that AX = B.


11

4 2
17) If A =
, find (A 2I) (A 3I).
1 1
2 1 1
t
t
t
18) If A =
. Find: (i) A A (ii) A A . Where A is the transpose of matrix A.
0
1

4 1
2
19) If M =
, show that: 6M M = 9I; where I is the unit matrix of order 2, i.e. 2 2.
1 2
2 6
20) If P =
and Q =
3 9

3 x

; find x and y such that PQ = null matrix.


y 2

2 cos 60 2 sin 30 cot 45 cosec 30


21) Evaluate without using tables,

.
cos 0 sec 60
sin 90
tan 45
22) State, with reason, whether the following are true and false. A, B & C are matrices of order 2 2.
ii) A B = B A
i) A + B = B + A
iii) (B C) A = B (C A)
iv) (A + B) C = A C + B C
v) A (B C) = A B A C
vi) (A B) C = A C B C
vii) A2 B2 = (A + B) (A B)
viii) (A B)2 = A2 2A B + B2
2 12
23) Find the value of x, given that A2 = B. If A =
,B=
0 1

4 x

.
0 1

3 2 x
5
24) Find x and y if,
= .
0 5 2
y

ANSWERS TO UNSOLVED EXERCISE:


CW Exercise 12.1:
1) i) False, orders of both the matrices are different
ii) True
iii) False, transpose of a 2 1 matrices is a 1 2 matrix
iv) True
v) False, it has only one column & may have many rows.
2) x = 3, y = 1 & z = 3
3) (i) [12, 8] (ii) [ 4, 2]

0 0
2 1 1
4) (i)
(ii)

1 7 7

5 2
10 5
0 1
5) Mt =
(i)
(ii)

3
4

5
8

1 0

2 0
7
6) i) [6, 5],
(iii)
4
4 1

4 3
4 3
2 3
7) i)
(ii)
(iii)

4 4
4 4
0 4

HW Exercise 12.1:
1) (i) a = 3, b = 8, c = 3 (ii) a = 3, b = 4, c = 2
3) (i) x = 4 & y = 0 (ii) x = 4 & y = 5
122

7
4
3
7
2) (i) (ii) (iii) (iv)
2
7
11
1
4) (i) [3 5] (ii) [3 9]

Universal Tutorials X ICSE Maths

Volume 1 of 2

Chapter 12: Matrices

123

CW Exercise 12.2:

7 14
1 3
34
1) (i) [15 6] (ii)
(iii)
(iv)

7
0
9 6
14
2 0
3)

1 1

2) (i) x = 2 & y = 1 (ii) x = 1 & y = 2.5

21 3
15 12
3 18
2 2
4) (i)
(ii)
(iii) 2 3 (iv)

18 45
12 9
6 6
2

1 0
1 4
5) (i)
(ii)

12 5
0 3
HW Exercise 12.2:
2 2
5 2
1) (i)
(ii)

2 2

2 7
4 1
2) (i)
(ii)

1
4

3 2

3) x = 8; y = 5; z = 9

4
3
0 3
4) (i)
(ii)
5
1

1
7

3 (iii) 3
4
1

3
3

1
3
2

CW Exercise 12.3:

6
1) (i) [6] (ii) [0 5] (iii) (iv) Not possible as the number of columns in first matrix is not equal
6
to number of rows in second matrix
5
5 0
10
50 25
3
5
2) M2 =
; M =
;M =

0 5
5 10
25 50
3) (i) x = 2, y = 26 (ii) x = 2, y = 1

3 2
4) (i)
(ii) Yes; BA is possible BA =
10 7

4 3 5

7 4 9
3 1 4

6) a = 2 and b = 1

4 12
4 8
7) (i)
(ii)
(iii) No
0 16
2 12
6 12
8)

8 9

9) a) i) 1 2 (ii) 1

1
(b) i) 2 1 (ii)
2

1

3

10) p = 3 & q = 4 or p = 4 & q = 3

HW Exercise 12.3:

4
6
5 4
1) (i)
(ii)
(iii) Matrix A (iv) Matrix B (v)

10 2

10 4
15 2

(vi)

10
14

5 16

63 61
63 61
2) (i)
(ii)
; yes
88 86
88 86
1
0
3
34 28

3) (i)
4
8 (iii) Not possible
(ii) 6
9
5
18 20 24
Volume 1 of 2

Universal Tutorials X ICSE Maths

123

124

3 1
18 7
2 1
18 6
4) (i)
(ii)
(iii)
(iv)

4 2
14 11
4 3
14 10
6) a = 0; b = 1
9) x = 12
Previous Board Question:

8) (i) x = 4; y = 3 (ii) x = 10, y = 8 (iii) x =

4 4
5)

0 0
5
3
;y=
2
2

10) A is the identity matrix

2
0 0
1) order is 2 1; X = 2)

3
0 0

11 3
3)

16 2

26
4) Yes,
0

13 14
5)
14 13

0
6)
0

7) x = 2, y = 1

4 1
8) M =

1 6

9) p = 8, q = 4

10)x = 2, y = 8

4 9
11)

5 4

2 2
12)
13) x = 36
3 4

1) x = 3, y = 2

2) x = 1, y = 6.5

3) (i) x = 3, y = 4 (ii) x = 3; y = 4

2
4) (i) 2 1 (ii)
2

1 0.5
5)

1 0

72
6) M = 10

7 5
14) x =

6 2
Miscellaneous:

14 13
13 14
8) (i)
(ii)

13 14
14 13

9) x = 1, y = 2

20 24
16 10
10) i)
(ii)

17 8
5 6.5
12) a=

7) a = 4, b = 2 & c = 4

14
1 38
11) x =
y=

3 28
10
4 0
0 0
13) (i)
(ii)
; No
4 4
0 8

3
4
,b=0&c=
2
3

15 7
18 24
14) (i)
(ii)
; Not equal
30 14
36 48

5 15
14 3
15) (i)
(ii)
; Not equal
5
5
4
4

4
16)
5

2 2
4
6 3

2 3 (ii)
18) (i) 2

3 5
2 3 5

4 0
17)

0 4

1 1
21)

1 1
22) i) True; addition of matrices is commutative
ii) False, subtraction of matrices not commutative
iii) True; multiplication of matrices is associative.
iv to vi) True, multiplication of matrices is distributive over addition
vii to viii) False, Laws of algebra or factorization & expansion are not applicable to matrices
23) 36
24) x = 3, y = 10
20) x = 6 & y = 1

124

Universal Tutorials X ICSE Maths

Volume 1 of 2

You might also like